You are on page 1of 394

A Unit of adda247.

com

PUZZLE-1
Directions (1-5): Study the following information carefully and answer the given
questions.

Eight friends Venkat, Abdulla, Rakshan, Vikram, Prakash, Suman, John and
Manoj live on eight different floors of a building but not necessarily in the same
order. The lowermost floor of the building is numbered 1 and the topmost floor of
the building is numbered 8. Each of them likes different teams of Champions trophy
cricket viz, England Team, India Team, Australia Team, Bangladesh Team, New
Zealand Team, Pakistan Team, South Africa Team and Sri Lanka Team but not
necessarily in the same order.

The one who likes India Team lives on an even-numbered floor but not on the
topmost floor. Only one person lives between Suman and the one who likes New
Zealand Team. Only two persons live between Suman and the one who likes India
Team. Neither Prakash nor Rakshan lives on the first floor. Only one person lives
between Rakshan and the one who likes Australia Team. Venkat lives just above
Suman. Only two persons live between Prakash and Venkat. The one who likes
New Zealand Team does not live on floor number one, Abdulla lives on an even-
numbered floor and just above Rakshan. The one who likes Bangladesh Team lives
on an even numbered floor and lives just above the person who likes Sri Lanka
Team. Rakshan does not like New Zealand Team or Sri Lanka Team. Only two
persons live between the one who likes Pakistan Team and the one who likes
England Team. Vikram does not like South Africa Team. The one who likes
Pakistan Team does not live on an odd-numbered floor. Manoj lives just below the
one who likes Sri Lanka Team.
1). Who lives on fifth floor?
Abdulla
Prakash
The one who likes Bangladesh Team
e) Both option b and d

2). John likes which of the following team?


South Africa Team
Australia Team
Bangladesh Team
Sri Lanka Team
England Team

3). How many persons are between the one who likes Bangladesh Team and
the one who stays on 4th floor?
One
Two
Three
Four
Five

4). Four of the following five are alike in a certain way and hence they form a group.
Which one of the following does not belong to that group?
Venkat
Rakshan
Manoj
Abdulla
John

5). Which of the following combinations is true?


1 – Vikram - South Africa Team
4 – Prakash - India Team
3 – Suman - Sri Lanka Team
6 – Manoj - Pakistan Team
7 – Rakshan - England Team
PUZZLE-2
Directions (6-10): Study the following information carefully and answer the given
questions.

Varun, Rajesh, Sunil, Jeeva, Vasanth, Parthiban and Avinash are seven
persons from different States, viz Gujarat, Bihar, Maharashtra, Haryana, Punjab,
Assam and Rajasthan. They go to top 4 cities in India, viz. Mumbai, Hyderabad,
Bangalore and Chennai only on Friday but not necessarily in the same order. At
least one Person goes to one city, but no city is visited by more than two Persons.
Parthiban, who is from Maharashtra, goes alone to Bangalore. The one who is from
Haryana does not go to Mumbai. Also, he never goes either with Vasanth or with
Avinash. Jeeva goes to Chennai with the person who is the Person from Rajasthan.
Sunil goes to Hyderabad. Avinash is not a Person from Rajasthan. The one who is
from Haryana goes to the city with the person who is the Person of Gujarat. The
one who is from Punjab goes to Chennai. Varun is a Person from neither Bihar nor
Rajasthan. The one who is from Assam goes to Mumbai neither with Vasanth nor
with Jeeva. The person who is from Bihar goes to the city with Varun.

6). Who among the following persons visit Hyderabad?


Jeeva and Rajesh
Vasanth and Sunil
Rajesh and Sunil
Parthiban and Rajesh
Cannot be determined

7). Rajesh is the person from which of the following state?


Haryana
Gujarat
Bihar
Rajasthan
e) Either Haryana/Gujarat

8). Which of the following city visited by Avinash?


Bangalore
Mumbai
Chennai
Hyderabad
None of these

9). Which of the following statements is wrong?


Jeeva – Hyderabad - Punjab
Vasanth – Chennai - Rajasthan
Parthiban – Bangalore - Maharashtra
Varun – Mumbai - Assam
All statements are true

10). Which of the following combinations is true?


Jeeva – Chennai - Bihar
Varun – Hyderabad - Assam
Vasanth – Chennai - Punjab
Avinash – Mumbai - Bihar
None of these

Explanation With Answer Key:


Directions (Q. 1-5):
1). Answer: e)
2). Answer: a)
3). Answer: c)
4). Answer: b)
5). Answer: d)

Directions (Q. 6-10):

6). Answer: c)
7). Answer: e)
8). Answer: b)
9). Answer: a)
10). Answer: d)

PUZZLE-3
Directions (Q. 6-11): Study the following information carefully and answer
the questions given below:
Seven employees H, I, J, K, L, M and N working in different companies
reach their offices every day by a train which stops at five stations A, B, C, D and E
respectively after leaving station Z.
Three of them board the train at station Z.
K deboards at the station next to the station at which M
deboards. I does not deboard either with H or with L.
N alone boards at station C and deboards with J after passing one station.
None of them boards at station B and none of them deboards the train at station A.
J boards with M but does not board with either I or with K.
L boards with two others and deboards alone after K.
I and K deboard together at station C.
H travels between two consecutive stations and deboard at the station
E. M does not board after L.

6). At which of the following stations does L deboard?


B
D
C
Can't be determined
None of these

7). At which of the following stations do J and M board the train?


A
C
D
E
None of these

8). After how many stations does L deboard?.


One
Two
Three
Four
None of these

9). How many persons deboard at station E?


None
One
Two
Three
None of these

10). H deboards the train how many stations after I deboards?


One
Two
Three
None
None of these

11). Who among the following board at station A?


I,K
M, L
K,J
L, M, J
None of these
Answers:
6)b 7)e 8)d 9)d 10)b 11)a

Explanation:
Directions (Q. 6-10):

PUZZLE-4
Directions (1-5): Study the following information carefully and answer the
given questions.
In HDFC Bank the staffs Gopinath, Sanjay, Anwar, Latif, Jemima, Shalini,
Monica, Mishraya are working and they come to the Bank by different express such
as Shiv Ganga Express, Malwa Express, Suryanagri Express, Magadh Express,
Pandian Express, Nellai Express, Archana Express and Vaishali Express not in the
respective order. And they are selected for special seminar. They planned to take
seminar individually with other staffs in different States such as Bihar, Haryana,
Rajasthan, Kerala, Karnataka, Maharashtra, Assam and Gujarat in a week starting
from Sunday to Saturday. The maximum number of seminar taken by each person
is one.
Gopinath going to take seminar before Sanjay and after Mishraya who did not
come to the bank by Suryanagri Express. The staff who is going to take seminar in
Rajasthan, come to Bank by Nellai Express. Anwar come to Bank by Malwa Express
and he did not take seminar on week’s starting day. The staff who take seminar in
Gujarat, attend seminar in Maharashtra and Karnataka in the previous day. Mishraya
likes to take seminar in Bihar and she did not come by Archana Express. Only two
persons giving their seminar on same day due to other commitments. The person who
is coming to Bank by the way of Pandian Express likes to take seminar on Thursday.
Sanjay likes to take seminar in Assam and the day in between the persons who take
seminar in Kerala and Gujarat. Monica comes to Bank by Archana Express and she did
not take seminar on Saturday. The persons who come to the bank by Magadh Express
and Malwa Express likes to take seminar on same day. The person who takes seminar
in Assam come to the bank by Shiv Ganga Express and attend Jemima’s seminar who
come to Bank by Pandian Express in the next day. Gopinath who did not come to the
bank by Vaishali Express taking seminar on the next day of Latif’s seminar. Latif who
making arrangement with another staff for taking seminar on same day and he did not
take seminar in Maharashtra.

1). Latif comes to the Bank by which express?


Magadh Express
Shiv Ganga Express
Pandian Express
Nellai Express
Malwa Express

2). Who takes the seminar on Friday?


The one who takes seminar in Haryana
Monica
Both a and b
The one who takes seminar in Assam
The one who come to the Bank by Shiv Ganga Express
3). Who come to the Bank by Nellai Express?
Mishraya
Latif
Monica
Shalini
Jemima

4). Two seminars held on which of the following day?


Thursday
Tuesday
Saturday
Friday
Monday

5). Which of the following combinations is true?


Mishraya - Archana Express – Bihar - Thursday
Sanjay - Shiv Ganga Express – Assam - Wednesday
Gopinath - Suryanagri Express – Kerala - Wednesday
Shalini - Vaishali Express – Rajasthan - Saturday
None of these

PUZZLE-5
Directions (6-10): Study the following information carefully and answer the given
questions.
Eight persons Akram, Manish, Vinay, Mishra, Harshan, Irfan, Ramkumar and Santosh
are seated around a circular table such that four faces the centre and four are facing
opposite to the centre. They like different IPL Cricket teams: Kolkata Knight Riders,
Rising Pune Giants, Gujarat Lions, Mumbai Indians, Sunrisers
Hyderabad, Delhi Daredevils, Kings XI Punjab and Royal Challengers Bangalore
not necessarily in the same order. The arrangement is based on the following rules:
One who likes Royal Challengers Bangalore is seated opposite to Akram. Mishra is
3rd to the right of the one who likes Delhi Daredevils who is seated between Manish
and the one who likes Gujarat Lions. Ramkumar is 3rd to the left of Vinay who likes
Kings XI Punjab. Santosh is to the immediate left of the one who likes Sunrisers
Hyderabad. One who likes Royal Challengers Bangalore is a neighbour of Mishra.
Akram does not like Delhi Daredevils and Ramkumar does not like Royal
Challengers Bangalore. Akram is 2nd to the left of Harshan who is not a neighbour
of Mishra. Either Akram or Ramkumar likes Sunrisers Hyderabad. Mishra and
Manish are seated facing the same direction.. The one who likes Gujarat Lions is
2nd to the right of the one who likes Rising Pune Giants. Mishra and Santosh do not
like Rising Pune Giants, Ramkumar faces outside the centre. One who likes
Mumbai Indians is facing the opposite direction faced by Vinay. Not more than two
persons who sits on consecutive seats face the same direction.

6). Who among the following is not facing centre?


Vinay
Harshan
Akram
Santosh
Irfan

7). Who among the following likes Delhi Daredevils?


Akram
Santosh
Irfan
Harshan
None of these
8). Who is sitting third to the right of the person who like Kings XI Punjab team?
The one who likes Gujarat Lions team
The one who likes Rising Pune Giants team
The one who likes Royal Challengers Bangalore team
The one who likes Delhi Daredevils team
None of these

9). The only person who is sitting between Santosh and Ramkumar likes which
of the following teams?
Gujarat Lions
Mumbai Indians
Sunrisers Hyderabad
Kings XI Punjab
Kolkata Knight Riders

10). Which of the following is correctly matched?


Mishra – Facing Centre – Kolkata Knight Riders
Santosh - Facing Outside – Gujarat Lions
Irfan – Facing Outside – Royal Challengers Bangalore
Vinay – Facing Centre – Mumbai Indians
Manish – Facing Outside – Rising Pune Giants

Explanation With Answer Key:


Directions (Q. 1-5):
1). Answer: a)
2). Answer: c)
3). Answer: d)
4). Answer: e)
5). Answer: b)

Directions (Q. 6-10):


6). Answer: a)
7). Answer: d)
8). Answer: b)
9). Answer: c)
10). Answer: e)

PUZZLE-6
Directions (1-5): Study the following information carefully and answer the
given questions.
Nine people Sandeep, Harish, Lenin, Ravi, Abbas, Dinesh, Nisar, Venkat and
Sumanth stay in a building. The building has nine floors and only one person stays
on one floor. Each of them likes different colour uniforms namely- Red, Blue,
Green, Yellow, White, Orange, Brown, Grey and Violet. Each person belongs to
different companies, i.e. Samsung, Onida, Panasonic, Videocon, LG, Whirlpool,
Toshiba, Sharp and Sony but not necessarily in the same order. The ground floor is
numbered 1, the floor above it is numbered 2, and so on, and the topmost floor is
numbered 9.

The one who belongs to Sharp Company stays on the 4th floor. There are 4 floors
between the floors on which Ravi & Nisar stay with Ravi living above Nisar. Lenin stays
on the floor immediately above Venkat's floor. The one who likes Red colour uniform
stays on an even numbered floor and is a neighbour of the one who likes Blue colour
uniform. Sumanth does not belong to Onida company. The one who belongs to
Samsung company stays on the topmost floor. The one who likes Brown colour
uniform stays immediately below the one who likes White colour uniform. Abbas likes
Grey colour uniform. Dinesh belongs to LG company and stays on 8th floor. There are
3 persons between the one who likes Yellow colour uniform and the one who like Grey
colour uniform. There are two floors between the floors on which people from Sony
company and Sharp company stay. Abbas stays immediately
below the floor on which the person from Videocon company lives. The person who
likes Orange colour uniform belongs to Onida company and stays on an odd
numbered floor. Sumanth stays on the bottommost floor. The person who likes
Green colour uniform is from Videocon company and stays on 6th floor. The
person from Toshiba company stays on 2nd floor. The persons from Whirlpool
company and Onida company are not the neighbours of the person from Videocon
company. Ravi likes Blue colour uniform and Nisar does not like Red colour
uniform. Harish does not live below the floor of Sandeep.

1). Which of the following persons sits exactly between Sumanth and the one who
likes Violet colour uniform?
Venkat and Nisar
Lenin and Abbas
Nisar and Ravi
Sandeep and Venkat
Harish and Abbas

2). Who lives in the Third floor?


Abbas
The one who likes Violet colour uniform
The one who is from Onida company
Dinesh
None of these

3). Green Colour uniform like by which of the following person?


Ravi
Sandeep
Nisar
Sumanth
Lenin
4). How many persons sit between Nisar and the one who likes Yellow
colour uniform?
Five
Three
Four
Two
Six

5). Which of the following combinations is true?


4 - Sharp - Lenin - Yellow colour uniform
3 –Onida – Nisar - White colour uniform
8 - Samsung – Harish - Green colour uniform
7 - Sony - Ravi - Blue colour uniform
None of these

PUZZLE-7
Directions (6-10): Study the following information carefully and answer the given
questions.
Nine persons Subash, Kumaran, Tharoon, Ajay, Yadav, Praveen, Achuthan, Niraj
and Ashok are sitting in a row facing in either north or south direction but not
necessarily in the same order. They work in different countries they are Australia,
India, Russia, USA, Japan, Kuwait, Canada, Thailand and UK but not necessarily
in this order. The following information is known about them.

Praveen and Ashok are immediate neighbours of Tharoon but face in opposite
directions to each other. Achuthan and Niraj face in the same direction. Subash
who works in Japan sits in the middle and faces towards south. The person working
in Kuwait sits to the immediate right of a person facing in north direction. The
person sitting at first and last position from left face in north direction and they work
in Canada and Russia respectively. The person working in Thailand sits to the
immediate right of Kumaran. Praveen does not sit with Subash. Yadav sits second
to the left of Ajay but does not work in Russia. Niraj sits two places away from
Subash. Ajay sits exactly between Achuthan and Niraj. Also, there is only one
person sitting between Achuthan and Niraj. Kumaran and Tharoon face in the
same direction. The person who works in India sits fourth to the left of the person
working in Kuwait but neither of them is Niraj. The persons working in UK and
Australia are sitting adjacent to each other. The person working in Australia sits
fourth to the right of Praveen.

6). Who works in Australia Country?


Kumaran
Praveen
Niraj
Ajay
Yadav

7). In which of the following country Achuthan works?


USA
Kuwait
Thailand
UK
Russia

8). Which of the following persons sits at the extreme ends of row?
Ajay and Tharoon
Kumaran and Ashok
Niraj and Achuthan
Ashok and Yadav
Yadav and Subash

9). The one who works in Japan sits third to the left of
______________ a) Achuthan
Praveen
The one who works in UK
The one who works in India
Both option c and d

10). Which of the following combinations is true?


Achuthan - Kuwait – Facing South
Tharoon – India – Facing South
Yadav – Australia – Facing North
Ajay – Canada – Facing South
None of these

Explanation With Answer Key:


Directions (Q. 1-5):

1). Answer: a)
2). Answer: c)
3). Answer: b)
4). Answer: e)
5). Answer: d)
Directions (Q. 6-10):

6). Answer: d)
7). Answer: b)
8). Answer: d)
9). Answer: e)
10). Answer: a)

PUZZLE-8
Directions (1-5): Study the following information carefully and answer the
given questions.

Twelve friends are sitting in two parallel rows of chairs containing six people
each, in such a way that there is equal distance between adjacent persons. In row 1:
Dinesh, Vignesh, Rakesh, Charan, Eshwar and Karthik are seated and all of them
are facing south. In row 2: Hafiz, Jaypal, Naren, Praveen, Murugan and Arjun are
seated and all of them are facing north. Each of them likes different car such as,
Maruti Suzuki, Hyundai, Honda, Toyota, Tata, Ford, Chevrolet, Volkswagen,
Mahindra, Nissan, Skoda and Fiat but not necessarily in the same order. Each of
them like different players: Roger Federer, Rafael Nadal, Novak Djokovic, Andy
Murray, Stan Wawrinka, Milos Raonic, Kei Nishikori, David Ferrer, Fernando
Verdasco, Jack Sock, Dominic Thiem and Pete Sampras. In the given seating
arrangement, each member seated in a row faces another member of the other row.
Eshwar, whose favourite player is Milos Raonic sits third to the left of the person
who likes Hyundai and not sitting at the extreme ends, Vignesh, who likes Toyota,
does not sit at an extreme end of the row. If Eshwar sits at an extreme end then
Vignesh doesn’t sit to the immediate right of the one who likes Hyundai. Naren and
Hafiz face the persons whose favourite players are Milos Raonic and Stan
Wawrinka respectively. There are two persons sitting on the right side of Jaypal,
who sits second to the right of the person, whose favourite player is Jack Sock.
Jaypal, who likes Tata, faces the immediate neighbour of Dinesh. Dinesh likes
Honda. Hafiz sits with the persons who like Tata and Fiat. Karthik faces the person
who likes Skoda car and Pete Sampras player but Karthik does not like Hyundai.
The one who likes Fiat car doesn’t sit with the one who likes Skoda car. The person,
who likes Nissan car, sits second to the right of the person who likes Ford. Praveen
does not like Skoda and faces the person, who likes Maruti Suzuki. The person who
likes Fernando Verdasco is an immediate neighbour of the person who likes
Volkswagen. Rakesh’s favourite player is Andy Murray and he doesn’t sit with
Dinesh. Naren does not like Rafael Nadal player. The person, whose favourite
player is Roger Federer, faces the person, who sits second to the right of the
person, whose favourite player is Stan Wawrinka. The person whose favourite
player is David Ferrer does not sit in row 2 and faces the person who likes Dominic
Thiem. Murugan faces the person who likes Kei Nishikori. Rakesh doesn’t face the
person who likes Ford car. The one who likes Chevrolet sits in Row 2.

1). Who likes the Jack Sock player?


Murugan
Hafiz
Arjun
Naren
Praveen
2). The one who likes Kei Nishikori sits third to the right of ______________
Dinesh
Vignesh
Eshwar
Karthik
Rakesh

3). Which of the following car likes by Hafiz?


Skoda
Hyundai
Honda
Ford
Volkswagen

4). Which of the following persons sits at the extreme ends of one row?
Praveen and Hafiz
Murugan and Naren
Charan and Karthik
Dinesh and Eshwar
Praveen and Arjun

5). Which of the following combinations is true?


Karthik – Maruti Suzuki - Novak Djokovic
Eshwar – Ford – Milos Raonic
Naren – Nissan – Stan Wawrinka
Vignesh – Toyota - David Ferrer
None of these
PUZZLE-9
Directions (6-10): Study the following information carefully and answer the given
questions.

The seven foreign banks Deutsche Bank, Doha bank, Barclays Bank, Citi Bank,
Standard Chartered Bank, Royal Bank of Canada, HSBC Bank are having their
headquarters in different places namely USA, Canada, China, Japan, Russia, UK
and Qatar not in the respective order. And the banks making contract with
insurance companies such as New India Assurance, The Oriental Insurance
Company, United India Insurance Company, National Insurance Company, Export
Credit Guarantee Corporation Of India, General Insurance Corporation and Life
Insurance Corporation. The insurance companies are located in different places
such as Chennai, Hyderabad, Bangalore, Delhi, Mumbai, Kolkata and Patna not in
the respective order.

Life Insurance Corporation and National Insurance Company are not contact
with Canada and UK based banks respectively. Either Deutsche Bank or HSBC
Bank likes to contract with United India Insurance Company which is not located
in Kolkata and Hyderabad
The insurance company which located in Mumbai contract with the bank that
is located in Japan.
Royal Bank of Canada contract with Hyderabad based insurance company and
is not located in Russia and Canada.
General Insurance Corporation is Bangalore based insurance company. And Kolkata
based insurance company not deals with HSBC Bank and Deutsche Bank.

USA and China based banks making contract with Delhi and Patna based
insurance companies respectively.
Deutsche Bank is not contracting with the Delhi and Mumbai based
insurance companies. Barclays Bank is having headquarters in Qatar.
Citi Bank do not contract with insurance companies located in Chennai, Mumbai
and also General Insurance Corporation Company
The Mumbai based The Oriental Insurance Company does not contract with
Standard Chartered Bank and Doha bank. HSBC Bank which has headquarters in
Japan doesn’t contract with Chennai based New India Assurance company.
General Insurance Corporation will contract with Qatar based bank.
Chennai based insurance company making contract with Standard Chartered
Bank which not have headquarters in Canada and UK
Export Credit Guarantee Corporation of India is not located in Patna and
Kolkata and makes deal with Doha bank Company

6). Which of the following bank headquarters located in Russia?


HSBC Bank
Barclays Bank
Royal Bank of Canada
Doha bank
Standard Chartered Bank

7). Citi Bank Contract with which of the following insurance company?
Export Credit Guarantee Corporation of India
National Insurance Company
United India Insurance Company
Life Insurance Corporation
New India Assurance

8). The Oriental Insurance Company contract with which of the following banks?
Barclays Bank
Royal Bank of Canada
Standard Chartered Bank
HSBC Bank
Doha bank
9). If ‘Deutsche Bank’ is related to ‘Kolkata’, ‘Citi Bank’ is related to
‘Hyderabad’, then which following is ‘Royal Bank of Canada’ related to?
Mumbai
Chennai
Delhi
Bangalore
Patna

10). Which of the following combinations is true?


Doha Bank – Russia – Export Credit Guarantee Corporation Of India - Chennai
Deutsche Bank – Japan – The Oriental Insurance Company - Mumbai
Citi Bank - Canada – National Insurance Company - Kolkata
Standard Chartered Bank – USA – New India Assurance - Delhi
None of these

Explanation With Answer Key:


Directions (Q. 1-5):

1). Answer: a)
2). Answer: c)
3). Answer: d)
4). Answer: e)
5). Answer: d)

Directions (Q. 6-10):

6). Answer: e)
7). Answer: b)
8). Answer: d)
9). Answer: a)
10). Answer: c)

PUZZLE-10
Directions (Q. 1-5): Study the following information carefully to answer the given questions:

There are six persons- Janani, Vimala, Ramya, Bharati, Hasina and Narmada living in a
triple floor building with six flats. The floors are First, Second and Third, each having two
flats. Those who have four or more degree occupy the Third floor and using different
Devices. The one who work in Yes Bank uses Mobile and she is not Narmada while the one
who work in Federal Bank uses Tablet. Both of them occupied same floor. Vimala who work
in HDFC Bank has three degree and not stayed in the Third floor. Ramya, an unmarried
woman, does not have any degree, occupies the Second floor. Hasina and Narmada have
Luxury watches while the rest have Sports watches. Bharati, a Second floor occupant, uses
Computer. One of the two persons having two degree uses Laptop and work in Axis Bank
also occupied in first floor. Two persons, of whom one work in ICICI Bank, do not using any
Devices.

1). What is the least number of degrees had by all the persons in the entire building?
14
16
15
17
Cannot be determined

2). Who among the following occupies the Third floor?


Janani
Hasina
Ramya
Bharati
None of these

3). In which Bank does Bharati work in?


ICICI Bank
Yes Bank
Axis Bank
Federal Bank
Cannot be determined

4). How many degrees does Janani have?


None
One
Two
Three
None of these
5). Which of the following device Ramya Using?
Tablet
Mobile
Tablet or Mobile
No Device

Computer

PUZZLE-11
Directions (Q. 6-10): Study the following information carefully and answer the
given questions

Eight friends- Vishnu, Suresh, Aravind, Selvam, Prem, Johnson, Deepak and Mahesh visit
different countries viz. Japan, Singapore, Russia and Switzerland are sitting around a
circular table facing the centre of the table. Each country is being visited by two people
only, but not necessarily in the same order. All these friends have different mobiles i.e. -
Samsung, Microsoft, Vivo, Oppo, Motorola, Redmi, HTC and Lenovo. No two people
visiting the same country are sitting adjacent to each other except those visiting Singapore.
The person who has Oppo mobile is sitting on the immediate left of the person who has
Lenovo mobile. Prem neither has Lenovo nor Vivo mobile. Mahesh has Redmi mobile and
visits Russia and is sitting to the immediate left of Suresh, who visits Singapore. Suresh
does not have Vivo mobile. Johnson has Motorola mobile and visits Singapore, who is
sitting opposite to Deepak. Only Vishnu, who has Microsoft mobile, is sitting between
Deepak, who has Oppo mobile and the person who has HTC mobile. Persons who visit
Japan are sitting opposite to each other. Each of the persons who visit Switzerland is sitting
adjacent to a person who visits Japan. Selvam does not have Lenovo mobile.

6). Which mobile is owned by Aravind?


Lenovo
Vivo
Samsung
Microsoft
None of these
7). Who has a Samsung mobile?
Suresh
Selvam
Aravind
Prem
None of these

8). Which of the following pair is sitting adjacent to the person who has Motorola mobile?
Suresh and Mahesh
Prem and Selvam
Selvam and Mahesh
Mahesh and Aravind
None of these

9). Who among the following visit Japan?


Aravind and Mahesh
Prem and Vishnu
Selvam and Aravind
Vishnu and Johnson
None of these

10). Who is sitting opposite to the person who has Redmi mobile?
The person who has Oppo mobile
The person who has HTC mobile
The person who has Microsoft mobile
The person who has Vivo mobile
None of these
ANSWERS - a,b,e,c,d,a,a,e,c,b
PUZZLE-12
Directions (Q. 6-11): Study the following information carefully and answer the
questions below:
A, B, C, D, E, F, G and H are sitting around a circular table facing the
centre. Each of them has a mobile of a different company viz, Samsung, Nokia,
LG, HTC, Lenovo, Moto C, Apple and Intex, but not necessarily in the same order.
Only two persons sit between G and the one who has
Apple. Only one person sits between E and the one who has
Nokia. B sits second to the left of the one who has Nokia.
H is an immediate neighbour of D. The one who has LG sits second to the right of
D is not an immediate neighbour of B or E and he does not have
Apple. E sits second to the left of the one who has Apple.
Only three people sit between the one who has LG and the one who has HTC. F
sits second to the left of the one who has HTC.
H does not have Lenovo. The one who has Lenovo is not an immediate
neighbour of G or H.
C sits on the immediate left of the one who has Lenovo.
The one who has Intex sits on the immediate right of the one who has Samsung.
6). Who among the following has Moto G?
A
B
H
C
None of these

7). Who among the following sits exactly between B and the one who is on
the immediate right of A?
The one who has Lenovo
The one who is opposite D
The one who has Intex
The one who has Moto G
None of these

8). How many persons sit between the one who has Samsung and the one who
has Lenovo?
One
Two
Three
Four
None of these

9). Which of the following statements is/are true?


Only one person sits between C and G.
The one who has HTC sits second to the left of C.
G has Intex.
The one who has Samsung sits second to the left of A.
None of these

10). C has which of the following mobiles?


Apple
Lenovo
Moto G
Nokia
None of these

11). What is the position of G with respect to the one who has Intex?
Second to the left
Third to the right
Third to the left
Fifth to the left
None of these

Explanation With Answer Key:

Directions (Q. 6-10):

6). B)
7). A)
8). C)
9). D)
10). A)
11). C)

PUZZLE-13
Directions (Q. 1-5): Study the following information carefully to answer the
given questions:
There are seven people Naveen, Kailash, Laxman, Lokesh, Santhosh,
Abinash and Udayan. They like different Sims, Wonders and were born in different
countries and months. They all were born in the same year. One of the Sim is JIO.
One of the Countries is Poland. One of the Wonders is Machu Picchu. One of the
persons was born in the month of August. The following information is known about
them. Abinash was born in India or in Malaysia. The one who was born in the
month of April likes Great Pyramid of Giza and Vodafone sim and is either Laxman
or Udayan. Lokesh is not the one who likes Petra. The one who likes Colosseum
likes Idea sim. The one who was born in Malaysia is the youngest one and is either
Kailash or Udayan. The one who likes Christ the Redeemer was born in Singapore
but is not Kailash. Santhosh was born in the month of January. Lokesh was born in
Bangladesh in the month of June and likes Docomo sim. The one who likes Taj
Mahal was born in October but is not the youngest person. Kailash likes Aircel sim.
The one who likes Colosseum was born in China but that person is not Laxman
who likes BSNL sim. The one who was born in Russia likes Airtel sim but is not
Santhosh. Abinash is the second youngest person. The one who likes Great Wall of
China is one month younger than Santhosh.

1). Who likes JIO sim?


Abinash
Udayan
Naveen
Kailash
Laxman

2). Which of the following Wonders does Lokesh like?


Taj Mahal
Machu Picchu
Great Pyramid of Giza
Great Wall of China
Christ the Redeemer

3). Which month Laxman was born?


February
April
August
October
Cannot be determined

4). Who was born in Poland?


Santhosh
Naveen
Kailash
Udayan
Laxman

5). Which of the following statements is correct?


Santhosh likes Machu Picchu Bank
Naveen likes Vodafone sim
Udayan was born in the month of August
Kailash was born in India
None of these

PUZZLE-14
Directions (Q. 6-10): Study the following information carefully and answer the given
questions.

E, F, G, H, I, J, K and L are eight family members sitting around a circular table


in a restaurant. All of them are facing the centre. Three of the family members are
engineering graduates and others are arts graduates. All the eight family members are
bankers. They are working in different banks Andra Bank, Dena Bank, Syndicate Bank,
Bank of India, Vijaya Bank, UCO Bank, Canara Bank and Allahabad Bank, but not
necessarily in the same order. There are two married couples, three brothers, two
daughters, one granddaughter and one grandson in the
group. Two of the female members in the group E and L get same degree. They
studied different degrees in colleges, they are B.com, BCA, M.com, BE, MBA,
ME and B.sc.

K, works in Andra Bank, is the head of the family, and is immediate


neighbour of L and H. The granddaughter J is sitting third to the left of K. The
grandmother of J is sitting on the immediate left of her. J is arts graduate. I is sitting
second to the right of his father K. He works in Vijaya Bank, and studied B.sc. Only
K’s daughter is sitting exactly between E and I. She studied MBA and she works in
UCO Bank. L works in Syndicate Bank and F works in Bank of India. The son of L is
sitting on the immediate left of his father. His father works in Dena Bank. G works in
UCO Bank and is sitting on the immediate left of her mother. H, the husband of the
one who works in Syndicate Bank, studied ME and his mother studied BE. The one
who works in Bank of India sits second to the right of E, who is an immediate
neighbour of the one who works in UCO Bank and the Canara Bank. The one who
works in Canara Bank does not studied B.com and the one who works in Andra
Bank does not studied M.com or BCA.

6). The only person who is sitting between J and G is working on which of
the following bank?
Dena Bank
Bank of India
Andra Bank
Syndicate Bank
Allahabad Bank

7). Who among the following working in Canara Bank?


J
H
G
E
e) None of these

8). Who among the following is studied B.com degree?


J
K
E
F
None of these

9). The person who studied BE is working in which of the following banks?
Allahabad Bank
Bank of India
Syndicate Bank
Both Allahabad Bank and Syndicate Bank
Canara Bank

10). Which of the following is correctly matched?


F – Bank of India – M.com
I – Allahabad Bank – B.sc
H – Dena Bank - ME
G – UCO Bank – ME
None of these

Explanation With Answer Key:


Directions (Q. 1-5):
1). Answer: a)
2). Answer: b)
3). Answer: c)
4). Answer: d)
5). Answer: e)

Directions (Q. 6-10):

Family Tree:
Male (+); Female (-)
6). Answer: e)
7). Answer: a)
8). Answer: b)
9). Answer: d)
10). Answer: c)

PUZZLE-15
Directions (Q. 1-6): Study the following information carefully and answer
the questions given below:
Eight people P, Q, R, S, T, U, V and W are sitting around a circular table
facing the centre, but not necessarily in the same order. Each of them belongs to a
different state, viz Delhi, Jharkhand, Punjab, MP, UP, Kerala, Bihar and TN, but
not necessarily in the same order.

The one who belongs to Jharkhand is on the immediate right of T. T is third to


the left of V. T is not from Delhi. Neither Q nor W is an immediate neighbour of T. U
belongs to MP and is third to the right of the one who belongs to Jharkhand. The
one who belongs to Bihar sits second to the left of T. The person who belongs to UP
sits second to the right of the one who belongs to Punjab. R sits third to the left of U.
The one who belongs to Kerala sits second to the left of the one who belongs to
MP. P sits exactly between W and U and belongs to Punjab. Q sits fourth to the
right of W.
1). Who among the following comes from TN?
S
T
Q
P
None of these

2). Which of the following statements is true about Q?


Q sits on the immediate left of W.
Q sits second to the right of U.
Q belongs to Kerala.
All are true
None of these

3). Who sits second to the right of the one who belongs to TN?
The one who belongs to Kerala
The one who is an immediate neighbour of V and R
The one who sits third to the left of the one who belongs to Punjab.
The one who sits third to the right of the one who belongs to UP
All of these

4). Who among the following comes from Delhi?


R
W
V
Can't be determined
None of these

5). Which of the following combinations is true?


U - Kerala
T – Delhi
R - MP
S - UP
Q - Jharkhand

6). The one who is exactly between S and R belongs to which of the
following states?
Bihar
Jharkhand
UP
MP
None of these
Explanation With Answer Key:
Directions (Q. 1-6):

1). Answer: b)
2). Answer: c)
3). Answer: e)
4). Answer: c)
5). Answer: d)
6). Answer: e)
PUZZLE-16
Directions (Q. 1-5): Study the following information carefully and answer the given
questions.

Sanjay, Vasu, Gandhi, Jairaj, Prathap, Kumar, Suresh and Ramnath are eight friends
sitting around a circular table. Four of them are facing towards the centre and four of
them are facing away from the centre. All of them like different Currencies, viz. Dollar,
Pound, Dinar, Peso, Euro, Rupee, Yen and Riyal, but not necessarily in the same order.
Prathap faces towards the centre and likes Dollar. Both the immediate neighbours of
Prathap face away from the centre and like Dinar or Peso. Jairaj faces away from the
centre. Both the immediate neighbours of Jairaj do not face away from the centre.
Prathap sits third to the right of Kumar, who likes Euro and faces away from the centre.
Gandhi sits third to the left of Kumar. The one who likes Dinar sits opposits of Kumar.
The one who likes Pound is not the immediate neighbour of Kumar and faces away
from the centre. Sanjay sits second to the left of Gandhi and does not like Yen or Riyal.
The one who likes Yen sits between Ramnath and Kumar. Vasu faces away from the
centre and does not like Pound.

1). Who among the following sits second to the right of Vasu?
Jairaj
Ramnath
Gandhi
Kumar
None of these

2). The only person who is sitting between Prathap and Suresh, likes which of
the following currency?
Dinar
Euro
Rupee
Dollar
e) Peso

3). Who like the Rupee?


Prathap
Vasu
Sanjay
Ramnath
Gandhi

4). How many persons sit between Jairaj and Prathap, count from the right of Jairaj?
Five
Four
Three
Two
None of these

5). Which of the following is correctly matched?


Gandhi - Facing Centre - Euro
Suresh - Facing Outside - Peso
Kumar - Facing Centre - Dollar
Sanjay – Facing Centre - Rupee
None of these

PUZZLE-17
Directions (6-10): Study the following information carefully and answer the given
questions.
There are nine people Balaji, Darshan, Kalam, Sethu, Bilal, Ramnath, Lalit, Yuvan
and Nakul who sits around a circular table. Some are facing towards and some are
facing away from the centre. They like different pens among Parker, Cello, Camlin,
Hero and Montex. Not more than 2 people like the same pens. The following
information is known about them. Kalam and Lalit form the only pair that is sitting
together and like the same pens. They are facing in opposite directions to each
other. The one who likes Camlin sits third to the left of Darshan but is neither Kalam
nor Ramnath. Balaji sits second to the left of Ramnath, who likes Cello. The one
who likes Parker is an immediate neighbour of Yuvan and Bilal but none of them sits
adjacent to Kalam or Lalit. Balaji, Nakul and Yuvan face in same direction as that of
Bilal. Nakul sits third to the right of Sethu, who is facing away from centre and likes
Hero. Bilal sits second to the left of Kalam and likes Cello. The people who like
Cello face towards the centre. Balaji neither likes Camlin or Hero.

6). Who sits second to the right of Darshan?


Yuvan
Lalit
Kalam
Nakul
Bilal

7). Which of the following person likes Parker?


Lalit and Balaji
Yuvan and Darshan
Sethu and Nakul
Bilal and Kalam
Balaji and Nakul

8). Which of the following pen like by Yuvan?


Camlin
Parker
Cello
Montex
None of these

9). How many persons facing outside the circle?


Two
Five
Three
Six
Four

10). Which of the following combinations is true?


Yuvan– Facing outside –Camlin
Bilal– Facing centre– Parker
Balaji– Facing centre–Cello
Kalam – Facing outside – Montex
None of these

Explanation With Answer Key:


Directions (Q. 1-5):

1). Answer: a)
2). Answer: e)
3). Answer: c)
4). Answer: b)
5). Answer: d)

Directions (Q. 6-10):

6). Answer: b)
7). Answer: e)
8). Answer: a)
9). Answer: c)
10). Answer: d)

PUZZLE-18
Directions (Q. 1-5): Study the following information carefully and answer
the questions given below:
There are eight persons D, E, G, H, R, S, T and U sitting around a circular table.
Some of the persons are not facing the centre. They all like four different types of
drinks, viz sprite, Slice, Limca and Pepsi, and each drink is liked by two persons,

Note: Same direction means that if one person is facing the centre then the
other person also faces the centre and vice versa. Opposite direction means if one
person is facing the centre then the other person faces outside and vice versa.
T sits on the immediate right of E, who likes Sprite. G sits third to the left of U,
who likes Pepsi and both are facing the same direction. G and E are not facing the
same direction but G is an immediate neighbour of R. R sits fourth to the left of T
and both are facing opposite directions but like the same soft drink. The persons
who like Sprite sit adjacent to each other, but face opposite directions. The persons
who like Slice sit opposite each other. The immediate neighbours of R are facing
the centre. The person who likes Pepsi is an immediate neighbour of the persons
who like Limca. H and S are immediate neighbours of U. H is not facing the centre
and likes Limca. The one who is on the immediate left of S is not facing the centre.
S sits second to the right of G.

1). Who among the following like Pepsi?


R, T
G, U
S, D
H, U
None of these

2). How many persons are not facing the centre?


One
Two
Three
Four
None of these

3). Which of the following statements is true about D?


D is an immediate neighbour of G and the person who likes Slice.
The immediate neighbours of D are facing the centre.
D is third to the right of the person who likes Limca.
None of the given statements is true
e) D sits opposite U, who likes Pepsi.

4). Who among the following sits third to the right of U?


T, who likes Pepsi
B, who likes Sprite
R, who likes Slice
Can't be determined
None of these

5). Four of the following five are alike in a certain way and hence form a
group. Which is the one that does not belong to that group?
S,U
R, H
G,D
E, H
U,E

PUZZLE-19
Directions (Q. 6-10): Study the following information carefully and answer
the questions given below:
There are four counters I, II, III and IV at the Railway station for Tatkal,
Platform, Reservation and General tickets, but not necessarily in the same order.
Four persons W, X, Y and Z are standing on these counters to buy a ticket, but
not necessarily in the same order.

W does not buy Tatkal or Reserved Ticket and he buys neither from counter
no. II nor from counter no. III. Z goes to the station to receive a guest. Y does not
buy ticket from counter no. II. X does not buy Tatkal ticket. Z buys ticket from
counter no. IV.

6). Which of the following tickets can be bought from counter no.
IV? a) Tatkal
Platform
Reservation
Can't be determined
None of these

7). W stands at which of the following counters?


I
II
IV
Either I or IV.
Can't be determined

8). Which of the following counters is for Tatkal tickets?


I
II
III
W
Can't be determined

9). Which of the following statements is not true about Y?


Y does not buy ticket from counter no. I.
Y does not buy Platform ticket.
Y buys Reservation ticket from counter no. III.
All statements are true
None of these

10). Which of the following statements is/are not true about counter no. I?
Counter no. I is for General ticket.
X does not buy ticket from counter no. I.
W stands at counter no. I.
Only a) and c) are not true
e) None of these

Answer:
1)b 2)c 3)e 4)b 5)e 6)b 7)a 8)c 9)c 10)e

Explanation:
Directions (Q. 1-5):

Directions (Q. 6-10):


PUZZLE-20
Directions (1-5): Study the following information carefully and answer the given
questions.
C, D, G, H, J, K and L lives in a seven storey building, The lower-most floor is
numbered 1; floor above it is numbered 2 and so on. There are four males and
three females among them.
Miss L lives in floor number 2. No female lives above the floor in which Miss D
lives. H lives in an odd numbered floor and he does not live either on the top-most
or lower-most floor. J has to use most number of stairs to reach to his flat. More
than anyone else in the building, None of the female lives at the lower-most or top-
most floor. K lives just a floor below his sister G's floor. No two male or female live
immediately above or below each other.

1). Who among the following exactly sitting between L and H?


G and K
J and D
G and D
J and K
C and D

2). Who among the following lives on floor number 5?


K
J
D
L
H

3). Which of the following floor number G Lives?


Two
Six
Five
Four
Three

4). Which of the following combinations is true?


7 - Female - G
3 - Male - C
6 - Female - D
5 - Male - J
None of these

5). If ‘J’ is related to ‘Floor number 4’, ‘G’ is related to ‘Floor number 1’, then which
following is ‘H’ related to?
Floor number 7
Floor number 2
Floor number 5
Floor number 4
Floor number 6

PUZZLE-21
Directions (6-10): Study the following information carefully and answer the given
questions.
Eight persons from eight floors and different states such as Maharashtra, Andra
Pradesh, Tamil Nadu, Kerala, Rajasthan, Haryana, Karnataka and Bihar eating
8 different foods such as Biryani, Dosa, Idly, Poori, Chapati, Parotta ,Noodles
and Upma . And each of them having a mobile in the brand names of Samsung,
Microsoft, Sony and Motorola
The person who is from Andra Pradesh eating Upma as a food
The person who is eating Noodles having Motorola mobile
The third floor stayed person eating Dosa and having a mobile in the brand name
of Samsung.
Idly is eaten by the person who is having Microsoft mobile.
Minimum of three persons stayed below of Bihar state man. And the person
who belongs to Kerala not stayed in the odd numbered floor.
The mobile’s brand name Microsoft and Motorola having same number of state-men
Tamil Nadu state man stayed two floor below from the person who is belongs to
Andra Pradesh.
The top floor is not occupied by the person from Karnataka.
The Sony owned person eating Biryani.
The persons from Kerala and Andra Pradesh having same mobile brand and both of
them not having Microsoft mobile and also three persons stayed in between them.

The persons who are eating Poori food and Dosa are having same mobile brand
with the one who is eating Chapati. The Sony owned person stayed in fourth floor.
There are four persons stayed in between of Karnataka and Kerala. The lowest
floor is occupied by the Rajasthan state man.
Only three persons having one kind of mobile brand name and the person
from Karnataka who is eating Chapati is one of them.
The state man from Haryana and the person who eating Chapati are not stayed in
the even numbered floor. Only one person having Sony mobile
The person from Maharashtra who likes to eat Parotta stayed in even numbered
floor and is having same mobile brand name with the person, who is eating Idly
also stayed in first floor.
Motorola mobile is owned by the person who is from Kerala.
The person who eating Upma stayed above the floor of the person who is eating
Noodles
The Haryana who is eating Dosa having same mobile brand with the state man
from Bihar
6). How many persons have Samsung mobiles?
One
Two
Three
Four
Five

7). Who lives in the sixth floor?


The one who is from Andra Pradesh
The one who is from Kerala
The one who is from Karnataka
The one who is from Maharashtra
The one who is from Bihar

8). Chapati food eats by which of the following state person?


Tamil Nadu
Andra Pradesh
Kerala
Maharashtra
Karnataka

9). Which of the following combinations is true?


2 – Idly – Motorola - Rajasthan
4 - Biryani - Sony - Tamil Nadu
8 – Biryani – Microsoft - Rajasthan
7 - Dosa – Samsung - Maharashtra
3 – Poori - Sony - Haryana

10). Which of the following person sits exactly between who are eating Biryani and
Upma?
a) The one who eats Dosa food
The one who eats Noodles food
The one who eats Parotta food
The one who eats Poori food
The one who eats Idly food

Explanation With Answer Key:

Directions (Q. 1-5):

1). Answer: a)
2). Answer: e)
3). Answer: d)
4). Answer: c)
5). Answer: b)

Directions (Q. 6-10):


6). Answer: c)
7). Answer: a)
8). Answer: e)
9). Answer: b)
10). Answer: d)

PUZZLE-22
Directions (Q. 1-5): Study the following information to answer the given questions.

Seven stores viz. A, B, C, D, E, F and G sold different number of TV's of a


particular brand viz. 4, 6, 9, 12, 15, 18 and 27 on seven different days of the same
week starting from Monday and ending on Sunday, but not necessarily in the
same order.
A sold the TV's on one of the days after Friday. Only three stores sold TV's
between A and the store which sold 12 TV's. The store which sold TV's on the day
immediately after the store which sold 12 TV's sold more than 12 TV's but not odd
number of TV's.
Only two stores sold TV's between F and the store which sold 4 TV's. The
store which sold 4 TV's sold these before F. 4 TV's were not sold on Monday.
Only three stores sold TV's between F and D. The difference between the
number of TVs sold on Wednesday and Saturday is more than 10.
B sold the TV's on a day immediately before the day on which 9 TV's were sold.
9 TV's were not sold on Friday.
Only two stores sold TV's between B and G.
Difference between the number of TV's sold by G and D is less than 5. C did not
sell the least number' of TV's.

1). In which of the following pairs did both the stores sell odd, number of TV's?
D, E
A, B
E, F
F, G
C, D

2). Which of the following stores sold 4 TV’s?


F
A
C
G
E

3). Based on the given arrangement, which of the following is true?


D sold the TVs on Wednesday.
Only one store sod TV's between C and F.
A sold 12 TV's on Sunday.
C sold 18 TV's on Thursday.
None of these

4). How many stores sold less number of TV's than B?


a) Five
Three
Six
Two
One

5). What is the difference between the number or TV’s sold on Monday and Friday?
9
12
18
3
11

PUZZLE-23
Directions (Q. 6-10): Study the following information carefully and answer
the questions given below:
J, K, L, M, N and O are six members of a group in which there are three
female members. All the three females work in three different departments, viz HR,
Accounts and Publishing. All the persons working in the same department are not
on the same floor. All the females sit on three different floors, viz 1st, 2nd and 3rd.
Two persons work on each floor and in each department.

K and N work in the same department but not in Publishing. N and J are on
the 1st and 3rd floors respectively. M works in the Accounts department. J works
in the same department in which O works. L, a male, works on the first floor. M is a
female but does not sit on the 2nd floor. J is not a female.

6). Which of the following pairs of employees works on the second floor?
N, K
J, O
K, O
K, I
e) O, N

7). O works in which of the following departments?


Accounts
Publishing
HR
Either a) or b)
None of these

8). Which of the following groups of employees are females?


M, N, O
J,M, O
M, O, K
L, K, N
None of these

9). Which of the following pairs of persons works in Accounts department?


L, M
O, M
I, M
L, O
Data inadequate

10). If O is transferred to HR and N is transferred to Accounts, who is to be


transferred to Publishing to maintain the original distribution of departments on each
floor?
K
L
M
Data inadequate
None of these
Answer:
1)b 2)e 3)d 4)c 5)a 6)c 7)b 8)a 9)a 10)d

Explanation:

Directions (Q. 1-5):

Directions (Q. 6-10):

Male (+), Female (-)


PUZZLE-24
Directions (Q. 1-5): Study the following information and answer the questions.

There are nine persons Prem, Vinoth, Harish, Ramesh, Satish, Atreya,
Mohan, Gowtham and Yadav stay on a nine floors building, but not necessarily in
the same order. Only one person stays on each floor. All of them clear either SBI
PO or IBPS PO examination, Each of them presently working as a clerk in different
banks i.e. Andra Bank, Bank of India, Canara Bank, Indian Bank, Syndicate Bank,
Punjab National Bank, UCO Bank, Vijaya Bank and IDBI Bank, but not necessarily
in the same order. Only five of them clear IBPS PO exam. The ground floor is
numbered 1 and the topmost floor is numbered 9. There are two floors between the
persons who working in a Punjab National Bank cleared SBI PO exam and who
working in Vijaya Bank cleared IBPS PO exam. Harish working in Canara Bank
cleared SBI PO exam. Atreya does not working in IDBI Bank but he cleared SBI PO
exam. Prem stays on even numbered floor below the even numbered floor on which
Gowtham stays. The one who working in Syndicate Bank cleared IBPS PO exam
stays on the fourth floor. Ramesh stays on the second floor and working in Indian
Bank cleared IBPS PO exam. The one who working in Bank of India cleared IBPS
PO exam stays on the topmost floor. Prem does not working in UCO Bank cleared
IBPS PO exam. Atreya working in Punjab National Bank cleared SBI PO exam and
stays on an odd-numbered floor. There is only one floor between the floors on which
Atreya and Mohan stay. Satish stays immediate above the floor on which Yadav
stay. Mohan does not stay on the ground floor. Vinoth stays immediate above the
floor on which Gowtham stay. There are three floors between the floors on which
Harish and Atreya stay. The one who working in Vijaya Bank cleared IBPS PO exam
stays immediate above Mohan .Satish stays on the fourth floor. The one who
working in IDBI Bank clearedSBI PO exam stays on the third floor.

1). Who lives on the 7th floor?


Mohan
Vinoth
The one who working in IDBI Bank cleared IBPS PO exam
Atreya
The one who working in Vijaya Bank cleared IBPS PO exam

2). The person who stays on the 4th floor working in which of the following bank?
Canara Bank
UCO Bank
Indian Bank
Syndicate Bank
Andra Bank

3). How many persons are between the one who working in Bank of India and
the one who stays on 4th floor?
Five
Three
Four
Seven
None of these

4). Which of the following bank Atreya working?


Andra Bank
Canara Bank
Syndicate Bank
IDBI Bank
Punjab National Bank

5). Four of the following five are alike in a certain way and hence they form a group.
Which one of the following does not belong to that group?
Canara Bank
Vijaya Bank
Bank of India
UCO Bank
IDBI Bank

PUZZLE-25
Directions (6-10): Study the following information carefully and answer the given
questions.

Seven people namely Lakshitha, Brinda, Krithika, Charvi, Neeja, Umika and
Mohini buy different company laptops viz. Dell, HP, Asus, Apple, Lenovo and Acer
and different company computers viz. Microsoft, Samsung, Sony and LG from
different sites say Amazon, Shopclues, Flipkart and Snapdeal. Not more than two
people buy from same site. It is known that each person buys both the items from
the same site. The people who buy HP company laptop get from the same site and
they don’t buy the same company computers and is neither Lakshitha nor Mohini.
Umika buys from Snapdeal. Charvi and Umika buy same company computers which
are bought by no one apart from them and they get from different sites. The only
person who orders LG company computer from Flipkart and is neither Brinda nor
Krithika. Charvi doesn’t buy on Amazon. Acer company laptop is get from Amazon
but not by Lakshitha. Krithika buy Asus company laptop. One who buys Microsoft
company computer buys from Amazon and is not Brinda and also buys either Dell
or Lenovo company laptop. The persons who buy Samsung company computer get
on different sites but not on Shopclues. Only one person buys Microsoft company
computer. Krithika doesn’t get Sony company computer. Umika neither orders Dell
or Apple company laptop.

6). Who among the following person buying in Shopclues?


Lakshitha
Krithika
Charvi
Brinda
Mohini
7). Who among the following persons bought the Samsung company computer?
Mohini
Krithika
Brinda
All of the above
None of these

8). Which of the following company laptop and computer bought by Charvi?
Dell and Samsung
Apple and Sony
Acer and LG
Apple and Microsoft
Lenovo and Sony

9). Which of the following company laptop bought by two persons?


HP
Acer
Asus
Dell
Lenovo

10). Which of the following combinations is true?


Neeja – HP – LG – Amazon
Krithika – Asus – Microsoft - Snapdeal
Lakshitha – Dell – Samsung - Amazon
Mohini - Lenovo - Samsung - Amazon
e) Brinda - HP - Samsung - Flipkart

Explanation With Answer Key:


Directions (Q.1-5):
1). Answer: a)
2). Answer: d)
3). Answer: c)
4). Answer: e)
5). Answer: b)
Directions (Q. 6-10):

6). Answer: c)
7). Answer: d)
8). Answer: b)
9). Answer: a)
10). Answer: e)

PUZZLE-26
Directions (1-5): Study the following information carefully and answer the given
questions.
Naresh, Fayaz, Suvetha, Milan, Lavanya, Sahana, Archana, Renuka, Sarojini,
Vidya, Geeta and Jemima are 12 persons sitting in two rows, among them Naresh,
Suvetha, Milan, Sahana, Sarojini and Geeta are facing north while the remaining are
facing south. Each person faces exactly one person in the other row. Each one of
them likes a different city from among London, Paris, Dubai, Tokyo, Rome, Vienna,
Mumbai, New York, Bangkok, Chennai, Amsterdam and Bangalore (not necessarily
in the same order) and belongs to one country each from among Australia, China,
South Africa, Japan, Canada, France, India, Netherland, Poland, Qatar, Thailand
and Iceland (again not necessarily in the same order).
The one who likes Rome is from France and is facing Sahana. None of Fayaz,
Jemima and Lavanya likes Rome or London. The persons from South Africa and
Iceland are the immediate neighbours of Vidya.
Fayaz is second to the right of Archana, who is from India. Fayaz like Mumbai city
and faces the person from Qatar, who is third to the right of Milan. Neither Naresh
nor Milan likes Paris city or from China.
Geeta, like the Dubai city, sits second to the right of the person who likes Chennai
city, who faces the person from Iceland, None of them sits at an extreme end.
The Paris city like by the person from Thailand, sits as far as possible from Milan.
Jemima, likes the Vienna city, sits opposite the person who likes the Amsterdam
city, who is an immediate neighbour of the person sitting at an extreme end.
The Mumbai and Tokyo city liker’s are from South Africa and Australia respectively.
The person who likes Vienna city and Tokyo city face the same direction.
Sahana, like the New York city and from Japan, sits equidistant in the same row
from the Bangkok and the city from China.
The city from Poland is opposite the city from Australia and second to the left of the
city from Japan. Naresh, who is not like the Amsterdam city, is third to the left of
Sarojini; and neither of them like Paris city.
Renuka likes the Bangalore city and is third to the right of person who likes Vienna
city from Canada.

1). Who sits second to the left of Vidya?


Lavanya
Fayaz
Renuka
Archana
Jemima

2). Which of the following person belongs to Qatar?


Geeta
Sarojini
Vidya
Naresh
Jemima

3). Which of the following persons sits at the extreme ends of one row?
Lavanya and Fayaz
Naresh and Suvetha
Milan and Suvetha
Archana and Renuka
Sahana and Milan

4). The one who belongs to Canada sits immediate of ______________


The one who belongs to South Africa
The one who belongs to France
The one who likes London city
The one who likes Mumbai city
The one who likes Tokyo city

5). Which of the following combinations is true?


Sahana - New York – South Africa
Renuka - Bangalore - Iceland
Sarojini - Chennai - China
Jemima – Mumbai - Canada
None of these

PUZZLE-27
Directions (Q. 6-10): Study the following information carefully and answer the given
questions.
Ten persons Harshika, Kokila, Uma, Meena, Chitra, Dhiya, Nisha, Prema, Thamarai
and Aruna are sitting in two rows with five persons in each row. The persons in row
one are facing south and the persons in row two are facing north. Each person in
row one faces a person from the other row. All of them have a book of different
authors, viz Chetan Bhagat, Kiran Desai, Arundhati Roy, Anita Desai, Salman
Rushdie, Vikram Seth, Amitav Ghosh, Jhumpa Lahiri, Anita Nair and Ruskin Bond,
but not necessarily in the same order.

The persons who like Salman Rushdie and Vikram Seth sit opposite each other.
Dhiya sits opposite to Harshika, who likes Chetan Bhagat. The one who likes Kiran
Desai sits opposite the one who likes Jhumpa Lahiri. Aruna is not facing north but
sits third to the left of Nisha, who likes Kiran Desai. There is only one person
between Kokila and Uma. Chitra sits at one of the ends of the row and likes Vikram
Seth. The one who likes Jhumpa Lahiri is on the immediate right of Meena, who
does not like Ruskin Bond. The persons who like Arundhati Roy and Anita Desai
respectively are not facing north. Uma likes Amitav Ghosh. The one who likes Anita
Desai sits opposite the one who is second to the right of Kokila. Thamarai does not
like Ruskin Bond. Chitra sits opposite the one who sits second to the left of the one
who likes Arundhati Roy.

6). Who have Anita Nair book?


Chitra
Kokila
Meena
Dhiya
Nisha

7). Who sits opposite to Kokila?


The one who have Chetan Bhagat book
The one who have Arundhati Roy book
Prema
The one who have Salman Rushdie book
Nisha

8). Who sits opposite to the one who sits second to the left of Chitra?
The one who have Arundhati Roy book
The one who have Anita Desai book
The one who have Anita Nair book
The one who have Kiran Desai book
None of these

9). Who among the following is the immediate neighbour of the one who have
Chetan Bhagat book?
The one who have Anita Nair book
Uma
Thamarai
The one who have Kiran Desai book
None of these
10). Who sits at the right end in Row II?
Prema
Aruna
Nisha
Chitra
Meena

Solutions:

1). Answer: e)
2). Answer: a)
3). Answer: c)
4). Answer: d)
5). Answer: b)

Facing Lavanya Renuka Vidya Fayaz Jemima Archana


South
(Tokyo) (Bangalore) (Rome) (Mumbai) (Vienna) (London)

(Australia) (Iceland) (France) (South (Canada) (India)


Africa)

Facing Milan Naresh Sahana Geeta Sarojini Suvetha


North
(Bangkok) (Chennai) (New (Dubai) (Amsterdam) (Paris)
York)
(Poland) (Netherland) (Qatar) (China) (Thailand)
(Japan)

6). Answer: c)
7). Answer: e)
8). Answer: a)
9). Answer: b)
10). Answer: d)

Facing South Prema Nisha (Kiran Dhiya Thamarai Aruna


(Ruskin Desai) (Arundhati (Anita Desai) (Salman
Bond) Roy) Rushdie)

Facing North Meena (Anita Kokila Harshika Uma (Amitav Chitra


Nair) (Jhumpa (Chetan Ghosh) (Vikram Seth)
Lahiri) Bhagat)

PUZZLE-28
Directions (Q. 1-5): Study the following information carefully and answer the
questions given below:
A, B, C, D, E, F, G and H are friends having different occupations, viz – Clerk,
Manager, PO, Professor, Teacher, Director, Supervisor and Editor. All are sitting
around a circular table facing the centre, but not necessarily in the same order.

C sits third to the right of the Teacher. Only two persons sit between the Teacher
and H. The Manager and the Clerk are immediate neighbours. Neither C nor H is a
manager or Clerk. The Manager is not an immediate neighbor of the Teacher. The
PO sits third to the right of D, who is not a Manager. F sits on the immediate right
of the Editor. The Director sits second to the left of A, who is not an immediate
neighbor of H. The Director is an immediate neighbor of both the professor and the
PO. C is not an Editor. B is not an immediate neighbour of C. E is not an immediate
neighbour of the Director. H is not a Professor and sits second to the left of the
Editor. E. is an immediate neighbour of D and C.

1). Who among the following is the Editor?


A
B
E
G
None of these

2). Which of the following statements is/are true?


A is an Editor and sits on the immediate right of F.
The Supervisor and the Professor are immediate neighbours of G.
F is a Teacher and sits second to the left of E.
C is a PO and sits second to the left of
None is true

3). How many persons sit between the Editor and the Supervisor?
None
One
Two
Three
None of these

4). What is the occupation of D?


Editor
Professor
Clerk
Director
None of these

5). Who among the following sits second to the left of the Supervisor?
Clerk
PO
Teacher
Director
None of these

Answers:
1)a 2)c 3)d 4)c 5)a
Explanation:
Directions (Q. 1-5):

PUZZLE-29
Directions (Q. 1-5): Study the following information carefully and answer the
given questions.
There are seven candidates – Akshar, Naren, Ganesh, Baskar, Vimal,
Mishra and Karthik – who attends different examinations which started on 25th
December and ended on 31st December. Each of them wrote different Exams, viz
SBI PO, IBPS PO, SYNDICATE PO, INDIAN PO, DENA PO, BOB PO and BOM
PO, but not necessarily in same order. They like different currencies, viz Rupee,
Dinar, Dollar, Pound, Riyal, Euro and Yen, but not necessarily in same order.

Karthik, who likes Euro, attended exam on fourth day of week but attended
neither SBI PO nor IBPS PO Exam. Two exams were held between the exams of
Karthik and Mishra, neither of whom attended on 1st day of week. There was one
exam between exam of Baskar and Ganesh. But Ganesh’s exam did not happen
either on 1st or on 3rd day of week. Ganesh likes Dinar currency and attended SBI
PO Exam. The one who attended SYNDICATE PO Exam on the last day of week,
likes Dollar currency. Naren did not attend exam on day either immediately before
or immediately after performance of Baskar, who does not like either Riyal or Rupee
or Pound currency. Vimal attended immediately after Ganesh exam and he likes
Rupee currency. Akshar does not like Riyal currency and attended INDIAN PO
Exam. Baskar did not attend either BOM PO or BOB PO Exam. One who attended
in DENA PO was scheduled immediately after SBI PO exam.

1). Which of the following exam held on 28th December?


IBPS PO
BOB PO
BOM PO
Either BOB PO or BOM PO
Either IBPS PO or INDIAN PO

2). Which of the following currency Mishra likes?


Dollar
Yen
Rupee
Euro
Dinar

3). Which of the following day SBI PO exam was organized?

30th December
25th December
29th December
31st December
26th December

4). Which of the following candidate like Pound currency?


a) Vimal
Akshar
Ganesh
Baskar
Mishra

5). Which of the following statement is true?


29th December – Ganesh - SBI PO - Riyal
30th December – Vimal - BOB PO - Rupee
26th December - Mishra - INDIAN PO - Pound
31st December – Akshar - SYNDICATE PO - Dollar
27th December – Baskar - IBPS PO - Yen

PUZZLE-30
Directions (Q. 6-10): Study the following information carefully to answer the given
questions:
Eight friends Sameer, Taranga, Devnath, Parvesh, Harendra, Jaswant,
Gurdeep and Waseem live on eight different floors of a building but not
necessarily in the same order. The lowermost floor of the building is numbered 1
and the topmost floor of the building is numbered 8. Each of them likes different
players of grand prix viz, Valtteri Bottas, Kimi Räikkönen, Max Verstappen, Nico
Rosberg, Daniel Ricciardo, Fernando Alonso, Lewis Hamilton and Sebastian
Vettel but not necessarily in the same order.

The one who likes Kimi Räikkönen lives on an even-numbered floor but not on
the topmost floor. Only one person lives between Jaswant and the one who likes Daniel
Ricciardo. Only two persons live between Jaswant and the one who likes Kimi
Räikkönen. Neither Harendra nor Devnath lives on the first floor. Only one person lives
between Devnath and the one who likes Max Verstappen. Sameer lives just above
Jaswant. Only two persons live between Harendra and Sameer. The one who likes
Daniel Ricciardo does not live on floor number one, Taranga lives on an even-
numbered floor and just above Devnath. The one who likes Nico Rosberglives
on an even numbered floor and lives just above the person who likes Sebastian
Vettel. Devnath does not like Daniel Ricciardo or Sebastian Vettel. Only two
persons live between the one who likes Fernando Alonsoand the one who likes
Valtteri Bottas. Parvesh does not like Lewis Hamilton. The one who likes Fernando
Alonso does not live on an odd-numbered floor. Waseem lives just below the one
who likes Sebastian Vettel.

6). Who lives on Fourth floor?


Parvesh
Taranga
Gurdeep
Sameer
Waseem

7). Jaswant lives on which of the following floor?


Seventh
Sixth
Third
Fifth
Fourth

8). Which of the following player Harendra likes?


Nico Rosberg
Kimi Räikkönen
Fernando Alonso
Max Verstappen
Daniel Ricciardo

9). Which of the following statement is true?


6 – Jaswant - Sebastian Vettel
4 – Taranga – Lewis Hamilton
3 – Devnath - Valtteri Bottas
2 – Gurdeep – Nico Rosberg
2 – Parvesh - Max Verstappen

10). Four of the following players form a group, which of the following player
does not belong to that group?
Lewis Hamilton
Nico Rosberg
Fernando Alonso
Valtteri Bottas
Kimi Räikkönen

Explanation With Answer Key:


Directions (Q. 1-5):

1). Answer: d)
2). Answer: a)
3). Answer: c)
4). Answer: b)
5). Answer: e)

Directions (Q. 6-10):


6). Answer: b)
7). Answer: a)
8). Answer: e)
9). Answer: c)
10). Answer: d)

PUZZLE-31
Directions (Q. 1-6): Study the following Information carefully and answer
the questions given below:
Nine people A, B, C, D, E, F, G, II and J are living in a nine-storey building.
The ground floor is numbered 1 and the topmost floor is numbered 9. All of them
like a different colour, viz Red, Pink, Green, Yellow, Orange, Violet, Black, Blue and
White, but not necessarily in the same order.

B likes Yellow and lives on an even-numbered floor. J lives on even-numbered


floor below the floor on which B lives. The one who likes Blue lives on the fourth floor.
E lives on the second floor and likes Green. The one who lives on the third floor likes
White. J does not like Blue. There are two floors between the floors on which the
persons who like Orange and Yellow respectively, live. There are three floors between
the floors on which G and C live respectively. D does not like White. There is one floor
between the floor on which D and C live respectively. F lives on a
floor immediately above A's floor. The one who lives on the topmost floor likes Red.
D does not live on the ground floor. G likes Pink. C does not live on topmost floor.
There is only one floor between the ones who like Black and Yellow respectively.

1). How many floors are there between the floor on which A lives and the floor on
which J lives?
One
Two
Three
None
None of these

2). On which of the following floors does C live?


First
Third
Fifth
Second
None of these

3). Which of the following colours does the person like who lives on the first floor?
Red
Black
Pink
Violet
None of these

4). J likes which of the following colours?


Blue
Violet
Yellow
Orange
e) None of these

5). Who lives on the floor which is exactly between the floor on which B lives and
the floor on which J lives?
A
G
E
D
None of these

6). Who among the following lives on the topmost floor?


E
A
C
H
None of these

Answers:
1)b 2)c 3)c 4)e 5)d 6)d
Explanation:
Directions (Q.1-6):
PUZZLE-32
Directions (Q. 1-5): Study the following information carefully and answer the given
questions.
Seven flights - Emirates, Singapore Airlines, Qatar Airways, Etihad Airways,
Lufthansa, Qantas Airways and Turkish Airlines - depart from Delhi Airport on
Friday. Each flight departure at different times 4 AM, 7 AM, 8 AM, 10 AM, 1 PM,
3 PM and 6 PM not necessarily in the same order.

Qatar Airways is the earliest to depart. Its departure time is not after 7 a.m. and its
travel time is two hours less than the travel time of Qantas Airways. Since Emirates
has to cover the least distance, it leaves last, but before 7 p.m. and still completes
its journey at 9 p.m. Etihad Airways leaves Delhi Airport at Friday 10 a.m. and takes
19 hours to reach its destination. There is a four hour gap between the departure
times of Lufthansa, leaving at 8 a.m. and Qatar Airways. Lufthansa takes one hour
more than the travel time of Qatar Airways to reach its destination. Emirates leaves
five hours after Singapore Airlines departs but takes only three hours to reach its
destination. Qantas Airways and Turkish Airlines take 10 hours and 8 hours to
reach their respective destinations. Qantas Airways leaves at 3 p.m. and there is
gap of 8 hours between the departure times of Qantas Airways and Turkish Airlines.
One flight reaches its destination at 6 p.m.

1). Which of the following flights reach the destination on Saturday?


Etihad Airways
Qantas Airways
Both Etihad Airways and Qantas Airways
Turkish Airlines
Singapore Airlines

2). How many flights departure between Singapore Airlines and Qantas Airways?
None
One
Two
Three
Four

3). Which of the following flight departure last on Friday?


Qantas Airways
Emirates
Etihad Airways
Turkish Airlines
Lufthansa

4). What is the difference between the departure timing of Etihad Airways
and Emirates flights?
2 hours
4 hours
6 hours
8 hours
10 hours

5). Which of the following statement is true as per the given information?
Singapore Airlines - 3:00 PM - 6:00 PM
Emirates - 6:00 PM - 8:00 PM
Turkish Airlines - 9:00 AM - 1:00 PM
Lufthansa - 10:00 AM - 7:00 PM
Qatar Airways - 4:00 AM - 12:00 noon

PUZZLE-33
Directions (Q. 6-10): Study the following information carefully and answer the given
questions.
In an Interview seven candidates such as Kevin, Monica, Akshara, Bala, Lokesh,
Varun and Ramesh are attending interview in three different panels at three different
sessions in a day. Only three candidates going to attend in a panel at different
session of the day. Four CEO’s Usha Ananthasubramanian, Subramaniakumar,
Rajkiran Rai and Sankara Narayanan are interviewed each panel, One of the three
panels has two CEO’s. Two panels will have only one CEO to interview the
people. One of the three panels has two CEO’s for recruiting the people in
afternoon session. All the CEO will be present at their allocated interview panels
and the session will start as per scheduled. Every candidates should be
interviewed in their respective allocated interview panels and only two panels
going to interview the same number of people.
Kevin and Monica not going to attend the interview at evening. The candidate
Bala interviewed by the CEO Rajkiran Rai at 9.00 A.M
Ramesh going to attend interview after 2.00 P.M
Lokesh and Kevin going to get interviewed at the same panels
Usha Ananthasubramanian is the CEO who is not going to interview the people
at panel III.
Rajkiran Rai is the CEO going to interview Varun and one other candidate
in different sessions.
In the Evening session Akshara and Varun going to interviewed and both of
them not interviewed at panel I.
Ramesh going to be interviewed at the same session with Lokesh and not in
the panel III
Monica’s interview time is 9.00 A.M. and He is not interviewed by
Subramaniakumar and Sankara Narayanan.

Akshara Interviewed at Panel II.


6). How many persons attend the interview on Afternoon session?
Two
Three
One
Either two or three
e) Either One or Two

7). Rajkiran Rai is the CEO going to interview on __________ session


Morning
Afternoon
Evening
Both Morning and Afternoon
Both Morning and Evening

8). Which of the following candidates attends the interview on afternoon session?
Kevin
Lokesh
Ramesh
All of the above
Only Kevin and Lokesh

9). If “Monica” related to “Akshara” and “Bala” related to “Varun” on a certain basis,
on the same basis “Kevin” related to?
Both Lokesh and Ramesh
Only Lokesh
Only Ramesh
Bala
Monica

10). Which of the following statement is true as per the given information?
More than two peoples attend interview at Morning session
Bala and Varun going to interview at panel I
Usha Ananthasubramanian is the CEO who is going to interview the people at panel
II
Monica attend the interview on Evening session
None of the options are true
Explanation With Answer Key:

Directions (Q. 1-5):

1). Answer: c)
2). Answer: a)
3). Answer: b)
4). Answer: d)
5). Answer: e)

Directions (Q. 6-10):

6). Answer: b)
7). Answer: e)
8). Answer: d)
9). Answer: a)
10). Answer: c)

PUZZLE-34
Directions (Q. 1-5): Study the following information carefully and answer the
given questions.
Six competitive exams – IBPS PO, IBPS Clerk, RRB PO, RRB Clerk, SBI PO and
SBI Clerk were written by three males – Ahmad, Gagan and Kaushik and three
females – Sandhya, Roshni and Charita. Three of these competitive exams were
held in Mumbai and the rest in Delhi. Each of these exams was organized by
different IT Companies– Infosys, TCS, HCL, Wipro, Oracle and Tech Mahindra
(may not be in the same order). Each exam was written by a male and a female.

None of the students wrote in a city twice and no two competitive exams were written
by the same pair of male and female. The females who written IBPS PO and IBPS
Clerk did not write the exam organized by Oracle and Tech Mahindra. The male
student was same for the exams – IBPS PO and IBPS Clerk. Gagan written RRB PO
but did not write the exam organized by Oracle or Infosys. Roshni did not write any
exam with Ahmad, but written RRB Clerk with Kaushik. Sandhya wrote an exam with
Kaushik. Tech Mahindra organized the exam in Mumbai and Wipro organized the
exam in Delhi which was written by Ahmad and HCL organized IBPS Clerk. SBI PO
was organized by Infosys. Oracle and TCS organized the exams in the same city.
IBPS Clerk and RRB PO exam were written by the same female.

1). Who among the following persons wrote IBPS PO exam?


Gagan and Roshni
Kaushik and Roshni
Ahmad and Sandhya
Gagan and Charita
Kaushik and Sandhya
2). Who among the following persons wrote the exam with Roshini in Delhi?
Kaushik
Gagan
Ahmad
Either Kaushik or Gagan
None of these

3). Which of the following IT company conduct SBI Clerk exam?


HCL
Tech Mahindra
TCS
Infosys
Oracle

4). Which of the following exam wrote by Ahmad and Charita?


IBPS PO
RRB PO
SBI PO
SBI Clerk
IBPS Clerk

5). Which of the following combinations is true?


IBPS Clerk – Ahmad – Charita – Delhi - HCL
RRB Clerk – Gagan – Roshni - Mumbai - Tech Mahindra
SBI PO – Kaushik – Sandhya - Mumbai - Oracle
RRB PO – Gagan – Charita - Delhi - TCS
All the statements are wrong
PUZZLE-35
Directions (Q. 6-10): Study the following information carefully and answer the given
questions.

Eight friends Ricardo, James, Shane, Roberto, Alberto, Lawrence, George and
Fernando live on eight different floors of a building but not necessarily in the same
order. The lowermost floor of the building is numbered 1 and the topmost floor of
the building is numbered 8. Each of them likes different brands of City viz, Mumbai,
Chennai, Hyderabad, Bangalore, Kolkata, Delhi, Kochi and Patna but not
necessarily in the same order.

The one who likes Chennai lives on an even-numbered floor but not on the topmost
floor. Only one person lives between Lawrence and the one who likes Kolkata. Only
two persons live between Lawrence and the one who likes Chennai. Neither Alberto
nor Shane lives on the first floor. Only one person lives between Shane and the one
who likes Hyderabad. Ricardo lives just above Lawrence. Only two persons live
between Alberto and Ricardo. The one who likes Kolkata does not live on floor
number one, James lives on an even-numbered floor and just above Shane. The
one who likes Bangalore lives on an even numbered floor and lives just above the
person who likes Patna. Shane does not like Kolkata or Patna. Only two persons
live between the one who likes Delhi and the one who likes Mumbai. Roberto does
not like Kochi. The one who likes Delhi does not live on an odd-numbered floor.
Fernando lives just below the one who likes Patna.
6). Alberto lives on which of the following floor?
7
5
3
4
2

7). Which of the following person like Kochi city?


a) George
Shane
Fernando
Lawrence
Roberto

8). How many persons are between the one who likes Patna and George?
One
Two
Three
Four
Five

9). Which of the following statement is true?


6 – Fernando - Mumbai
1 –George- Hyderabad
4 – James - Chennai
2 – Shane - Delhi
8 – Ricardo - Kolkata

10). Four of the following five are alike in a certain way and hence they form
a group. Which one of the following does not belong to that group?
Patna
Hyderabad
Mumbai
Kolkata
Bangalore

Explanation With Answer Key:


Directions (Q. 1-5):
1). Answer: c)
2). Answer: a)
3). Answer: b)
4). Answer: e)
5). Answer: d)

Directions (Q. 6-10):

6). Answer: b)
7). Answer: a)
8). Answer: d)
9). Answer: c)
10). Answer: e)
PUZZLE-36
Directions (1-5): Study the following information carefully and answer the given
questions.
Ten students namely viz Varun, Kishore, Prakash, Sundar, Bhavesh, Jeevika,
Usha, Harsha, Durga and Subha called for document verification but not necessarily
in the same order have document verification on five different days starting from
Monday to Friday of the same week. Students have document verification at two
different time slots, i.e 10.00 A.M and 02.00 P.M.

Durga have document verification on Tuesday at 10.00 A.M. The number of people
who have document verification between Usha and Sundar is same as the number
of people who have document verification between Prakash and Harsha. Kishore
have document verification immediately before Durga. Durga does not have
document verification on any of the days before Usha. The one who have document
verification at 10.00 A.M immediately before Subha. Harsha does not have
document verification at 02.00 P.M. Sundar have document verification immediately
after the day of one who have document verification on Monday. Jeevika does not
have document verification at 02.00 P.M. Sundar does not have document
verification on any one of the days after Bhavesh. Only three people have document
verification between Usha and Bhavesh. Neither Bhavesh nor Usha does not have
document verification on Friday. Only two people have document verification
between Jeevika and Subha. Jeevika does not have document verification on any of
the days after Harsha.

1). Which of the following students attends document verification on Wednesday?


Durga and Prakash
Kishore and Bhavesh
Bhavesh and Prakash
Usha and Harsha
Subha and Sundar
2). Jeevika and Varun have document verification on which of the following day?
Friday
Thursday
Wednesday
Monday
Tuesday

3). How many persons have document verification after Sundar?


Two
Three
Four
Five
Six

4). Four of the following students form a group, which of the following student
does not belong to that group?
Harsha
Varun
Kishore
Subha
Prakash

5). Which of the following statement is true as per the given information?
Varun attend document verification after six students
Durga and Prakash attends document verification on same day
Two students attend document verification between Sundar and Varun
Five students have document verification after Bhavesh
None of these
PUZZLE-37
Directions (6-10): Study the following information carefully and answer the given
questions.
Bikas, Jalal, Nanda, Tarun, Manas, Francis, Ikshan and Larshan are eight
employees of Indian Bank who attends training in the four months (May,
September, November and December) of the same year. The trainings are held in
each month on either 17th or 28th. Only two trainings are held in a month. It is
further known that:

Bikas attends training in a month with 30 days. Only two people attend trainings
after Nanda. Bikas and Tarun attend their training on consecutive turns. Tarun and
Manas attend trainings on same date. Manas attends training in a month with 31
days. Larshan attends training before Francis and Ikshan. Jalal or Larshan are not
the first to go for training. Neither Ikshan nor Jalal has training on 28th. Ikshan and
Francis do not have trainings in the same month.
6). Which of the following persons exactly attends training between Bikas
and Nanda?
Jalal
Larshan
Nanda
Francis
Ikshan

7). Ikshan attends training on which of the following month?


November
September
May
December
None of these

8). Which of the following persons attends the training on December month?
Ikshan and Francis
Jalal and Francis
Manas and Bikas
Jalal and Nanda
Tarun and Jalal

9). Four of the following form a group, which of the following does not belong to that
group?
Manas
Tarun
Ikshan
Larshan
Jalal

10). Which of the following statement is true as per the given information?
Larshan attends the training on December
Ikshan attends the training on 28th November
Tarun and Bikas attend the training on September
Nanda and Francis attends the training on 17th
None of these

Answer Key:
1).c 2).b 3).e 4).a 5).d 6).e 7).a 8).b 9).d 10).c

Explanation With Answer Key:


Directions (Q. 1-5):
Directions (Q. 6-10):

PUZZLE-38
Directions (Q. 1-5): Study the following information carefully and answer
the questions given below.
Eight persons P, Q, R, S, T, U, V and W stay in an apartment in eight different
floors but not necessarily in the same order. The lowermost floor of an apartment is
numbered 1 and topmost floor is numbered 8. They are born in different month i.e.
January, February, March, April, May, June, July and August.
S lives on the odd numbered floor above the floor on which P lives. Q has born in the
month of January and lives in an odd numbered floor, below the floor on which P
lives but not immediate below. The number of floors between W & U is double the
number of floors between P & Q. There is no month gap between month of V and W
was born. V lives neither immediate above nor immediate below on which W live.
One who has born on May month lives on even numbered floor but not on the
eighth floor. R is adjacent only with one who born in the month of minimum number
of days amongst all. P lives on the fourth floor and does not born on the month
which has maximum number of days. One who has born on March lives on the
topmost floor. One who has born in June month lives below the floor on one who
born in July month. U lives one of the floors above W.

1). Who among the following person had born on May month?
S
T
U
V
W

2). Who lives on floor number 3?


R
S
T
U
V

3). Which of the following combinations of month, person and floor is true?
June - P - Four
May - T - Six
Feb – U -Seven
Feb - W - Seven
None is true
4). Which of the following statement is/are definitely true?
W is an immediate neighbour of Q
Two floors in between P and T
W is not lives immediate below the floor on which R lives
One who born in June and August are adjacent to each other
Both a) and c)

5). Four of the following five are alive in a certain way based on the given
information and so form a group. Which is the one that does not belong to that
group?
Q
W
R
V
S

PUZZLE-39
Directions (Q. 6-10): Study the following information carefully and answer
the questions given below.

There are eight persons namely A, B, C, D, E, F, G and H born in the same month
of different years 1935, 1946, 1961, 1969, 1974, 1992, 1996, 2004. Their ages are
calculated as on the same month of 2016. A is born before independence. E is born
in even number year, which is completely divisible by four but, not completely
divisible by three. H is 8 years elder to B, who is 5 years older than D. G is 46 year
younger than A. C is not youngest among all.

6). Who among the following was born in 1974?


G
D
E
F
H

7). Who among the following is the youngest?


B
E
F
D
G

8). Which of the following year B is born?


1946
1961
1974
1996
none of these

9). How many people is/are born before H?


One
Two
Three
Four
No one

10). What is the age of G?


46 years younger than A
50 years younger than A
56 years younger than B
Both a) and c)
None of these
Answers:
1). d) 2). c) 3). c) 4). e) 5). b) 6). b) 7). c) 8). e) 9). b) 10). a)

Explanation:
Directions (Q. 1-5):

Directions (Q. 6-10):


PUZZLE-40
Directions (1-5): Study the following information carefully and answer the given
questions.
Each of the five students - Sukumar, Gokul, Arvind, Sharma and Vikram - studies
5 subjects -Current Affairs, Reasoning, Aptitude, Computer and English - in a
week starting on Monday and ending on Friday.
Exactly five subjects were studied on each of the five days, no two of which
were same.
No student studied more than one subject on any of the five days.
Arvind studied Aptitude on Tuesday.
Sharma studied English on Wednesday. He didn't study Current Affairs on
Thursday.
Vikram studied Computer on Monday.
Sukumar studied the same subject on Monday which Sharma studied on Friday.
The subject was not Aptitude.
Gokul studied Current Affairs, Reasoning and Computer on Monday,
Wednesday and Friday respectively.
Arvind studied the same subject on Monday which Sukumar studied on Friday.

1). Which of the following subjects learned by Sharma on Wednesday?


Reasoning
Aptitude
Current Affairs
Computer
English

2). Gokul learned how many subjects between Current Affairs and Computer?
None
One
Two
Three
Four

3). Sukumar studied Aptitude on which of the following day?


Thursday
Wednesday
Friday
Monday
Tuesday

4). If Arvind and Vikram interchange her subjects on all days, then Vikram
studies reasoning on which of the following day?
Thursday
Friday
Monday
Wednesday
Tuesday

5). Which of the following statement is true as per the given information?
Monday – Reasoning - Current Affairs – English – Computer - Aptitude
Tuesday – Computer – Aptitude – English - Current Affairs - Reasoning
Wednesday – Aptitude – Reasoning – Computer – English - Current Affairs
Thursday – Reasoning – Aptitude – Current Affairs – Computer - English
Friday – English – Computer - Current Affairs – Aptitude - Reasoning

PUZZLE-41
Directions (6-10): Study the following information carefully and answer the given
questions.
Gowri, Jadhav, Lakshita, Dulal, Pakshi, Wamika and Ramani are seven
people live on seven different floors of a building but not necessarily in the same
order. The lower most floor of the building is numbered 1, the one above that is
numbered 2 and so on till the topmost floor is numbered 7. Each one of them born
in different years. i.e. 1989, 1988, 1987, 1990, 1991, 1992 and 1993 (But not
necessarily in the same order). The one who born on 1989 lives immediately above
the one who born on 1993. Only one person lives between Jadhav and Pakshi.
Jadhav lives on one of the floors above Pakshi. Neither Lakshita nor Gowri born on
1990. Pakshi does not born on 1987. Gowri lives on an odd numbered floor but not
on the floor numbered three. The one who born on 1991 lives immediately above
Gowri. Only two people live between Gowri and the one who born on 1987. The one
who born on 1988 lives on one of the odd numbered floors above Dulal. Only three
people live between Lakshita and the one who born on 1988. The one who born on
1987 lives immediately above Lakshita. Wamika born on 1987.

6). Which of the following persons exactly sit between Ramani and Gowri?
The one who lives in sixth floor
Jadhav
The one who born on 1991
All of the above
None of these

7). Dulal born on which of the following year?


1991
1992
1993
1990
1989

8). How many persons stay between the one who born in 1991 and Wamika?
Three
Four
Five
None
One

9). Four of the following persons form a group, which of the following person
does not belong to that group?
Jadhav
Lakshita
Wamika
Dulal
Pakshi

10). Which of the following statement is true as per the given information?
5 – Pakshi - 1989
7 – Ramani - 1992
2 – Wamika - 1988
1 – Lakshita - 1990
6 – Jadhav - 1991
Answer Key:
Direction (01-05):

Students Monday Tuesday Wednesday Thursday Friday


Current
Sukumar Reasoning Computer Aptitude English
Affairs
Current
Gokul English Reasoning Aptitude Computer
Affairs
Current
Arvind English Aptitude Computer Reasoning
Affairs
Current
Sharma Aptitude English Computer Reasoning
Affairs
Current
Vikram Computer Reasoning English Aptitude
Affairs

e) 2. d) 3. b) 4. a) 5. c)

Direction (06-10):

Floor Persons Years


7 Ramani 1990
6 Jadhav 1991
5 Gowri 1988
4 Pakshi 1989
3 Dulal 1993
2 Wamika 1987
1 Lakshita 1992

6. d) 7. c) 8. a) 9. b) 10. e)
PUZZLE-42
Directions (1-5): Study the following information carefully to answer
these questions.
A, B, C, D, E, F and G are seven students of the Engineering college. All of them
belongs to different branch of Engineering: Mechanical, Chemical, Electrical, Computer
Science, IT, Textile and Electronics but not necessarily in the same order. Each one of
them likes different sports Snooker, Golf, Tennis, Badminton, Cricket, Football and
Volleyball not necessarily in the same order. All of them are of different height. D is
taller than C and F. E is taller than G.B studies in Mechanical and likes Football. E
studies in Computer Science but not likes Tennis or Cricket. The one who studies in IT
likes Snooker. F likes Golf but does not studies in Chemical or Electronics. C is taller
than E. B is taller than only A.A person who studies in Electrical likes Badminton. The
one who studies in Electronics does not like Cricket. G studies in IT and C likes
Tennis. The one who is tallest likes Badminton.

Q1.Who plays the Cricket?


D
(b)A (c)E
(d)D or E

None of these

Q2.E plays which game?


Badminton
Golf
Cricket
Volleyball
None of these

Q3.Which of the following combinations of branch-person-sports is definitely


correct?
Electrical – B –Badminton
Chemical – E – Volleyball
Electronics – D – Cricket
Chemical – D – Cricket
None of these

Q4.Who studies Textile?


C
D
G
F
None of these

Q5.Who is the Golfer?


C
F
D
Cannot be determined
None of these

Solutions(1-5):

D>C>E>G>B>A
From given information, we know about F that he is surely less than D and can
come anywhere from D to B.

S1. Ans.(b)
Sol.

S2. Ans.(d)
Sol.

S3. Ans.(e)
Sol.

S4. Ans.(d)
Sol.

S5. Ans.(b)
Sol.

PUZZLE-43
Directions (6-10): Study the following information carefully to answer
these questions.
Seven family members G, H, J, K, L, M and N are going to different countries viz. UK,
US, Germany, France, Italy, Russia, China not necessarily in the same order. Each
one of them has a different profession from amongst CA, Admin, Accountant, HR,
Manager, Engineer and Professor not necessarily in the same order.
G is the father of K's only brother L. The one who is wife of L is anEngineer and she is
going to France. The one who is brother of L is anAdmin and he does not go UK.H is
the sister-in-law of K. The person who is CA goes to US. M goes to Italy. The one who
is Professor goes to China. K is uncle of N.M and N are brothers. L is a Manager and he
goes to Germany. G is an HR.The one who is sister of N does not go to US.J is sister of
M.

Q6.Who is the Accountant?


J
N
K
M
None of these

Q7.Which of the following combinations of person, profession and country


is definitely correct?
G – HR – Germany
K – Professor - China
L – Manager - US
G – HR – UK
None of these

Q8.Who is going to US?


J
K
M
N
None of these

Q9. Granddaughter of G goes to which country?


Russia
China
US
(d)Chandigarh
None of these

Q10.Who is the Professor?


N
J
M
L
None of these

Solution(6-10):
S6. Ans.(d)
Sol.

S7. Ans.(d)
Sol.

S8. Ans.(d)
Sol.

S9. Ans.(b)
Sol.

S10. Ans.(b)
Sol.

PUZZLE-44
Directions (11-15): Read the following information carefully and answer
the questions given below.
Eight friends P,Q,R,S,T,U, V and W are seated in a straight line at an equal distance
between each other, but not necessarily in the same order. Some of them are facing
north and some are facing south. T is an immediate neighbour of one who is sitting at
an extreme end of the line. Only three people sit between T and V. S sits second to the
right of V. S does not sit at an extreme end of the line. W sits on the immediate left of P.
W is not an immediate neighbour of V. The immediate neighbour of P faces opposite
directions.(i.e. If one neighbour faces north then other faces south and vice versa.) The
persons sitting at the extreme ends faces opposite directions.(i.e. If one person faces
north then other faces south and vice versa.)Q sits second to the left of U. U faces
north. U is not an immediate neighbour of T. The immediate neighbours of U faces
same directions.(i.e. If one neighbour faces north then other also faces north and if one
neighbour faces south then other also faces south).Both T and Q face a direction
opposite to that of S.(i.e. If S faces north then T and Q faces south and vice-versa.)

Q11. As per the following arrangements, which of the following statements is not
true with respect to P . ?
P faces south.
P is fourth to the right of Q.
P is 2nd to left of T.
P is between V and W.
None of these

Q12. How many person sit on the left of Q ?


One
Two
Three
Four
None of these

Q13. What is the position of S with respect to P ?


Immediate left
Third to left
Third to right
Fourth to left
None of these

Q14. Which of the following is immediate neighbour of Q?


T,S
T,U
V,U
R,S
None of these

Q15 Four of the given five are alike in a certain way based on the given
arrangement and hence form a group. Which of them does not belong to
that group .?
R
S
P
U
V

Solutions(11-15):
S11. Ans.(c)
S12. Ans.(b)
S13. Ans.(c)
S14. Ans.(a)
S15. Ans.(d)

PUZZLE-45
Directions (6-10): Read the following information carefully and answer the
questions that follow.
Ten persons are sitting in two parallel rows containing five persons in each. In row 1, P,
Q R, S, and T are sitting and all of them are facing south. In row 2, A, B, C, D, and E are
sitting and all of them are facing north. In the given seating arrangement, each member
seated in a row faces another member of the other row but not necessarily in the same
order. Each of them is going to watch different movie viz. Bahubali, Alien, Half
Girlfriend, Radha, Posto, Dino Planet, Lahoriye, Sarkar, Rosetta, and Smurfs but not
necessarily in the same order.
There are two persons sitting between the person who is going to watch Bahubali who
sits at an extreme end, and T. A, who sits middle of the row, and not an immediate
neighbour of B, who is not going to watch Smurfs. E is sitting at an extreme end of the
row. T is going to watch Posto , sits on the immediate right of the person who is going to
watch Alien and faces the immediate neighhour of B. R does not sit at the extreme left
end of the row. A does not watch Rosetta. There is only one person between Q and S,
who is going to watch Bahubali. D is going to watch Dino Planet, is an immediate
neighbour of the person is going to watch Smurfs and does not face the person who is
going to watch Half Girlfriend. C, who is going to watch Radha, is an immediate
neighbour of the person who is going to watch Rosetta, who in turn faces the immediate
neighbour of the person who is going to watch Posto. There are two persons between
the person who is going to watch Lahoriye and the person who is going to watch Half
Girlfriend. R does not watch Lahoriye. E does not watch Sarkar.

Q6. Who among following sit at the extreme end of the line?
R,C
S,A
Q,B
E,T
B,S

Q7. The immediate neighbor of A faces whom among the following?


The person who who is going to watch Bahubali
The person who who is going to watch Half Girlfriend
The person who is going to watch Lahoriye.
The person who who is going to watch Rosetta.
Can't be determined

Q8.Who among the following sits diagonally opposite to R?


E-Smurfs
D-Dino Planet
A-Radha
E-Rosetta
None of these

Q9. Four of the following five are alike in a certain way based on their seating
positions and so form a group. Which of the following is different from the group?
T
Q
A
P
D

Q10. Which of the following statements is/are definitely false?


P-Lahoriye
E-Smurfs
T-Posto
C-Radha
All are true

Solution(6-10):

S6. Ans.(e)
S7. Ans.(c)
S8. Ans.(a)
S9. Ans.(a)
S10. Ans.(e)
PUZZLE-46
Directions (6-10): Study the information and answer the given questions:

Eight friends A, B, C, D, E, F, G, and H are sitting in a straight line but not necessarily in
the same order. Some of them are facing north while some are not. All of them like
different colour viz. violet, red, blue, black, white, yellow, pink and orange but not
necessarily in the same order. The following information is known about them.
A sits fourth to right of one who likes orange colour. H likes pink colour and sits second
to right of A. There are two friends sit between H and G, who likes yellow colour. One of
immediate neighbor of G sits extreme end. There are four friends sit between A and F,
who likes white colour. Immediate neighbor of G faces opposite direction(opposite
direction means if one neighbor of G faces north then other neighbor of G faces south
and vice-versa). E likes black colour and does not sit adjacent to H. The person who
likes red colour sits second to right of G. A does not like violet colour. The friends who
sit on the extreme end of line faces opposite direction(Faces opposite direction means if
one friend faces north then other friend faces south and vice-versa). B sits immediate
left of C. The one who likes red colour sits immediate right of H, who faces south
direction. C and D face same direction.

Q6. A likes which of the following colour?


black
yellow
red
blue
pink

Q7. How many persons faces north direction?


two
four
three
five
None of these

Q8. Who among following sits fifth to right of G?


D
H
C
B
F

Q9. Who among following likes violet colour?


D
H
C
B
F
Q10. If F is related to violet and A is related to black, in the same way H is related
to?
Blue
(b)Black
White
Red
Orange
Solution (6-10):

S6. Ans.(d)
S7. Ans.(c)
S8. Ans.(d)
S9. Ans.(c)
S10. Ans.(e)

PUZZLE-47
Directions (11-15): Study the information and answer the given questions:
Seven people P, Q, R, S, T, U and V live on separate floors of a 7-floor building.
Ground floor is numbered 1, first floor is numbered 2 and so on until the topmost floor
is numbered 7. Each one of them belongs to different cities, viz Noida, Delhi, Agra,
Meerut, Kanpur, Pune and Puri but not necessarily in the same order. Only three
people live above the floor on which P lives. Only one person lives between P and the
one belongs to Pune. U lives immediately below the one who belongs to Delhi. The
one belongs to Delhi lives on an even-numbered floor. Only three people live between
the ones belongs to Pune and Agra. T lives immediately above R. T does not belong to
Agra. Only two people live between Q and the one who belongs to Kanpur. The one
who belongs to Kanpur live below the floor on which Q lives. The one who belongs to
Noida does not live immediately above or immediately below Q. S does not live
immediately above or immediately below P. V does not belong to Meerut.

Q11. Which of the following is true with respect to V as per the given information?
The one who lives immediately below V is belong to Delhi.
V lives on floor no. 7.
V lives immediately below T.
V lives on the lowermost floor.
V belongs to Pune.

Q12. Who among the following lives on floor no. 3?


The one who belongs to Meerut
The one who belongs to Kanpur
R
V
T

Q13. Who lives on the floor immediately above T?


P
Q
S
V
U

Q14. S belongs to which of the following city?


Delhi
Pune
Agra
Kanpur
Meerut

Q15. How many people live between the floors on which S and the one who
belongs to Delhi?
None
Two
One
More than three
Three
Solution(11-15):
S11. Ans.(a)
S12. Ans.(b)
S13. Ans.(e)
S14. Ans.(e)
S15. Ans.(b)

PUZZLE-48
Directions (1-5): Study the following information carefully and answer
the questions given below:
There are eight friends A, B, C, D, E, F, G and H are sitting around a square table in
such a way that four of them sit at the four corners of the square table while four sit
in the middle of each of four sides. The one who sits at the four corners face the
center and those who sit in the middle of the sides face outward. Each of them likes
different colour, viz Red, Green, Violet, Black, White, Pink, Blue and Orange.
F sits third to left of the one who likes Green. The one who likes Green faces outward.
Only two friends sit between F and A. The one who likes Red sits on the immediate right
of A. The one who likes White sits second to right of B, who is not immediate neighbor
of A or F.B does not like Green. Only one friend sits between H and one who likes
White. E sits on the immediate left of the one who likes Black. B does not like Black. D
likes Violet but he is not immediate neighbor of H. The one who likes Orange is an
immediate neighbor of D. The one who likes Pink is an immediate neighbor of C. C is an
immediate neighbor of both E and the one who likes Orange.

Q1. Who among following likes Pink colour?


E
A
H
G
F

Q2. Who among following sits opposite to D?


G
F
C
B
A

Q3. Who among following sits second to right of D?


A
E
C
B
G
Q4. Four of the following five are alike in a certain way and hence they form
a group. Which one of the following does not belong to that group?
A
H
B
E
F

Q5. Who among following sits diagonally opposite to H ?


E
B
F
A
None of these

Solution(1-5):

S1. Ans.(a)
S2. Ans.(e)
S3. Ans.(c)
S4. Ans.(a)
S5. Ans.(b)

PUZZLE-49

Directions (1-5): Study the following information carefully and answer the
questions given below:

Farmer A of Nirman Nagar goes to trade fair with their family after completing their
daily work. There are nine members in the family after including farmer, i.e. A, B, C,
D, E, F, G, H and I. Each member eats different food items viz. Chocolate, Pizza,
Burger, Rasmalai, Samosa, Jalebi, Khulfi, Dosa and Idli. The relation of farmer with
the family members is defined as Father, Mother, Brother, Sister, Wife, Daughter,
Son, and Brother-in-law but not in the same order. All the family members are
enjoying circular ride, but not necessarily in the same order. Some of them are facing
towards centre while others are facing outward.
I is brother of A’s daughter. A’s brother is seated on the immediate right of the
brother of A’s sister and both face the same direction. I’s father eats Pizza. H has a
sister. The mother of A’s son is E and sits second to the left of the married son of A’s
father. F’s daughter-in-law eats Idli. A faces outward. F is a male. A’s father sits third
to the right A’s daughter. Either I or H sits third to the left of the mother of C, but
both I and H face the same direction. A’s brother and sister eats Burger and Chocolate
respectively. G is not father of H. In two generations, the members of the same
generation are sitting immediate next to each other and face the same direction. A’s
father does not eat Dosa and Khulfi. F’s grandson eats Rasmalai. B is a female and of
the same generation as of A. G sits on the immediate left of the granddaughter of A’s
father. The one, who is the wife of F likes Kulfi . A’s brother-in-law neither eats Dosa
nor Samosa. A’s father doesn’t sit immediate next to his children. G is facing the
centre and is second to the right of A. B sits second to the left of F. No three
members seated together can face the same direction.
Q1. How is B’s husband related to A’s father?
Brother
Father
Son
Grandson
Son-in-law

Q2. Who sits third to the right of A’s mother?


B
H’s brother-in-law
A’s brother-in-law
E’s husband
Both (b) and (c)

Q3. Who among the following sits exactly between A and his wife?
F
A’s sister
A’s daughter
D
None of these

Q4. Four of the following five are alike in a certain way and hence they form a
group. Which one of the following does not belong to that group?
C
F
H
I
A

Q5. Which of the following persons faces inside?


B, G, F and I.
A, G, D and I.
G, D, A’s father and G’s wife.
D, G, B and C.
(e) None of these

Solution(1-5):

S1. Ans.(e)
S2. Ans.(e)
S3. Ans.(b)
S4. Ans.(b)
S5. Ans.(c)

PUZZLE-50
Directions(6-10): Answer the questions on the basis of the information given
below.
Six kids– Z, X, C, V, B and N- watch in different Cartoon serials on their TV viz.
Ducktails, Doremon, Shinchain, Baloo, Chota beem and Hanuman – and they are lives
on six different floors of a building, numbered 1 to 6. They like six different Cartoon
Characters viz. Dholu, Sizuka, Munni, Lui, Jiyan and Pumba, in no particular order.
Each Cartoon serial is cast on a different day from Monday to Saturday. X likes Jiyan
and Cartoon serial likes by him is Doremon and lives on the 6th floor of the building. B
and Z lives on even numbered floors who’s cartoon cast on Saturday and Monday. The
one who watch Ducktails is lives on the lowest floor and it is cast on Tuesday. Baloo
Cartoon serial watch by that kid who lives on the floor next to X and it is cast on
Thursday. The kid who watches Hanuman likes Pumba and his cartoon is cast on
Monday. N likes Lui and he lives on first floor. One of the kid who watches Cartoon
serials which cast on friday lives on even numbered floors. V lives two floors below C
and he likes Munni. B, who does not live immediately above or below of C, likes Dholu
and his serial cast on Saturday. The kid who likes Chota beem is lives on an even
numbered floor.
Q6. Who is watching Ducktails cartoon serial?
Z
N
V
X
C

Q7. Which Character does B like?


Pumba
Jiyan
Dholu
Lui
Sizuka

Q8. Who among the following watch that cartoon which is cast on Thursday?
V
N
Z
X
C

Q9. If V is related to C and B is related to Z, in the same way N is related to?


X
N
Z
V
C

Q10. Which of the following combination is correct?


V – Shinchain – Sizuka
C – Baloo – Munni
Z – Chota beem – Pumba
X – Doremon – Dholu
N – Ducktails – Lui

Solution(6-10):
S6. Ans.(b)
S7. Ans.(c)
S8. Ans.(e)
S9. Ans.(d)
S10. Ans.(e)

PUZZLE-51
PUZZLE-52

PUZZLE-53
PUZZLE-54
PUZZLE-55
PUZZLE-56
PUZZLE-57
PUZZLE-58
PUZZLE-59
PUZZLE-60
PUZZLE-61
PUZZLE-62
PUZZLE-63
PUZZLE-64
PUZZLE-65
PUZZLE-66
PUZZLE-67
PUZZLE-68
PUZZLE-69
PUZZLE-70
PUZZLE-71
PUZZLE-72
PUZZLE-73
PUZZLE-74
PUZZLE-75

PUZZLE-76
Directions (1-5): Study the following information carefully and answer the
given questions.
There are seven boys Sudhan, Hassan, Nakul, Rushil, Vishnu, Ishat and Amir who
participated in a game competition which started on Monday and ended on Sunday. In
the first round of the competition, each of them played game under different teams
Red, Blue, Green, Yellow, Pink, Orange and White, but not necessarily in the same
order. They like different States, Maharashtra, Kerala, Assam, Bihar, Tamil
Nadu, Gujarat and Haryana, but not necessarily in the same order. Hassan did not
play on the day either immediately before or immediately after the game of Rushil,
who does not like either Tamil Nadu or Maharashtra or Bihar State. Two games
were held between the game of Amir and Ishat, neither of them played on Monday.
There was one game between the games of Rushil and Nakul. But Nakul’s game did
not happen either on Monday or on Wednesday. Nakul likes Kerala State and
played Red. The one, who played under Green team on the last day of competition,
likes Assam State. Vishnu played immediately after Nakul and he likes Maharashtra
State. Sudhan does not like Tamil Nadu State and played under Yellow team. Rushil
did not play for either White or Orange ream. The one who played for Pink team was
scheduled immediately after the game of Red team. Amir, who likes Gujarat, played
on the fourth day of the competition but played neither Red team nor Blue team.

1). Who among the following like Tamil Nadu state?


Hassan
The one who played for Yellow team
The one who played for Pink team
The one who played for Red team
Vishnu

2). The one who played on Friday likes which of the following state?
Tamil Nadu
Gujarat
Haryana
Kerala
Maharashtra

3). Who among the following like Bihar State?


Amir
Hassan
Sudhan
Vishnu
Ishat

4). Amir plays for which of the following team?


Orange
White
Red
Green
Either Orange or White

5). Which of the following statement is true?


Thursday – Amir – White - Haryana
Tuesday – Sudhan – Yellow - Bihar
Monday – Rushil - Blue - Haryana
Sunday – Ishat – Pink - Assam
Saturday – Vishnu - Orange - Maharashtra

PUZZLE-77
Directions (6-10): Study the following information carefully and answer the given
questions.

Seven friends namely Subha, Praveen, Zaheer, Ganesh, Bharathi, Kaviya and Nisar
attend different Exams coaching namely NIACL Clerk, SBI PO, BOB PO, NICL AO,
DENA PO, BOI PO and LVB PO, not necessarily in the same order, from Monday to
Sunday (of the same week). Subha attends a coaching on Friday. Only two people
attend coaching between Subha and the one who is attending NICL AO class. Kaviya
attends a coaching immediately before Ganesh, Neither Kaviya nor Ganesh is
attending NICL AO class. Only one person attends a coaching between Kaviya and the
one who is attending BOB PO. The one who is attending BOB PO class does not have
a coaching on Monday. Praveen attends a coaching immediately
before the one who is attending NIACL Clerk class. Subha is not attending NIACL
Clerk class. Only one person has a coaching between Kaviya and Bharathi. Zaheer
is attending BOI PO class. Only two people have coaching between Zaheer and
the one who is attending DENA PO class. Bharathi is not attending LVB PO class.

6). BOI PO class held on which of the following day?


Friday
Saturday
Sunday
Monday
Tuesday

7). How many persons attend the class between Bharathi and Nisar?
Four
Three
Five
Two
One

8). Praveen attends which of the following coaching class?


LVB PO
NICL AO
SBI PO
BOB PO
NIACL Clerk

9). Who among the following attend the class on Wednesday?


Bharathi
Ganesh
Nisar
Subha
e) Kaviya

10). Which of the following statement is true?


Sunday – Zaheer - BOB PO
Friday – Kaviya - BOI PO
Thursday – Ganesh - DENA PO
Tuesday – Bharathi - SBI PO
Wednesday – Praveen - NICL AO

Explanation with answers key:


Directions (Q. 1-5):

1). Answer: a)
2). Answer: d)
3). Answer: c)
4). Answer: e)
5). Answer: b)

Directions (Q. 6-10):


6). Answer: c)
7). Answer: a)
8). Answer: b)
9). Answer: e)
10). Answer: c)

PUZZLE-78
Directions (1-5): Study the following information carefully and answer the
given questions.

There are nine persons Ratan, Samar, Darshan, Gokul, Yoonus, Vasu, Manish, Faraz
and Bibek stay on a nine floors building, but not necessarily in the same order. Only
one person stays on each floor. All of them have either Computer or Laptop of different
companies i.e. Microsoft, Acer, Samsung, Lenovo, Apple, HP, Asus, Dell and Sony, but
not necessarily in the same order. Only five of them have Laptop. The ground floor is
numbered 1 and the topmost floor is numbered 9. There are two floors between the
floors on which the persons having a HP Computer and Dell Laptop stay. Darshan
owns a Samsung Computer. Vasu does not own a Sony Computer. Ratan stays on
even numbered floor below the even numbered floor on which Faraz stays. The one
who owns Apple Laptop stays on the fourth floor. Gokul stays on the second floor and
owns the Lenovo Laptop. The one who owns the Acer Laptop stays on the topmost
floor. Ratan does not own Asus Laptop. Vasu owns a
HP Computer and stays on an odd-numbered floor. There is only one floor
between the floors on which Vasu and Manish stay. Yoonus stays immediate
above the floor on which Bibek stay. Manish does not stay on the ground floor.
Samar stays immediate above the floor on which Faraz stay. There are three floors
between the floors on which Darshan and Vasu stay. The one who owns the Dell
Laptop stays immediate above Manish. Yoonus stays on the fourth floor. The one
who owns the Sony Computer stays on the third floor.

1). Vasu lives on which of the following floor?

1st floor
3rd floor
5th floor
7th floor
9th floor

2). Which company laptop used by the person who lives in 4th floor?
Apple
Lenovo
Asus
Acer
Dell

3). Four of the following five are alike in a certain way and hence they form a group.
Which one of the following does not belong to that group?
Ratan
Darshan
Vasu
Bibek
Manish
4). How many persons are between the one who uses Lenovo laptop and the
one who uses Microsoft computer?
Two
Three
Four
Five
One

5). Which of the following person uses Sony computer?


Gokul
Darshan
Faraz
Bibek
Samar

PUZZLE-79
Directions (6-10): Study the following information carefully and answer the
given questions.
Eight friends Deepan, Jalil, Nagaraj, Sharma, Hanuman, Tilak, Sukumar and
Prasanna live on eight different floors of a building but not necessarily in the same
order. The lowermost floor of the building is numbered 1 and the topmost floor of
the building is numbered 8. Each of them likes different players of grand prix viz,
Kimi Räikkönen, Daniel Ricciardo, Max Verstappen, Lewis Hamilton, Fernando
Alonso, Sebastian Vettel, Jenson Button and Nico Rosberg but not necessarily in
the same order.

The one who likes Daniel Ricciardo lives on an even-numbered floor but not on the
topmost floor. Only one person lives between Tilak and the one who likes Fernando
Alonso. Only two persons live between Tilak and the one who likes Daniel Ricciardo.
Neither Hanuman nor Nagaraj lives on the first floor. Only one person lives between
Nagaraj and the one who likes Max Verstappen. Deepan lives just above Tilak. Only
two persons live between Hanuman and Deepan. The one who likes Fernando
Alonso does not live on floor number one, Jalil lives on an even-numbered floor and
just above Nagaraj. The one who likes Lewis Hamilton lives on an even numbered
floor and lives just above the person who likes Nico Rosberg. Nagaraj does not like
Fernando Alonso or Nico Rosberg. Only two persons live between the one who
likes Sebastian Vettel and the one who likes Kimi Räikkönen . Sharma does not like
Jenson Button. The one who likes Sebastian Vettel does not live on an odd-
numbered floor. Prasanna lives just below the one who likes Nico Rosberg.

6). Sukumar likes which of the following player?


Jenson Button
Max Verstappen
Lewis Hamilton
Nico Rosberg
Kimi Räikkönen

7). Four of the following five are alike in a certain way and hence they form a group.
Which one of the following does not belong to that group?
Deepan
Nagaraj
Prasanna
Jalil
Sukumar

8). How many persons are between the one who likes Lewis Hamilton and the
one who stays on 4th floor?
One
Two
Three
Four
e) Five

9). Who lives on fifth floor?


Jalil
Hanuman
The one who likes Lewis Hamilton
The one who likes Fernando Alonso
Both option b and d

10). Which of the following combinations is true?


1 – Sharma - Jenson Button
4 – Hanuman - Daniel Ricciardo
3 – Tilak - Nico Rosberg
6 – Prasanna - Sebastian Vettel
7 – Nagaraj - Kimi Räikkönen

Explanation With Answers Key:


Directions (Q. 1-5):

1). Answer: c)
2). Answer: a)
3). Answer: e)
4). Answer: b)
5). Answer: d)

Directions (Q. 6-10):

6). Answer: a)
7). Answer: b)
8). Answer: c)
9). Answer: b)
10). Answer: d)

PUZZLE-80
Directions (1-5): Study the following information carefully and answer the
given questions.

Eight Bank MD and CEOs – Shika Sharma, Rakesh Sharma, Kishor Kharat,
Koteeswaran, Arundhati, Srinivas, Chanda Kochhar and Usha - have come together to
discuss a strategy for the development of the banks. They are sitting around a round
table such that exactly half of them are facing the centre. Each MD and CEO heads a
different bank viz, Indian Bank, Axis Bank, ICICI Bank, UCO Bank, Dena Bank, State
Bank of India, Canara Bank and Bank of India. Of these, Uco Bank and
Bank of India fall in the same zone. Kishor Kharat, Rakesh Sharma,
Koteeswaran and Srinivas are the male CEOs.
Rakesh Sharma, from Indian Bank and facing the centre, sits opposite to the CEO
from Dena Bank. Usha, an immediate neighbour of Rakesh Sharma, is fourth to the
right of the CEO from State Bank of India, who is facing the centre. Kishor Kharat,
from Canara Bank, sits second to the right of Srinivas and both are facing outside.
The CEO from ICICI Bank also the person whose bank name starts with the
Second alphabet letter is facing the centre. He is also to the immediate right of
Usha. To the immediate left of Koteeswaran is Arundhati, who is facing outside and
is opposite to Chanda Kochhar, who is the CEO from Bank of India and faces the
centre. The CEOs from the same zone are sitting opposite each other. Only two
males face the centre. No two people of the same gender sit next to each other.

1). Who sits second to the left of the person the one who represents for
Canara Bank?
The one who represents for Dena Bank
Shikha Sharma
Chanda Kochhar
The one who represents for Indian Bank
None of these

2). Usha represents for which of the following bank?


State Bank of India
Bank of India
ICICI Bank
Indian Bank
Axis Bank
3). How many persons sit between the person who represent for ICICI Bank and
the person who represent for Indian Bank? (Count from the left of the person who
represent for ICICI Bank)
Four
Two
Five
One
Three

4). Who represents for UCO Bank?


Usha
Arundhati
Shikha Sharma
Srinivas
None of these

5). Which of the following combinations is true?


Srinivas – Facing Centre – Dena Bank
Kishore Kharat – Facing Outside – UCO Bank
Koteeswaran – Facing Centre – State bank of India
Usha – Facing Outside – Axis Bank
None of these

PUZZLE-81
Directions (6-10): Study the following information carefully and answer the given
questions.
Eight persons Suresh, Prakash, Arun, Kavi, Mohan, Jaffar, Naveen and
Dinesh are seated around a circular table such that four faces the centre and four
are facing opposite to the centre. They like different Pro Kabaddi League teams:
Bengaluru Bulls, U Mumba, Puneri Paltan, Bengal Warriors, Jaipur Pink Panthers,
Patna Pirates, Telugu Titans and Dabang Delhi not necessarily in the same order.
The arrangement is based on the following rules:
One who likes Dabang Delhi is seated opposite to Suresh
Kavi is 3rd to the right of the one who likes Patna Pirates who is seated
between Prakash and the one who likes Puneri Paltan
Naveen is 3rd to the left of Arun who likes Telugu Titans
Dinesh is to the immediate left of the one who likes Jaipur Pink Panthers
One who likes Dabang Delhi is a neighbour of Kavi
Suresh does not like Patna Pirates and Naveen does not like Dabang Delhi
Suresh is 2nd to the left of Mohan who is not a neighbour of Kavi
Either Suresh or Naveen likes Jaipur Pink Panthers
Kavi and Prakash are seated facing the same direction.
The one who likes Puneri Paltan is 2nd to the right of the one who likes U
Mumba Kavi and Dinesh do not like U Mumba, Naveen faces outside the centre
One who likes Bengal Warriors is facing the opposite direction faced by Arun.
Not more than two persons who sits on consecutive seats face the same direction.

6). Who among the following likes Patna Pirates?


Suresh
Dinesh
Jaffar
Mohan
None of these

7). The only person who is sitting between Dinesh and Naveen likes which of
the following teams?
Puneri Paltan
Bengal Warriors
Jaipur Pink Panthers
Telugu Titans
e) Bengaluru Bulls

8). Who among the following is not facing centre?


Arun
Mohan
Suresh
Dinesh
Jaffar

9). Who is sitting third to the right of the person who like Telugu Titans team?
The one who likes Puneri Paltan team
The one who likes U Mumba team
The one who likes Dabang Delhi team
The one who likes Patna Pirates team
None of these

10). Which of the following is correctly matched?


Kavi – Facing Centre – Bengaluru Bulls
Dinesh - Facing Outside – Puneri Paltan
Jaffar – Facing Outside – Dabang Delhi
Arun – Facing Centre – Bengal Warriors
Prakash – Facing Outside – U Mumba

Answer Key:
1. a) 2. e) 3. c) 4. b) 5. d)

6. d) 7. c) 8. a) 9. b) 10. e)

PUZZLE-82
Directions (1-5): Study the given information carefully to answer the given questions.

Seven people - P, Q, R, S, T, U and V live on seven different floors of a


building but not necessarily in the same order. The lower most floor of the building
is numbered 1, the one above that is numbered 2 and so on till the top most floor is
numbered 8. Each one of them likes a different watch— Casio, Citizen, Fossil,
Seiko, Tissot and Fastrack, but not necessarily in the same order.
U lives on floor numbered 4. Only two people live between U and the one
who likes Citizen. Only three people live between the one who likes Citizen and the
one who likes Fastrack. S lives on one of the even numbered floors above the one
who likes Fastrack. Only one person lives between S and the one who likes Seiko.
V lives one of the odd numbered floors below the one who likes Seiko. Only one
person lives between V and the one who likes Fossil. P lives with immediately
above or immediately below the one who likes Fossil. T lives immediately above P.
Only one person lives between Q and the one who likes .Casio. The one who likes
Tissot lives on one of the floors above the one who likes Omega.

1). Who lives immediately below S ?


T
Q
U
The one who likes Omega
The one who likes Tissot

2). How many people live above the one who likes Seiko ?
More than three
Two
None
Three
One

3). Which of the following watches does T like ?


Omega
Fastrack
Fossil
Tissot
Casio
4). On which of the following floor numbers does Q live ?
7
5
3
2
6

5). How many people live between the one who likes Fastrack and the one who
likes Casio ?
None
One
More than three
Two
Three

Answer Key:
1).b 2).d 3).c 4).b 5).b
Explanation:
Directions (Q. 1-5):
PUZZLE-83
Directions (1-5): Study the following information carefully and answer the given
questions.
Nine persons Lalith, Devan, Mahendran, Ragu, Indran, Samuel, Vadivel,
Mani, Sathish are sitting in a row facing in either north or south direction but not
necessarily in the same order. They work in different companies they are Sony,
Nestle, Microsoft, IBM, Wipro, Bajaj, Apple, Samsung and TCS but not
necessarily in this order. The following information is known about them.

Samuel and Sathish are immediate neighbours of Mahendran but face in opposite
directions to each other. Vadivel and Mani face in the same direction. Lalith who
works in Wipro sits in the middle and faces towards south. The person working in
Bajaj sits to the immediate right of a person facing in north direction. The person
sitting at first and last position from left face in north direction and they work in Apple
and Microsoft respectively. The person working in Samsung sits to the immediate
right of Devan. Samuel does not sit with Lalith. Indran sits second to the left of Ragu
but does not work in Microsoft. Mani sits two places away from Lalith. Ragu sits
exactly between Vadivel and Mani. Also, there is only one person sitting between
Vadivel and Mani. Devan and Mahendran face in the same direction. The person
who works in Nestle sits fourth to the left of the person working in Bajaj but neither
of them is Mani. The persons working in TCS and Sony are sitting adjacent to each
other. The person working in Sony sits fourth to the right of Samuel.

1). In which of the following company Vadivel works?


IBM
Bajaj
Samsung
TCS
Microsoft

2). The one who works in Wipro sits third to the left of ______________
Vadivel
Samuel
The one who works in TCS
The one who works in Nestle
Both option c and d

3). Who works in Sony Company?


Devan
Samuel
Mani
Ragu
Indran

4). Which of the following persons sits at the extreme ends of row?
Ragu and Mahendran
Devan and Sathish
Mani and Vadivel
Sathish and Indran
Indran and Lalith

5). Which of the following combinations is true?


Vadivel - Bajaj – Facing South
Mahendran – Nestle – Facing South
Indran – Sony – Facing North
Ragu – Apple – Facing South
None of these
PUZZLE-84
Directions (6-10): Study the following information carefully and answer the given
questions.

Virat Kohli, Saina Nehwal, Abhinav Bindra, Vijender Singh, Anwar Ali and
Sardar Singh are six sports players, who belong to six different states viz–Haryana,
Punjab, Maharashtra, Assam, Bihar and Rajasthan but not necessarily in the same
order, each one has won the trophy in different countries, viz Australia, Malaysia,
Singapore, China, Argentina and Brazil but not necessarily in the same order. They
are sitting around a circular table facing the centre. The persons who have won the
trophy in China, Singapore and Argentina are neither from Rajasthan nor from
Haryana. The persons who are from Assam and Punjab have won the trophies
neither in China nor in Brazil. Virat Kohli is neither from Rajasthan nor sits on the
immediate left of the person who won the trophy in Malaysia. The only person who
is between Anwar Ali and Sardar Singh has won the trophy in Australia. The person
who is on the left side of the person from Rajasthan has not won the trophy in Brazil.
Vijender Singh has won the trophy in Malaysia and is from Punjab. He is facing the
person who has won the trophy for Australia. One who has won the trophy for China
is sitting opposite the person from Bihar, while the person from Punjab is on the left
of the person who has won the trophy in Argentina. One who has won the trophy in
Brazil is on the immediate right of the person from Rajasthan but on the left of the
person who has won the trophy in Singapore. Abhinav Bindra has not won the
trophy in Singapore and Sardar Singh has not won the trophy in China.

6). Which of the following player belongs to Rajasthan?


Abhinav Bindra
Virat Kohli
Saina Nehwal
Anwar Ali
Sardar Singh
7). Saina Nehwal belongs to which of the following state?
Punjab
Rajasthan
Haryana
Maharashtra
Bihar

8). Who sits second to the left of the player the one who won the trophy in Malaysia?
Virat Kohli
The player who won the trophy in China
The player who belongs to Rajasthan
The player who won the trophy in Brazil
The player who belongs to Maharashtra

9). Which of the following player won the trophy in China?


Abhinav Bindra
Anwar Ali
Saina Nehwal
Vijendar Singh
Virat Kohli

10). Which of the following combinations is true?


Abhinav Bindra – Haryana – Australia
Anwar Ali – Punjab - China
Virat Kohli – Assam - Argentina
Sardar Singh – Bihar - Singapore
None of these

Answer Key:
1). b) 2). e) 3). d) 4). d) 5). a)

6). a) 7). e) 8). d) 9). b) 10). c)

PUZZLE-85

Directions (1-5): Study the following information carefully and answer the
given questions.

Seven people namely Sahith, Umesh, Maaran, Duruvan, Wasim, Aadhav and
Gopi buy different colour shirts viz. Red, Blue, Green, Yellow, White and Orange and
different colour pants viz. Black, Grey, Sandal and Brown from different shops say
Kumaran Silks, Saravana Stores, Chennai silks and Pothys. Not more than two people
buy from same shop. It is known that each person buys both the items from the same
shop. The people who buy Blue colour shirt get from the same shop and they don’t buy
the same colour pants and is neither Sahith nor Gopi. Aadhav buys
from Pothys. Duruvan and Aadhav buy same colour pants which are bought by no
one apart from them and they get from different shops. The only person who orders
Brown colour pant from Chennai silks and is neither Umesh nor Maaran. Duruvan
doesn’t shop on Kumaran Silks. Orange colour shirt is get from Kumaran Silks but
not by Sahith. Maaran buy Green colour shirt. One who buys Black colour pant buys
from Kumaran Silks and is not Umesh and also buys either Red or White colour
shirt. The persons who buy Grey colour pant get on different shops but not on
Saravana Stores. Only one person buys Black colour pant. Maaran doesn’t get
Sandal colour pant. Aadhav neither orders Red or Yellow colour shirt.

1). Who among the following person shopping in Saravana Stores?


Sahith
Maaran
Duruvan
Umesh
Gopi

2). Who among the following persons bought the Grey colour pant?
Gopi
Maaran
Umesh
All of the above
None of these

3). Which of the following colour shirt and pant bought by Duruvan?
Red and Grey
Yellow and Sandal
Orange and Brown
Yellow and Black
White and Sandal
4). Which of the following colour shirt bought by two persons?
Blue
Orange
Green
Red
White

5). Which of the following combinations is true?


Wasim – Blue – Brown – Kumaran silks
Maaran – Green – Black - Pothys
Sahith – Red – Grey - Kumaran Silks
Gopi - White - Grey - Kumaran Silks
e) e) Umesh - Blue - Grey - Chennai silks

PUZZLE-86
Directions (6-10): Study the following information carefully and answer the given
questions.
There are eight people namely A, B, C, D, E, F, G and H who sit around a
circular table some of them facing towards centre and some of them facing away
from the centre. They like to buy different mobiles. They are Samsung Galaxy C7
Pro, Samsung Galaxy J2 Prime, Samsung Galaxy J5, Samsung Galaxy On5,
Samsung Galaxy J2 Ace, Samsung Galaxy J1, Samsung Galaxy A7 and
Samsung Galaxy C9 Pro. The following information is known about them.

H’s neighbours face in same direction but opposite to that of H. G sits third to the left of
E who faces towards the centre. D’s spouse who likes to buy Samsung Galaxy J5 sits
third to the left of D. C sits third to the right of a male and they face in the same
direction. B’s immediate neighbours face in same direction but opposite to that of B.
The female who likes to buy Samsung Galaxy J2 Prime sits opposite to the male who
buys Samsung Galaxy C7 Pro. A who wants Samsung Galaxy A7 sits second to the left
of D and they face in the same direction. The immediate neighbours of C
like to have Samsung Galaxy J2 Prime and Samsung Galaxy C9 Pro. B and F sit
together and none of them sits with A and D. G’s spouse who likes to buy Samsung
Galaxy On5 sits second to the left of G. The immediate neighbours of H’s only
sister who are neither C nor E are A and D. B who buys Samsung Galaxy J1 is the
only one whose both the immediate neighbours are of opposite gender to B.

6). Who likes to buy Samsung Galaxy J2 Prime?


B
E
D
H
A

7). How many persons facing centre?


Five
Six
Four
Three
Two

8). Who is sitting second to the left of person who likes to buy Samsung Galaxy J1?
The one who uses Samsung Galaxy J2 Ace
The one who uses Samsung Galaxy C9 Pro
The one who uses Samsung Galaxy J1
The one who uses Samsung Galaxy J2 Prime
The one who uses Samsung Galaxy J5

9). How many persons sit between who likes to buy Samsung Galaxy J5 and who
likes to buy Samsung Galaxy J2 Ace, Count from the right of the person who likes
to buy Samsung Galaxy J5?
a) 1
5
3
2
4

10). Which of the following is correctly matched?


G – Male – Samsung Galaxy J2 Ace
H – Female – Samsung Galaxy A7
A – Female – Samsung Galaxy J1
C – Male – Samsung Galaxy On 5
None of these

Answer Key:

Direction (01 to 05):


Person Shirt Colour Pant Colour Shops
Sahith Red Black Kumaran Silks
Umesh Blue Grey Chennai silks
Maaran Green Grey Pothys
Duruvan Yellow Sandal Saravana Stores
Wasim Blue Brown Chennai silks
Aadhav White Sandal Pothys
Gopi Orange Grey Kumaran Silks

1. c) 2. d) 3. b) 4. a) 5. e)

Direction (06 to 10):


6. c) 7. a) 8. e) 9. b) 10. d)

PUZZLE-87
Directions (1-5): Study the following information carefully and answer the
given questions.
Ishanth, Thilak, Yaswanth, Gulam, Charles, Kavin, Rashid, Bhasker, Jeyaraj and
Eeswar have some Bikes and cars. Both kind of vehicles are 1-10 in number. No
2 persons have the same number of same vehicles. It is also known that:
Thilak has 6 bikes more than Rashid.
Thilak has 10 vehicles in total.
Eeswar has 3 cars.
The total number of cars owned by Yaswanth and Jeyaraj is equal to the number
of cars that Bhasker has. Yaswanth has less number of cars than Jeyaraj.
Only Bhasker has 18 vehicles in total such that the number of cars he has is
more than his bikes.
Only Yaswanth has equal number of bike and
cars. Only Rashid has cars twice than the bikes.
Ishanth has 9 vehicles in total. He has more cars than bikes.
The number of bikes owned by Gulam is equal to the number of cars owned
by Charles.
Charles has 8 vehicles in total.
Charles and Kavin have 16 cars in total.
Jeyaraj and Eeswar have 16 bikes in total.

1). Who among the following have 3 bikes?


Bhasker
Eeswar
Rashid
Kavin
Thilak

2). What is the sum of total number of vehicles owned by Gulam and
Jeyaraj together?
25
20
17
22
28

3). Who among the following have 10 cars?


Yaswanth
Bhasker
Charles
Jeyaraj
Gulam

4). Which of the following combinations is true?


Rashid – 6 bikes – 3 cars
Eeswar - 10 bikes – 8 cars
Thilak – 1 bikes – 9 cars
Gulam – 7 bikes – 4 cars
None of these

5). If ‘Ishanth’ is related to ‘Gulam’, ‘Charles’ is related to ‘Thilak’, then which


following person is ‘Rashid’ related to?
Yaswanth
Gulam
Kavin
Ishanth
Eeswar

PUZZLE-88
Directions (6-10): Study the following information carefully and answer the given
questions:
Six cricketers Amla, Maxwell, Watson, Gayle, Narine and Lynn are going to
play for IPL Teams namely Punjab, Kolkata and Bangalore. They are coming in three
different Cars such as, Hyundai, Ford and Toyota. The Cricketers are making
presence in advertisements such as, Samsung, Oppo, Vivo, Motorola, Micromax and
Gionee not necessary in the same order. And they like different bats they are Adidas,
GM Icon, Puma, SS Ton, Reebok not necessary in the same order.

One of the players from Bangalore and Punjab teams coming through same cars.
Amla who belongs to the Kolkata team does not come with Maxwell by Toyota.
The one who presenting ad in Oppo and likes GM Icon bat going to play with
the cricketer who likes SS Ton bat. One of the player from Punjab presenting ad
in Motorola.
Each cars carries same number of cricketers and one of the cricketers comes by
Toyota, playing for Punjab with his team mate Watson. Amla not making presents
in Motorola and Gionee ads.
Lynn and Gayle not comes by same Cars but plays in same team. And one of them,
makes presence in Vivo ad and also likes Puma bat.
Maxwell and Narine did not play for the same team but they comes by same
cars. The two players who comes by Ford like SS Ton bat and both of them not
play for Kolkata.
Lynn does not plays for Punjab And Maxwell do not like Adidas bat and SS Ton
bat. Amla and Gayle do not come by Ford.

6). Which of the following player presenting in the Gionee advertisement?


Watson
Narine
Amla
Maxwell
Lynn

7). Which of the following players come by the Toyota car?


Amla and Gayle
Gayle and Lynn
Watson and Maxwell
Narine and Maxwell
Gayle and Lynn

8). Which of the following players like the SS Ton bats?


Watson and Lynn
Amla and Lynn
Narine and Amla
Gayle and Maxwell
None of these

9). The person who presenting in Samsung advertisement like which of the
following bats?
Puma
Adidas
Reebok
SS Ton
GM Icon

10). Which of the following combinations is true?


Narine - Kolkata – Micromax – Adidas - Toyota
Lynn - Bangalore – Gionee - SS Ton - Hyundai
Watson - Punjab - Motorola - SS Ton - Ford
Maxwell – Punjab – Samsung - GM Icon - Toyota
None of these

Answer Key:
Direction (01 to 05):
Name Bikes Cars
Rashid 3 6
Bhasker 8 10
Yaswanth 2 2
Gulam 7 4
Ishanth 4 5
Thilak 9 1
Charles 1 7
Kavin 5 9
Jeyaraj 6 8
Eeswar 10 3
1. c) 2. a) 3. b) 4. d) 5. e)
Direction (06 to 10):

6. e) 7. d) 8. a) 9. b) 10. c)

PUZZLE-89
Directions (1-5): Study the following information carefully and answer the
given questions.
Ten persons from different banks viz. Allahabad Bank, Bank of India, Canara
Bank, Dena Bank, Syndicate Bank, UCO Bank, Indian Bank, Andra Bank, Vijaya
Bank and Bank of Baroda are sitting in two parallel rows containing five people
each, in such a way that there is an equal distance between adjacent persons. In
row-1 Arun, Barvesh, Jagadish, Naren and Prathap are seated and all of them are
facing south. In row-2 Gowshiik, Vikaas, Chandru, Raagavan and Krishnan are
seated and all of them are facing north. Therefore in the given seating
arrangement, each member seated in a row faces another member of the other
row. (All the information given above does not necessarily represent the order of
seating in the final arrangement.)

Gowshiik faces one of the immediate neighbours of the person from Bank of India.
Gowshiik does not face Arun. The person from Allahabad Bank sits second to the right
of the person from Andra Bank. Naren faces one of the immediate neighbours of the
person from Canara Bank. Raagavan is not from Canara Bank. Naren is not from
Syndicate Bank. Chandru sits second to the left of the persons from Bank of Baroda.
Arun sits third to the right of person from UCO Bank. Only one person sits between
the person from Vijaya Bank and Vikaas. The person from Dena Bank sits to the
immediate right of Vikaas. Jagadish sits to the immediate left of the person
who faces Vikaas. Only two people sit between Barvesh and Prathap. The person
from Syndicate Bank sits second to the right of the one who faces Raagavan.
Raagavan does not sit at an extreme end of the line. One of the immediate
neighbours of the person from Syndicate Bank faces Vijaya Bank.
1). Which of the following Bank likes by Chandru?
UCO Bank
Andra Bank
Dena Bank
Allahabad Bank
Indian Bank

2). Which of the following persons sits at the extreme ends of one row?
Arun and Barvesh
Chandru and Gowshiik
Naren and Jagadish
Vikaas and Raagavan
Prathap and Arun

3). Who sits second to the left of the person the one who likes Canara Bank?
Krishnan
The one who likes Syndicate Bank
The one who likes Dena Bank
Chandru
None of these

4). Who likes the Vijaya Bank?


Raagavan
Arun
Krishnan
Jagadish
e) Naren

5). The one who likes Bank of Baroda sits opposite to the _____________
The one who likes Andra Bank
The one who likes Allahabad Bank
Naren
Both a and c
None of these

PUZZLE-90
Directions (6-10): Study the following information carefully and answer the given
questions.

Nine people Daniel, Akash, Giri, Kiran, Sadiq, Yogendra, Firoz, Hemant and
Balaji stay in a building. The building has nine floors and only one person stays on
one floor. Each of them likes different company shoes namely- Reebok, Bata, Nike,
Adidas, Liberty, Woodland, Puma, Paragon and Action. Each person belongs to
different states, i.e. Bihar, Haryana, Karnataka, Kerala, Maharashtra, Rajasthan,
Punjab, Gujarat and Sikkim but not necessarily in the same order. The ground floor
is numbered 1, the floor above it is numbered 2, and so on, and the topmost floor is
numbered 9.

The one who belongs to Gujarat stays on the 4th floor. There are 4 floors between the
floors on which Kiran & Firoz stay with Kiran living above Firoz. Giri stays on the floor
immediately above Hemant's floor. The one who likes Reebok stays on an even
numbered floor and is a neighbour of the one who likes Bata. Balaji does not belong to
Haryana. The one who belongs to Bihar stays on the topmost floor. The one who likes
Puma stays immediately below the one who likes Liberty. Sadiq likes Paragon.
Yogendra belongs to Maharashtra and stays on 8th floor. There are 3 persons between
the one who likes Adidas and the one who like Paragon. There are two floors between
the floors on which people from Sikkim and Gujarat stay. Sadiq stays immediately
below the floor on which the person from Kerala lives. The person who
likes Woodland belongs to Haryana and stays on an odd numbered floor. Balaji
stays on the bottommost floor. The person who likes Nike is from Kerala and
stays on 6th floor. The person from Punjab stays on 2nd floor. The persons from
Rajasthan and Haryana are not the neighbours of the person from Kerala. Kiran
likes Bata and Firoz does not like Reebok. Akash does not live below the floor of
Daniel.

6). Who lives in the Third floor?


Sadiq
The one who likes Action shoes
The one who is from Haryana
Yogendra
None of these

7). Nike Shoes like by which of the following person?


Kiran
Daniel
Firoz
Balaji
Giri

8). Which of the following persons sits exactly between Balaji and the one who likes
Action shoes?
Hemant and Firoz
Giri and Sadiq
Firoz and Kiran
Daniel and Hemant
Akash and Sadiq
9). How many persons sit between Firoz and the one who likes Adidas?
Five
Three
Four
Two
Six

10). Which of the following combinations is true?


4 - Gujarat - Giri - Adidas
3 –Haryana – Firoz - Liberty
8 - Bihar – Akash - Adidas
7 - Sikkim - Kiran - Bata
None of these

Answer Key:

1. e) 2. b) 3. a) 4. c) 5. d)
Floor States Names Shoes
9 Bihar Akash Adidas
8 Maharashtra Yogendra Reebok
7 Sikkim Kiran Bata
6 Kerala Daniel Nike
5 Karnataka Sadiq Paragon
4 Gujarat Giri Action
3 Haryana Hemant Woodland
2 Punjab Firoz Liberty
1 Rajasthan Balaji Puma
6. c) 7. b) 8. a) 9. e) 10. d)

PUZZLE-91
Directions (1-5): Study the following information carefully and answer the
given questions.
8 people Loknath, Rohan, Gupta, Prabhas, Habib, Nitin, Tanvir and Wahid
sit around a circular table facing the center. They belong to Austria, Kuwait, China,
Japan, USA, UK, Malaysia and Singapore. They belong to different ages youngest
being 17 and oldest being 64 years old.

One from Singapore is as much older than Gupta, who is from Japan, as the age of
Loknath. Difference of ages of Prabhas and Rohan is 41. One who belongs to UK sits
2nd right to one from Austria. One from Japan sits 2nd left to one from Singapore. Age
of Habib is 36 years. Loknath is 3rd youngest among all the people. Two people sit
between Nitin and the one whose age is 35. One who belongs to Malaysia and one
who belongs to Singapore sit adjacent to each other and Prabhas is not from Malaysia.
Wahid is the youngest of them and Prabhas is not the oldest. One who belongs to
China is opposite to one whose age is 47. One person sits between Rohan, who is
from USA, and Habib. Nitin sits 3rd left of the from UK who is the oldest person them.
Loknath is sitting 3rd left from the one who is from China.
Sum of ages of Rohan and Wahid is equal to the age of Gupta. One whose age is
35 sits 3rd right of the one whose age is 18. The age of Nitin is average of ages of
Gupta and Prabhas. One from USA and UK are sitting adjacent to each other.
One person sits between Nitin and one who is from Malaysia.

1). Who sits third to the left of the person the one who is from Japan?
Loknath
Rohan
Tanvir
Habib
Nitin

2). What is the age of Nitin?


18
36
47
24
31

3). Which of the following persons belongs to China?


Habib
Tanvir
Rohan
Gupta
Nitin

4). Wahid belongs to which of the following country?


Kuwait
UK
Japan
Austria
e) Malaysia

5). Which of the following combinations is true?


Prabhas – Malaysia - 59
Gupta – Japan - 36
Loknath – Kuwait - 24
Tanvir – UK - 64
None of these

PUZZLE-92
Directions (6-10): Study the following information carefully and answer the given
questions.

Six friends Aditi, Geeta, Mayuri, Bhuvana, Vasanti and Leela stay on a 6
floor building with the ground floor numbered as floor number 1 and so on. All the
friends have dresses on different colours such as; Red, Blue, Green, Yellow,
Orange and Pink not necessarily in the same order. They also like different days
via; Sunday, Monday, Tuesday, Wednesday, Thursday and Friday not necessarily
in the same order. The arrangement is based on the following rules:
The person on the top floor having dress on colours either Pink or Green
The one who likes Monday lives 2 floors below the one who has dress as Green
Vasanti likes Friday and lives on an even numbered floor
One who have dress on Green colour likes either Wednesday or
Tuesday Mayuri lives 4 floors below Geeta
One who lives on 2nd floor has the dress of Yellow colour and likes neither
Friday nor Monday
Bhuvana’s dress colour is Orange and she lives 2 floors above
Mayuri Mayuri does not like Yellow
One who has the dress of Red colour likes Thursday
One who likes Wednesday lives on an odd numbered floor
Aditi likes Tuesday and stays just above the person who likes
Wednesday One who has Orange colour dress does not like Wednesday.

Which of the following person like Monday?


Geeta
Leela
Mayuri
Aditi
Bhuvana

The one who likes Tuesday have which of the following colour dress?
Pink
Green
Orange
Blue
Red

Which of the following colour dress and day like by Mayuri?


Blue - Tuesday
Red - Thursday
Green - Friday
Pink - Monday
Blue - Wednesday

Who lives in the second floor?


The one who likes Sunday
Leela
The one who have Yellow colour dress
All of the above
None of these

10) Which of the following combinations is true?


3 – Bhuvana – Orange - Friday
1 - Mayuri - Yellow- Thursday
5 – Geeta – Green - Wednesday
6 – Aditi –Orange- Tuesday
None of these

Explanation With Answer Key:

Directions (Q. 1-5):

Answer: b)
Answer: c)
Answer: a)
Answer: e)
Answer: d)
Directions (Q. 6-10):

Answer: e)
Answer: a)
Answer: b)
Answer: d)
Answer: c)

PUZZLE-93

Directions (1-6) : Read the following information and answer the given
question.Seven people, namely O, P, Q, R, S, T and U have to attend a practical but
not necessarily in the same order, on seven different months (of the same year)
namely February, March, April, June, August, September and November. Each of
them also likes a different subject namely Chemistry, Biology, Psychology, Hindi,
English, Geography and Account but not necessarily in the same order.

The one who likes Hindi will attend a practical in a month which has 31 days. Only
two people will attend a practical between the one who likes Hindi and T. Only three
people will attend a practical between T and Q. Only one person will attend a
practical between Q and the one who likes Accounts. The one who likes Chemistry
will attend a practical in one of the months before the one who likes Accounts. The
one who likes Chemistry will attend a practical in the month which has less than 30
days. Only two people will attend a practical between the one who likes Chemistry
and S.
Only one person will attend a practical between S and the one who likes
Psychology. The one who likes Psychology will attend a practical on one of the
months after S. Only two people will attend a practical between the one who likes
Psychology and P. The one who likes Geography will attend a practical immediately
before P. Only three people will attend a practical between P and the one who likes
English. Only one people will attend a practical between O and R. O will attend a
practical in a month which has 31 days.
1). Who amongst the following likes Biology ?
P
T
U
R
O

2). How many people will attend a practical after P ?


Three
One
Two
None
More than three

3). Which of the following represents the people who will attend a practical
immediately before and immediately after O ?
T, P
P, Q
S, Q
S, P
U, P
4). As per the given arrangement February is related to T and March is related to P
following a certain pattern, which of the following is August related to following the
same pattern ?
U
Q
R
S
O

5). Which of the following represents the month in which U will attend a practical?
November
February
April
June
Cannot be determined
6). Who amongst the following likes English ?
S
T
U
R
Other than those given as options

PUZZLE-94
Directopms (7-12) : Study the following information and answer the given questions.
Seven people namely D, E, F, G, H, I and J like seven different monuments namely,
Taj Mahal, Red Fort, Charminar, India Gate, Qutub Minar, Victoria Memorial and
Sanchi Stupa. Each of them work in either of the three fields viz. Economics,
Management and Pharmacy with atleast two of them in a field. (Note.: None of the
information given is necessarily in the same order.) The one who likes Qutub Minar
works in field of Management only with G. The one who likes Charminar works with
the one who likes Sanchi Stupa. J works with the one who likes India Gate. I neither
works with G nor in the field of Pharmacy. J does not like Charminar. D likes Red
Fort. D does not work with J. F works with only one person. F does not like Qutub
Minar. H works with I. I does not like Charminar. Neither G nor F likes Taj Mahal.

7).Which of the following combinations represents the combination of people who


work in the field of Economics ?
The ones who like Victoria Memorial, Taj Mahal and India Gate.
The ones who like Charminar and Sanchi Stupa.
The ones who like Char Minar, Sanchi Stupa and Victoria Memorial.
The ones who like Taj Mahal and India Gate.
The Ones who like Sanchi Stupa, Red Fort and Char Minar.

8). Which of the following statements is true ?


H likes Sanchi Stupa
E works only with J.
All of the given statements are true.
Both E and G work in the, same field.
Only two people work in ti field of economics.

9). Which of the following combinations represents th`e field in which H works and
the monument he likes ?
Economics-Taj Mahal
Pharmacy-India Gate
Management- India Gate
Economics-Charminar
Pahrmacy-Charminar
10). Four of the following five are alike in a certain way as per the given
arrangement and hence form a , group. Which of the following does not belong to
that group ?
HI
JF
GE
EJ
ID

11). Which of the following monuments does E like ?


Taj Mahal
Qutub Minar
Sanchi Stupa
Char Minar
India Gate

12). Who amongst the following likes Victoria Memorial ?


a) H b) J c) I d) G e) F
Answer Key:

Month People Subject


February U Chemistry
March T Geography
April P Biography
June S Accountant
August O Hindi
September Q Psychology
November R English

1). A 2). E) 3). C 4). B 5). B 6).D


Management E Qutub Minar
G Victoria Memorial
Economics I Sanchi Stupa
H Charminar
D Red Fort
Pharmacy F India Gate
J Taj Mahal

7). E 8). D 9). D 10). D 11).B 12). D

PUZZLE-95
Directions (1-5): Study the following information carefully and answer the
given questions.
The seven mobile manufacturing companies Samsung, Nokia, Lenovo,
HTC, Oppo, Motorola, Vivo are having their plant in places namely Maharashtra,
Andra Pradesh, Karnataka, Haryana, Tamil Nadu, West Bengal and Bihar not in
the respective order. And the companies making contract with Sim dealers such as
AIRTEL, VODAFONE, IDEA, BSNL, AIRCEL, DOCOMO and JIO. The Sim dealers
are located in different places such as Chennai, Hyderabad, Bangalore, Delhi,
Mumbai, Kolkata and Patna not in the respective order.
JIO and BSNL are not contact with Andra Pradesh and West Bengal based
companies respectively. Either Samsung or Vivo likes to contract with IDEA which is
not located in Kolkata and Hyderabad
The dealer which located in Mumbai contract with the company that is located
in Haryana.
Motorola contract with Hyderabad based dealer and is not located in Tamil
Nadu and Andra Pradesh.
DOCOMO is Bangalore based dealer. And Kolkata based dealer not deals with
Vivo and Samsung.
Maharashtra and Karnataka based companies making contract with Delhi and Patna
based Sim dealers respectively.
Samsung is not serving to the Delhi and Mumbai based dealer. Lenovo is
having plant in Bihar.
HTC do not dealing with dealers of Chennai, Mumbai and also DOCOMO dealer
The Mumbai based VODAFONE does not contract with Oppo and Nokia. Vivo which
is located in Haryana doesn’t deal with Tamil Nadu based AIRTEL dealer.
DOCOMO will contract with Bihar based company.
Chennai based dealer making deal with Oppo which is not located in Andra
Pradesh and West Bengal
AIRCEL is not from Patna and Kolkata and makes deal with Nokia Company

1). HTC Contract with which of the following dealer?


AIRCEL
BSNL
IDEA
JIO
AIRTEL

2). VODAFONE contract with which of the following companies?


Lenovo
Motorola
Oppo
Vivo
Nokia

3). Which of the following company plant located in Tamil Nadu?


Vivo
Lenovo
Motorola
Nokia
Oppo

4). Which of the following combinations is true?


Nokia – Tamil Nadu – AIRCEL - Chennai
Samsung – Haryana – VODAFONE - Mumbai
HTC - Andra Pradesh – BSNL - Kolkata
Oppo – Maharashtra – AIRTEL - Delhi
None of these

5). If ‘Samsung’ is related to ‘Kolkata’, ‘HTC’ is related to ‘Hyderabad’, then


which following is ‘Motorola’ related to?
Mumbai
Chennai
Delhi
Bangalore
Patna

PUZZLE-96
Directions (6-10): Study the following information carefully and answer the given
questions.
Eight persons from different countries such as Austria, Belgium, Canada,
Denmark, Finland, Germany, Iceland and Mauritius eating 8 different fruits such
as Apple, Orange, Banana, Strawberry, Pears, Grapes ,Pineapple and Papaya .
And each of them having a watch in the brand names of Fastrack, Citizen, Titan
and Timex

The person who is from Belgium eating Papaya as a fruit. The person who is eating
Pineapple having Timex watch. The third floor stayed person eating Orange and having
a watch in the brand name of Fastrack. Banana is eaten by the person who is having
Citizen watch. Minimum of three persons stayed below of Mauritius country
man. And the person who belongs to Denmark not stayed in the odd numbered floor.
The watch’s brand name Citizen and Timex having same number of country-men.
Canada country man stayed two floor below from the person who is belongs to Belgium.
The top floor is not occupied by the person from Iceland. The Titan owned person
eating Apple. The persons from Denmark and Belgium having same watch brand and
both of them not having Citizen watch and also three persons stayed in between them.
The persons who are eating Strawberry fruit and Orange are having same watch brand
with the one who is eating Pears. The Titan owned person stayed in fourth floor. There
are four persons stayed in between of Iceland and Denmark. The lowest floor is
occupied by the Finland country man. Only three persons having one kind of watch
brand name and the person from Iceland who is eating Pears is one of them. The
country man from Germany and the person who eating Pears are not stayed in the even
numbered floor. Only one person having Titan watch. The Austrian who likes to eat
Grapes stayed in even numbered floor and is having same watch brand name with the
person , who is eating Banana also stayed in first floor. Timex watch is owned by the
person who is from Denmark. The person who eating Papaya stayed above the floor of
the person who is eating Pineapple. The Germany who is eating Orange having same
watch brand with the country man from Mauritius

6). Pears fruit eat by which of the following country person?


Canada
Belgium
Denmark
Austria
Iceland

7). Which of the following person sits exactly between who are eating apple
and papaya?
The one who eats Orange fruit
The one who eats Pineapple fruit
The one who eats Grapes fruit
The one who eats Strawberry fruit
The one who eats Banana fruit

8). Who lives in the sixth floor?


The one who is from Belgium
The one who is from Denmark
The one who is from Iceland
The one who is from Austria
The one who is from Mauritius

9). How many persons have Fastrack watches?


One
Two
Three
Four
Five

10). Which of the following combinations is true?


2 – Banana – Timex - Finland
4 - Apple - Titan - Canada
8 – Apple – Citizen - Finland
7 - Orange – Fastrack - Austria
None of these
Answer Key:
Direction (01 to 05):

b) 2. d) 3. e) 4. c) 5. a)

Direction (06 to 10):

6. e) 7. d) 8. a) 9. c) 10. b)
PUZZLE-97
Directions (1-5): Study the following information carefully and answer the given
questions.

Twelve friends are sitting in two parallel rows of chairs containing six people each,
in such a way that there is equal distance between adjacent persons. In row 1:
Dravid, Laxman, Sachin, Ganguly, Sehwag and Kumble are seated and all of them
are facing south. In row 2: Rohit, Yuvraj, Raina, Dhoni, Kohli and Dhawan are
seated and all of them are facing north. Each of them likes different month such as,
January, February, March, April, May, June, July, August, September, October,
November and December but not necessarily in the same order. Each of them like
different countries: USA, China, Australia, New Zealand, South Africa, West Indies,
England, Bangladesh, India, Sri Lanka, Pakistan and Zimbabwe. In the given
seating arrangement, each member seated in a row faces another member of the
other row.

Sehwag, whose favourite country is West Indies sits third to the left of the person
who likes February and not sitting at the extreme end. Laxman, who likes April,
does not sit at an extreme end of the row. If Sehwag sits at an extreme end then
Laxman doesn’t sit to the immediate right of the one who likes February. Raina and
Rohit face the persons whose favourite countries are West Indies and South Africa
respectively. There are two persons sitting on the right side of Yuvraj, who sits
second to the right of the person, whose favourite country is Sri Lanka. Yuvraj, who
likes May, faces the immediate neighbour of Dravid. Dravid likes March. Rohit sits
with the persons who like May and December. Kumble faces the person who likes
November month and Zimbabwe country but Kumble does not like February. The
one who likes December month doesn’t sit with the one who likes November month.
The person, who likes October month, sits second to the right of the person who
likes June. Dhoni does not like November and faces the person, who likes January.
The person who likes India is an immediate neighbour of the person who likes
August. Sachin’s favourite country is New Zealand and he doesn’t sit with Dravid.
Raina does not like China country. The person, whose favourite country is USA,
faces the person, who sits second to the right of the person, whose favourite
country is South Africa. The person whose favourite country is Bangladesh does not
sit in row 2 and faces the person who likes Pakistan. Kohli faces the person who
likes England. Sachin doesn’t face the person who likes June month. The one who
likes July sits in Row 2.
The one who likes England sits third to the right of ______________
Dravid
Laxman
Sehwag
Kumble
Sachin

Which of the following month likes by Rohit?


November
February
March
June
August

Which of the following combinations is true?


Kumble – January - Australia
Sehwag – June – West Indies
Raina – October – South Africa
Laxman – April - Bangladesh
None of these

Who likes the Sri Lanka country?


a) Kohli
Rohit
Dhawan
Raina
Dhoni

Which of the following persons sits at the extreme ends of one row?
Dhoni and Rohit
Kohli and Raina
Ganguly and Kumble
Dravid and Sehwag
Dhoni and Dhawan

PUZZLE-98
Directions (6-10): Study the following information carefully and answer the given
questions.

In Infosys company the staffs Vijay Kumar, Mithran, Sathya, Akilan, Sahana,
Gayathri, Hafiza, Varsha are working and they come to the company by different
way of rivers such as Ganges River, Yamuna River, Brahmaputra River, Mahanadi
River, Godavari River, Narmada River, Krishna River, Kaveri River not in the
respective order. And they are selected for special seminar. They planned to take
seminar individually with other staffs in different Cities such as Mumbai,
Bangalore, Kolkata, Chennai, Hyderabad, Delhi, Ahmadabad, Kochi in a week
starting from Sunday to Saturday.
Vijay Kumar going to take seminar before Mithran and after Varsha who did not come
by the way of Brahmaputra River

The staff who is going to take seminar in Kolkata, come to company by the way
of Narmada River.

Sathya come to company by the way of Yamuna River and he did not take seminar
on week’s starting day.
The staff who take seminar in Kochi, attend seminar in Delhi and Hyderabad in
the previous day.

Varsha likes to take seminar in Mumbai and she did not come by Krishna River.

Only two persons giving their seminar on same day due to other commitments

The person who is coming to company by the way of Godavari River likes to
take seminar on Thursday.

Mithran likes to take seminar in Ahmadabad and the day in between the persons
who take seminar in Chennai and Kochi.

Hafiza come to company by the way of Krishna River and she did not take
seminar on Saturday.

The persons who come by the way of Mahanadi River and Yamuna River likes
to take seminar on same day.

The person who takes seminar in Ahmadabad come by the way of Ganges River and
attend Sahana’s seminar who come to company by Godavari River in the next day.

Vijay Kumar who did not come by the way of Kaveri River taking seminar on the next
day of Akilan’s seminar. Akilan who making arrangement with another staff for
taking seminar on same day and he did not take seminar in Delhi.
Who takes the seminar on Friday?
The one who takes seminar in Bangalore
Hafiza
Both a and b
The one who takes seminar in Ahmadabad
The one who come to the company by the way of Ganges River

Who come to the company by the way of Narmada River?


Varsha
Akilan
Hafiza
Gayathri
e) Sahana

Two seminars held on which of the following day?


Thursday
Tuesday
Saturday
Friday
Monday

Akilan comes to the company by the way of which river?


Mahanadi River
Ganges River
Godavari River
Narmada River
Yamuna River

Which of the following combinations is true?


Varsha - Krishna River – Mumbai - Thursday
Mithran - Ganges River – Ahmadabad - Wednesday
Vijay Kumar - Brahmaputra River – Chennai - Wednesday
Gayathri - Kaveri River – Kolkata - Saturday
None of these

Answer Key:
Direction (01-05):
c 2) d 3) d 4) a 5) e

Direction (06-10):

6) c 7) d 8) e 9) a 10) b

PUZZLE-99
Directions (1-5): Study the following information carefully and answer the
given questions.
There are nine people Udit, Emaan, Jagan, Mandeep, Sankar, Murali, Akbar,
Nirmal and Rasul who sits around a circular table. Some are facing towards and some
are facing away from the centre. They like different airports among Kurnool Airport,
Rajkot Airport, Shimla Airport, Dhanbad Airport and Gwalior airport. Not more than 2
people like the same airports. The following information is known about them. Jagan
and Akbar form the only pair that is sitting together and like the same airports. They
are facing in opposite directions to each other. The one who likes
Shimla Airport sits third to the left of Emaan but is neither Jagan nor Murali. Udit sits
second to the left of Murali, who likes Rajkot Airport. The one who likes Kurnool
Airport is an immediate neighbour of Nirmal and Sankar but none of them sits
adjacent to Jagan or Akbar. Udit, Rasul and Nirmal face in same direction as that of
Sankar. Rasul sits third to the right of Mandeep, who is facing away from centre and
likes Dhanbad Airport. Sankar sits second to the left of Jagan and likes Rajkot
Airport. The people who like Rajkot Airport face towards the centre. Udit neither
likes Shimla Airport or Dhanbad Airport.

Which of the following person likes Kurnool Airport?


Akbar and Udit
Nirmal and Emaan
Mandeep and Rasul
Sankar and Jagan
Udit and Rasul

How many persons facing outside the circle?


Two
Five
Three
Six
Four

Which of the following airport like by Nirmal?


Shimla Airport
Kurnool Airport
Rajkot Airport
Gwalior Airport
None of these

4. Who sits second to the right of Emaan?


Nirmal
Akbar
Jagan
Rasul
Sankar

Which of the following combinations is true?


Nirmal– Facing outside –Shimla Airport
Sankar– Facing centre– Kurnool Airport
Udit– Facing centre–Rajkot Airport
Jagan – Facing outside – Gwalior Airport
None of these

PUZZLE-100
Directions (6-10): Study the following information carefully and answer the given
questions.
Twelve countries have sent one representative Raheem, Bibek, Jackson, Dylan,
Milton, Abdul, Mohamed, Patel, Irfan, Sundar, Salman and Abbas each for an
OPEC Summit. They meet in a rectangular summit hall where four representatives
occupy the four corners and two representatives each occupy places on each of the
four sides. All of them face the center of the hall. The following information is also
given:
Abdul, from Nigeria, sits second to the left of Mohamed, from Kuwait.
Abbas, the representative from Saudi Arabia is diagonally opposite to Abdul,
who sits second to the right of Dylan.
Dylan, the representative from Iran occupies the third place to the left of Bibek,
who is the representative from Angola.
Raheem, from Algeria, faces Mohamed while Irfan sits between Patel and Dylan.
The representatives from Ecuador and Qatar are diagonally opposite to each other
and Jackson from Ecuador is fourth to the left of Raheem.
Sundar, from Venezuela, is sixth to the right of Salman, who represents UAE.
Milton sits third to the right of Bibek. There are representatives from countries in
Iraq and Libya as well.

Patel likes which of the following countries?


Libya
Iraq
Either Iraq or Libya
Nigeria
Iran

How many persons it between Irfan and Bibek? (Counted from the left of Irfan)
Seven
Four
Six
Five
Three

Who sits third to the right of Mohamed?


Sundar
Dylan
Salman
Irfan
Either Sundar or Salman

Irfan is a representative of which of the following country?


Algeria
Qatar
UAE
Venezuela
Kuwait
Who sits third to the right of Raheem?
Abdul
Milton
Irfan
Dylan
None of these

Answer Key:
Direction (01-05):

e) 2. c) 3. a) 4. b) 5. d)

Direction (06-10):
6. c) 7. a) 8. e) 9. b) 10. d)

PUZZLE-101

Directions (1-5): Study the following information carefully and answer the
given questions.

Eight friends- Darshana, Hasina, Tanika, Bhavani, Vidya, Iniya, Akhila and Usha
visit different countries viz. Australia, Japan, Germany and Brazil are sitting around a
circular table facing the centre of the table. Each country is being visited by two people
only, but not necessarily in the same order. All these friends like different Dams i.e. -
Mitti dam, Salal Dam, Konar Dam, Bhadra Dam, Mettur Dam, Vaigai Dam, Tehri Dam
and Bargi Dam. No two people visiting the same country are sitting adjacent to each
other except those visiting Japan. The person who likes Bhadra Dam is sitting on the
immediate left of the person who like Bargi Dam. Vidya neither like Bargi Dam nor
Konar Dam. Usha likes Vaigai Dam and visits Germany and is sitting to the immediate
left of Hasina, who visits Japan. Hasina does not like Konar Dam. Iniya likes Mettur
Dam and visits Japan, who is sitting opposite to Akhila. Only
Darshana, who like Salal Dam, is sitting between who like Bhadra Dam and the
person who like Tehri Dam. Persons who visit Australia are sitting opposite to
each other. Each of the persons who visit Brazil is sitting adjacent to a person who
visits Australia. Bhavani does not like Bargi Dam.

1). Who sits second to the left of the person the one who likes Mitti Dam?
The one who likes Bhadra Dam
The one who likes Bargi Dam
Usha
Bhavani
The one who likes Mettur Dam

2). Who of the following person likes Tehri Dam?


Hasina
Akhila
Darshana
Bhavani
Vidya

3). Which of the following persons visiting Germany?


Darshana and Usha
Usha and Hasina
Akhila and Iniya
Bhavani and Vidya
Darshana and Tanika

4). Which of the following dam like by Tanika?


Tehri Dam
Mettur Dam
Bargi Dam
Vaigai Dam
e) Salal Dam

5). Which of the following combinations is true?


Usha – Germany – Bargi Dam
Bhavani – Brazil – Konar Dam
Akhila – Australia – Bhadra Dam
Hasina – Japan – Mitti Dam
None of these

PUZZLE-102
Directions (6-10): Study the following information carefully and answer the given
questions.

Hariram, Nikhil, Vikas, Gaurav, Lalit, Ramkumar, Chandrika, Jamuna, Mala, Pallavi,
Kavita and Dhanya are 12 persons sitting in two rows, among them Hariram, Vikas,
Gaurav, Ramkumar, Mala and Kavita are facing north while the remaining are
facing south. Each person faces exactly one person in the other row. Each one of
them likes a different festival from among Dussehra, Diwali, Mahashivaratri, Holi,
Ugadi, Onam, Lohri, Pongal, Bonalu, Navaratri, Brahmotsavam and Bohag Bihu
(not necessarily in the same order) and belongs to one state each from among
Maharashtra, Kerala, Tamil Nadu, Assam, Manipur, Bihar, Gujarat, Karnataka,
Rajasthan, Sikkim, Nagaland and Haryana (again not necessarily in the same
order).

The one who likes Ugadi is from Bihar and is facing Ramkumar. None of Nikhil,
Dhanya and Lalit likes Ugadi or Dussehra. The persons from Tamil Nadu and
Haryana are the immediate neighbours of Pallavi.
Nikhil is second to the right of Chandrika, who is from Gujarat. Nikhil like Lohri
festival and faces the person from Sikkim, who is third to the right of Gaurav.
Neither Hariram nor Gaurav likes Diwali festival or from Kerala.
Kavita, like the Mahashivaratri festival, sits second to the right of the person who
likes Navaratri festival, who faces the person from Haryana, None of them sits at
an extreme end.
The Diwali festival like by the person from Nagaland, sits as far as possible from
Gaurav.
Dhanya, likes the Onam festival, sits opposite the person who likes the
Brahmotsavam festival, who is an immediate neighbour of the person sitting at
an extreme end.
The Lohri and Holi festival liker’s are from Tamil Nadu and Maharashtra
respectively. The person who likes Onam festival and Holi festival face the
same direction.
Ramkumar, like the Pongal festival and from Assam, sits equidistant in the
same row from the Bonalu and the festival from Kerala.
The festival from Rajasthan is opposite the festival from Maharashtra and second
to the left of the festival from Assam. Hariram, who is not like the Brahmotsavam
festival, is third to the left of Mala; and neither of them like Diwali festival.
Jamuna likes the Bohag Bihu festival and is third to the right of person who
likes Onam festival from Manipur.

6). Which of the following person belongs to Sikkim?


Kavita
Mala
Pallavi
Hariram
Dhanya

7). Who sits second to the left of Pallavi?


Lalit
Nikhil
Jamuna
Chandrika
Dhanya

8). Which of the following persons sits at the extreme ends of one row?
Lalit and Nikhil
Hariram and Vikas
Gaurav and Vikas
Chandrika and Jamuna
Ramkumar and Gaurav

9). Which of the following combinations is true?


Ramkumar - Pongal – Tamil Nadu
Jamuna - Bohag Bihu - Haryana
Mala - Navaratri - Kerala
Dhanya – Lohri - Manipur
None of these

10). The one who belongs to Manipur sits third to the left of ______________
The one who belongs to Tamil Nadu
The one who belongs to Bihar
The one who likes Dussehra festival
The one who likes Lohri festival
The one who likes Holi festival

Answer Key:
1. b) 2. e) 3. a) 4. c) 5. d)

6. a) 7. e) 8. c) 9. b) 10. d)

PUZZLE-103
Directions (Q. 1-5): Study the following information carefully and answer the questions
given below:

There are five classes which have been scheduled for students in the days of the week
starting from Monday and ending on Friday. The subjects are Maths, Reasonin English,
Computer and GA. There are five teacherswhgo' belong to a different cities, viz Delhi,
Kolkata, Allahabad Bhopal and Patna. Among the five teachers only two are males.
The teacher who teaches Computer belongs to Delhi The one who teaches GA belongs
neither to Kolkata nor to Bhopal. A female belongs to Bhopal.
The one who belongs to Patna is a female and she teaches on Tuesday. The
Reasoning classes are scheduled on Wednesday. The males can take classes
alternately but not on Monday. Neither Maths nor Computer can be scheduled on
Monday. English classes are scheduled immediately after Reasoning. The one who
teaches Reasoning does not belong to Bhopal.
1). The teacher of which subject belongs to Kolkata?
Reasoning
Maths
Computer
Can't be determined
None of these

2). Male members take the classes on which' of the following days?
Wednesday & Monday
Thursday & Friday
Thursday
Wednesday & Friday
None of these

3). WhO among the following takes the class on Monday?


The one who belongs to Patna
The one who belongs to Allahabad
The one who teaches Reasoning
The one who teaches English
None of these

4). Who among the following takes class of Computer?


The one who takes class on Friday
The one who belongs to Bhopal
The one who takes class on Wednesday
The one who belongs to Patna
e) None of these

5). Which of the following combinations is true?


Female — Wednesday — Maths — Kolkata
Female — Thursday — English — Delhi
Male Friday Computer — Delhi
Male — Thursday — English — Bhopal
None of these

PUZZLE-104
Directions (6-11): Study the following information carefully and answer the questions
given below:
Eight friends P, Q, R, S, T, U, V and W are sitting 'around a circular table, but not facing
the centre. Each of them has a different rank in final-semester exam, viz 1 st to 8th, but
not necessarily in the same order.

The one who got 1st rank sits on the immediate left of the one who got 8th rank. V and
T are not immediate neighbours. V, who does not have 5th rank, sits on the immediate
right of W. P is second to the right of the one whose rank is 7th. S's rank is neither 6th
nor 7th. There are two persons sitting between R and the one whose rank is 6th. There
is only one person between those whose ranks are 6th and 8th. T and Q are immediate
neighbours. The one whose rank is 5th sits second to the right of U. W's rank is neither
1st nor 2nd but he is an immediate neighbour of the one whose rank is 5th. The one
whose rank is 1st cannot sit with the person whose rank is 4th. P and R, whose rank is
3rd, can never sit together. There are two persons between the persons whose ranks

are 7th and 5th.


6). What is the position of W with respect to the person whose rank is 6th?
Third to the right
Immediate left
Fourth to the left
Third to the left
From the right
7). How many persons are there between R and Q?
One
Two
Three
None
None of these

8). Who among the following sits second to the left of the one who sits opposite the
person whose rank is 2nd?
1)T
2) Q
3) U
4) V
5) S

9). Who among the following sits third to the right of S?


The one whose rank is 5th
The one whose rank is 6th
The one whose rank is 7th
The one whose rank is 1st
None of these

10). What is the position of P with respect to V?


Second to the left
Third to the right
Fifth to the right
Can't be determined
Third to the left

11). Which of the following statements is/are true?


a) S's rank is 4th.
R is second to the right of the one whose rank is 6th.
The one whose rank is 8th sits opposite V.
None is true
The one whose rank is 7th sits second to the right of P.

Solution(1-5):

SUBJECT DAY CITY GENDER


Maths Tuesday Patna Female
Reasoning Wednesday Kolkata Male
English Thursday Bhopal Female
Computer Friday Delhi Male
GA Monday Allahabad Female

1) a 2) d 3) b 4) a 5) c

Solution ( 6-11):

6) d 7) c 8) a 9) c 10) b 11) c
PUZZLE-105
Directions (1-5): Study the following information carefully and answer the given
questions.

There are seven boys, Sukumar, Gokul, Nagesh, Basha, Vinayak, Manoj and
Rajesh who participated in a game competition which started on Monday and ended
on Sunday. In the first round of the competition, each of them played different
games, Cricket, Football, Hockey, Tennis, Boxing, Badminton and Volleyball, but
not necessarily in the same order. They like different colours, Red, Blue, Green,
Yellow, Orange, White and Pink, but not necessarily in the same order

Gokul did not play on the day either immediately before or immediately after the
game of Basha, who does not like either Orange or Red or Yellow colour. Two games
were held between the game of Rajesh and Manoj, neither of whom played on
Monday. There was one game between the game of Basha and Nagesh. But
Nagesh’s game did not happen either on Monday or on Wednesday. Nagesh likes
Blue colour and played Cricket. The one, who played Hockey on the last day of
competition, likes Green colour. Vinayak played immediately after Nagesh and he
likes Red colour. Sukumar does not like Orange colour and played Tennis. Basha did
not play either Volleyball or Badminton. The one who played Boxing was scheduled
immediately after the game of Cricket. Rajesh, who likes White, played on the fourth
day of the competition but played neither Cricket nor Football.

1). Manoj likes which of the following colour and plays which of the following game?
Blue - Cricket
Green - Hockey
Yellow - Hockey
Pink - Tennis
White - Boxing

2). Who among the following plays Football game?


a) Gokul
Nagesh
Sukumar
Basha
Vinayak

3). Which of the following games held on Tuesday?


Cricket
Hockey
Boxing
Basha
Tennis

4). The person who plays Hockey like which of the following colour?
Green
Yellow
Blue
White
Orange

5). Which of the following combinations is true?


Tuesday – Sukumar – Orange - Tennis
Wednesday - Basha – Pink - Cricket
Saturday – Vinayak – Red - Boxing
Friday – Nagesh – Blue - Football
None of these
PUZZLE-106
Directions (6-10): Study the following information carefully and answer the given
questions.
M, N, O, P, Q, R and S lives in a seven storey building. The lower-most floor
is numbered 1; floor above it is numbered 2 and so on. There are four males and
three females among them.
Miss S lives in floor no. 2.
No female lives above the floor in which Miss N lives.
P lives in an odd numbered floor and he doesn't live either on the top-most or
lower-most floor.
Q has to use most no. of stairs to reach to his flat.
More than anyone else in the building, None of the female lives at the lower-most
or top-most floor.
R lives just a floor below his sister O's floor.
No two male or female live immediately above or below each other.

Who among the following lives on floor number 5?


R
Q
N
S
P

Which of the following floor number O Lives?


Two
Six
Five
Four
Three
If ‘Q’ is related to ‘Floor number 4’, ‘O’ is related to ‘Floor number 1’, then
which following is ‘P’ related to?
Floor number 7
Floor number 2
Floor number 5
Floor number 4
Floor number 6

Who among the following exactly sitting between S and P?


O and R
Q and N
O and N
Q and R
M and N

Which of the following combinations is true?


7 - Female - O
3 - Male - M
6 - Female - N
5 - Male - Q
None of these
Answer Key:
Direction (01-05):
Days Person Game Colour
Monday Gokul Orange Badminton/Volleyball
Tuesday Sukumar Yellow Tennis
Wednesday Basha Pink Football
Thursday Rajesh White Volleyball/Badminton
Friday Nagesh Blue Cricket
Saturday Vinayak Red Boxing
Sunday Manoj Green Hockey
b) 2. d) 3. e) 4. a) 5. c)
Direction (06-10):
Floor Gender Person
7 Male Q
6 Female N
5 Male P
4 Female O
3 Male R
2 Female S
1 Male M

6. e) 7. d) 8. b) 9. a) 10. c)

PUZZLE-107
Directions (1-5): Study the following information carefully and answer the
given questions.
Eight friends - Ajith, Vijay, Surya, Vikram, Siva, Jeeva, Madhavan and
Karthi likes to work in different banks viz, Syndicate Bank, Allahabad Bank, Bank
of India, Canara Bank, Dena Bank, Indian Bank, Bank of Baroda, and Vijaya Bank,
not necessarily in the same order. Each of them also owns a mobile from among
Samsung, HTC, Microsoft, Oppo, Vivo, Micromax, Lenovo and Apple, not
necessarily in the same order. It is also known that:

Ajith likes Allahabad Bank and does not own the HTC while Vikram does not work in
Canara Bank and owns the Oppo. Siva neither works in Canara Bank nor in Indian
Bank but owns either the Vivo or Microsoft. The HTC is not owned by Surya or Jeeva
while Karthi owns the Apple. The person who works in Indian Bank owns the Micromax
but is not Surya. The person who works in Canara Bank owns neither the Lenovo nor
the Apple. Indian Bank is not the bank of which Madhavan works, who owns the
Lenovo. Surya does not work in Bank of Baroda and does not own the
Samsung. The person who owns the HTC works neither work in Canara Bank nor in
Bank of India and the person who works in Vijaya Bank owns neither the Lenovo
nor the HTC. Vikram works in Dena Bank while Madhavan works neither in Bank of
India nor in Bank of Baroda.

1). Who among the following owns the HTC mobile?


Madhavan
Siva
Surya
Vijay
None of these

2). Who works in Vijaya Bank?


Karthi
Jeeva
Siva
Either Karthi or Siva
None of these

3). The person works in Bank of Baroda owns which of the following phones?
HTC
Oppo
Microsoft
Apple
Lenovo

4). Which of the following combinations is true?


Ajith - Syndicate Bank - Samsung
Jeeva - Indian Bank - HTC
Vikram - Dena Bank - Oppo
Madhavan - Allahabad Bank - Lenovo
e) None of these

5). If ‘Vijay’ is related to ‘Oppo’, ‘Vikram’ is related to ‘Micromax’, then which


following is ‘Jeeva’ related to?
Samsung
Microsoft
HTC
Lenovo
Apple

PUZZLE-108
Directions (6-10): Study the following information carefully and answer the
given questions.
Six friends Priya, Jaya, Richa, Deepa, Suvetha and Harini participated in the
cultural fest held in their college. Each of them participates in different dances:
Bharatanatyam, Kathak, Kathakali, Kuchipudi, Odissi and Mohiniyattam, not in
same order starting from Monday upto Sunday. There is a holiday in between when
no event is scheduled.

Holiday is not on Monday or Saturday. Deepa performs on Sunday and Bharatanatyam


is scheduled on Wednesday. There is two days gap between Holiday and Suvetha's
performance. Priya performs after Suvetha and Jaya performs before Holiday. There is
two days gap between Kuchipudi and Kathakali competitions and Kathakali is not
performed on Monday. Mohiniyattam is scheduled on the next day of Richa's
performance. Deepa is not a performing Kuchipudi. Odissi is not done by Suvetha But it
is scheduled before Kathak Competition. Kuchipudi is scheduled before
holiday.Suvetha and Harini did not perform on Monday.

6). The one who perform on Saturday perform which of the following dance?
Odissi
Kathak
Mohiniyattam
Bharatanatyam
Kuchipudi

7). Which of the following person performs on Tuesday?


Richa
Priya
Suvetha
Jaya
Harini

8). Which of the following days Kuchipudi dance performs?


Monday
Saturday
Tuesday
Wednesday
Thursday

9). Which of the following dance perform by Richa?


Odissi
Mohiniyattam
Kathakali
Kuchipudi
Bharatanatyam

10). Which of the following combinations is true?


Tuesday - Deepa - Mohiniyattam
Monday - Harini - Odissi
Sunday - Priya - Kathakali
Saturday - Harini - Kathak
None of these
Answer Key:

Person Banks Mobiles


Ajith Allahabad Bank Samsung
Vijay Bank of Baroda HTC
Surya Canara Bank Microsoft / Vivo
Vikram Dena Bank Oppo
Siva Bank of India / Vijaya Bank Vivo / Microsoft
Jeeva Indian Bank Micromax
Madhavan Syndicate Bank Lenovo
Karthi Vijaya Bank / Bank of India Apple

1. d) 2. d) 3. a) 4. c) 5. e)

Day Person Event


Monday Richa Odissi
Tuesday Suvetha Mohiniyattam
Wednesday Jaya / Priya Bharatanatyam
Thursday Priya / Jaya Kuchipudi
Friday - -
Saturday Harini Kathak
Sunday Deepa Kathakali

6. b) 7. c) 8. e) 9. a) 10. d)
PUZZLE-109
Directions (1-5): Study the following information carefully and answer the given
questions:
Seven Summits ASEAN Summit, BRICS Summit, SAARC Summit, NAM
Summit, BIMSTEC Summit, SKOCH Summit, and OPEC Summit were scheduled
to be held in China, Japan, Russia, Iceland, India, Germany and Canada, on one
day in a week starting from Monday and ending on Sunday.

BIMSTEC Summit was held in Iceland on Friday. Only one summit was held
between BIMSTEC Summit and BRICS. SKOCH Summit was held immediately
after SAARC but immediately before OPEC. SKOCH Summit was not held in India.
The summit held on Monday was held in Russia. Only one summit was held
between OPEC Summit and the summit held in China. The SAARC was not held in
China. Summit in Japan was held immediately before summit in India. NAM Summit
was not held on Monday. Summit in Canada was not held after summit in China.

1). On which of the following days was the SAARC Summit held?
Monday
Tuesday
Wednesday
Thursday
Friday

2). If Russia is related to Japan and OPEC is related to NAM in a certain way, then to
which of the following would India be related to, following the same pattern?
Germany
Canada
China
Russia
India
3). Which of the following summits was held on Monday?
OPEC
SKOCH
SAARC
ASEAN
None of these

4). In which of the following countries was OPEC Summit held?


Canada
China
Russia
Germany
India

5). How many summits were held between ASEAN Summit and NAM Summit?
Four
One
Two
Three
Five

PUZZLE-110
Directions (6-10): Study the following information carefully and answer the
given questions:
Eight friends Praveen, Salim, David, Arafath, Yadav, Naresh, Arnold and
Johnson sitting around a rectangular table, four of them facing inside the table
and four of them facing outside.
David is going to Sikkim and sitting third to the right of Yadav. David is facing outside
the table. Arnold who is going to Assam is sitting third to the right of Naresh and facing
outside the table. Salim is facing inside and sitting second right of Arnold.
Person going to Punjab sitting second left of person who is going to Odisha. Person
going to Karnataka sitting second left of person who is going to Manipur. Naresh is
going to Bihar and sitting fourth left of Praveen who is facing opposite direction of
Salim. Yadav is going to Haryana and facing inside the table. Naresh is facing
outside the table. The persons going to Punjab and Manipur are facing each other.
Arafath is not going to either Manipur or Odisha and facing same direction of
Yadav. Johnson is facing opposite direction of Arnold, is sitting to the immediate left
of Yadav.

6). What is the position of Yadav with respect to David?


Second to the left
Fourth to the left
Third to the left
Third to the right
None of these

7). Who of the following is sitting third to the left of Yadav?


Praveen
Salim
David
Arnold
Arafath

8). Praveen is facing which direction?


Inside the table
Outside the table
Inside or outside
Cannot be determined
None of these

9). Which of the following pair is correct?


David – Sikkim
Arafath – Punjab
Praveen – Karnataka
Yadav – Haryana
All of these

10). Four of the following five are same in a certain way so form a group. Which
of the following does not belong to that group?
Yadav, Naresh
Arafath, David
David, Salim
Praveen, Salim
Johnson, Arnold

Answer Key:
Direction (01 to 05):

Days Country Subject of summit


Monday Russia ASEAN
Tuesday Canada SAARC
Wednesday Japan SKOCH
Thursday India OPEC
Friday Iceland BIMSTEC
Saturday China NAM
Sunday Germany BRICS
b) 2. c) 3. d) 4. e) 5. a)
Direction (06 to 10):
6. d) 7. a) 8. b) 9. e) 10. c)

PUZZLE-111
Directions (Q. 1-5): Study the following information carefully and answer the
given questions.

Shah Rukh Khan, Aamir Khan, Akshay Kumar, Hrithik Roshan, Ranbir Kapoor, Ajay
Devgan, Abhishek Bachchan and Irrfan Khan are eight fighters sitting around a
circular table. Four of them are facing towards the centre and four of them are facing
away from the centre. All of them like a different National parks, viz. Jim Corbett
National Park, Ranthambore National Park, Kaziranga National Park, Periyar
National Park, Bandipur National Park, Manas National Park, Madhav National Park
and Satpura National Park, but not necessarily in the same order. Ranbir Kapoor
faces towards the centre and likes Jim Corbett National Park. Both the immediate
neighbours of Ranbir Kapoor face away from the centre and like Kaziranga National
Park or Periyar National Park. Hrithik Roshan faces away from the centre. Both the
immediate neighbours of Hrithik Roshan do not face away from the centre. Ranbir
Kapoor sits third to the right of Ajay Devgan, who likes Bandipur National Park and
faces away from the centre. Akshay Kumar sits third to the left of Ajay Devgan. The
one who likes Kaziranga National Park sits opposits of Ajay Devgan. The one who
likes Ranthambore National Park is not the immediate neighbour of Ajay Devgan
and faces away from the centre. Shah Rukh Khan sits second to the left of Akshay
Kumar and does not like Madhav National Park or Satpura National Park. The one
who likes Madhav National Park sits between Irrfan Khan and Ajay Devgan. Aamir
Khan faces away from the centre and does not like Ranthambore National Park.

1). Who like the Manas National Park?


Ranbir Kapoor
Aamir Khan
ShahRukh Khan
Irrfan Khan
Akshay Kumar

2). How many persons sit between Hrithik Roshan and Ranbir Kapoor, count
from the right of Hrithik Roshan?
Five
Four
Three
Two
None of these

3). The only person who is sitting between Ranbir Kapoor and Abhishek
Bachchan, likes which of the following parks?
Kaziranga National Park
Bandipur National Park
Manas National Park
Jim Corbett National Park
Periyar National Park

4). Who among the following sits second to the right of Aamir Khan?
a) Hrithik Roshan
Irrfan Khan
Akshay Kumar
Ajay Devgan
None of these

5). Which of the following is correctly matched?


Akshay Kumar - Facing Centre - Bandipur National Park
Abhishek Bachchan - Facing Outside - Periyar National Park
Ajay Devgan - Facing Centre - Jim Corbett National Park
Shah Rukh Khan – Facing Centre - Manas National Park
None of these

PUZZLE-112
Direction (6-10): Study the given information carefully to answer the given questions:

Ten Persons belongs to different states i.e. Assam, Bihar, Gujarat, Haryana,
Maharashtra, Karnataka, Kerala, Nagaland, Sikkim and Manipur are sitting in
two parallel rows containing five persons each, in such a way that there is an
equal distance between adjacent persons. In row-1 Rajesh, Vidya, Surya,
Prakash and Deepika are seated (not necessarily in the same order) and all of
them are facing south. In row-2 Sanjay, Dinesh, Balaram, Janani and Kaushik
are seated (not necessarily in the same order) and all of them are facing north.
Therefore in the given seating arrangement each member seated in row faces
another member of the other row. All of them have relation with each other.

Only one person sits between Surya’s wife and Rajesh’s brother. Only two persons sit
between Surya’s father and Janani’s father. Dinesh’s father sits at one of the end.
Neither Vidya nor Janani’s grandfather faces Janani. Dinesh belong to Haryana and
Vidya is neither belongs to Assam nor Manipur. Janani’s husband, who belongs to
Karnataka, sits third to the right of Janani’s uncle. Sanjay sits second to the left of
Janani’s husband. Rajesh has two sons. The person facing Janani’s brother, who
belongs to Gujarat, sits immediate right of Rajesh’s daughter-in-law. Deepika is the
grandmother of Sanjay and Janani. Prakash is the brother-in-law of Deepika. Surya
is the father of Sanjay and Husband of Vidya. Sanjay’s grandfather, who belongs
to Assam, is not an immediate neighbour of Vidya. Kaushik is the husband of
Janani and brother of the one, who belongs to Maharashtra. Janani belongs to
Kerala and Deepika belongs to Sikkim. Surya neither belongs to Nagaland nor
Bihar. Balaram belongs to Maharashtra. Prakash does not belong to Bihar.

6). Who is second to the left of the person who belongs to Sikkim?
The One who belongs to Manipur
Rajesh
Deepika
The One who belongs to Bihar
The One who belongs to Nagaland

7). How is Sanjay related to Rajesh?


Son
Brother
Son in law
Grand son
None of these

8). What is the position of Kaushik with respect of Kaushik’s wife?


Third to the right
Immediate left
Second to the right
Immediate right
Second to the left

9). Balaram’s Brother belongs to, which of the following state?


Gujarat
Manipur
Assam
Bihar
Karnataka

10). Which of the following statement is true?


Rajesh belongs to Assam and he is the father of Prakash
Balaram belongs to Manipur and he is the brother of Kaushik
Vidya belongs to Bihar and she is the wife of Surya
Sanjay belongs to Kerala and he is the brother of Janani
None of these

Explanation:
Directions (Q. 1-5):

1). Answer: c)
2). Answer: b)
3). Answer: e)
4). Answer: a)
5). Answer: d)
Directions (Q. 6-10):

Family tree:

6). Answer: a)
7). Answer: d)
8). Answer: b)
9). Answer: e)
10). Answer: c)

PUZZLE-113
Directions (Q. 1-5): Study the following information carefully and answer the
given questions.
Ten Persons are sitting in two parallel rows of six seats each. One seat is vacant in
each row. Mahima, Sathya, Karthick, Vimal and Priyanka are sitting in row-1 facing
south. Lokesh, Murugan, Lalitha, Meena and Begum are facing north. Each likes a
different brand of Tooth pastes i.e. Colgate, Pepsodent, Sensodyne, Oral- B, Dant
Kanti, Vicco, Amway, Himalaya, Close Up and Meswak. Meena sits third to the right of
Lalitha and likes Oral- B. Only two people sit between Murugan and the vacant seat.
Murugan does not like Dant Kanti or Sensodyne Tooth paste. Priyanka is not
an immediate neighbour of Karthick. Sathya likes Meswak. The one who likes
Sensodyne Tooth paste faces the one who likes Himalaya. The one who likes
Sensodyne sits opposite to the one who sits third right of the person who sits
opposite to Meena. Karthick is not an immediate neighbour of Vimal. Begum, who
likes neither Dant Kanti nor Vicco, does not face the vacant seat. Neither Meena nor
Lalitha sits at any of the extreme ends of the row. Vimal faces Lalitha. Vacant seats
are not opposite to each other. Two seats are there between Karthick and Sathya,
who sits third right of the one who likes Amway. The one who likes Close Up Tooth
paste faces the one who likes Oral- B. The persons who like the Colgate and
Himalaya are adjacent to each other. Vacant seat of row – 1 is not an immediate
neighbour of Vimal. Murugan sits at one of the extreme ends of the row. Lalitha
does not like Colgate and Himalaya. Vacant seat of row-1 does not face Meena who
doesn’t sit at any of the extreme ends of the row.

1). Which of the following brand Begum likes?


Meswak
Oral- B
Amway
Pepsodent
Close Up

2). Who among the following likes Vicco?


Begum
Meena
Murugan
Vimal
None of these

3). How many people will sit between Lokesh and Murugan?
a) One
Two
Three
Four
More than four

4). Which of the following is correctly matched?


Vimal – Meswak
Lokesh – Dant Kanti
Begum – Amway
Murugan – Close Up
Lalitha – Oral- B

5). Four among the following form a group in a certain way. Which of the
following does not belong to Group?
Priyanka, Meswak
Lokesh, Sensodyne
Murugan, Dant Kanti
Sathya, Close Up
Begum, Oral- B

PUZZLE-114
Directions (Q. 6-10): Study the following information carefully and answer the given
questions.
Abhinav plays Pokemon Go. He caught 7 different Pokemons on 7 different days of
the week from Monday to Sunday. One of the Pokemons is Ratata. These Pokemon
had 2 characteristics: Type and Combat Power (CP). One of the Pokemons is
Poison type.
The following information is known:

• The combat powers of the 7 Pokemons are 25, 31, 38, 46, 49, 52 and 60.
Charmander is not the Ground type Pokemon.
The Pokemon caught on Monday has CP 46 while Spearow was the last Pokemon
to be caught.
Squirtle, an Electric Pokemon, was caught on Wednesday.
There is only one Pokemon whose CP is lower than that of Pidgey.
The number of Pokemons caught before Pikachu is the same as the number of
Pokemons caught after Charmander. Pikachu was caught before Charmander.
The Pokemon that has the highest CP was not caught on Saturday.
A Rock type Pokemon whose CP is 25 was caught on Friday and a Water type
Pokemon whose CP is 52 was caught on Tuesday.
Bulbasaur is a Fire type Pokemon which was not caught on either Thursday or
Saturday and Spearow is a Fighting type Pokemon.
The Pokemon caught on Sunday has CP less than 40.

6). What is the CP of Charmander?


38
46
49
60
52

7). Which is the rock type Pokemon?


Ratata
Pikachu
Charmander
Bulbasaur
Pidgey

8). What is the difference between the CP of Squirtle and that of the fighting type
Pokemon?
14
24
27
22
11

9). Arrange the Pokemons in the order they were caught:


Bulbasaur, Pikachu, Ratata, Pidgey, Charmander
Bulbasaur, Pikachu, Pidgey, Ratata, Charmander
Pikachu, Bulbasaur, Pidgey, Ratata, Charmander
Bulbasaur, Pidgey, Pikachu, Ratata, Charmander
Pikachu, Bulbasaur, Pidgey, Charmander, Ratata

10). Bulbasaur is related to 'Electric' in some manner and Ratata is related to


'Fighting' in that manner. Which Pokemon is related to 'Rock' in the same manner?
Pikachu
Pidgey
Charmander
Spearow
Squirtle

Explanation With Answers Key:


Directions (Q. 1-5):
1). Answer: d)
2). Answer: c)
3). Answer: a)
4). Answer: b)
5). Answer: e)

Directions (Q. 6-10):

6). Answer: c)
7). Answer: a)
8). Answer: d)
9). Answer: b)
10). Answer: e)

PUZZLE-115
Directions (Q. 1-5): Study the following information and answer the questions
given below.
In a conference 8 people Rekha, Salma, Ganesh, Mishra, Gibbs, Sahana, Pooja
and Aarya from different countries Australia, Germany, China, Japan, Russia,
Switzerland, Poland and Brazil not necessarily in same order sitting around a
rectangular table. 3 persons are sitting on each longer side and each on the smaller
sides. Mishra is sitting second to the right of the person who is from Brazil. Gibbs is
sitting third to the left of the person who is from Switzerland. Sahana and Pooja are
sitting opposite each other. Ganesh is sitting diagonally opposite the person from
Poland. Salma is sitting opposite the person who is from Australia. The person
from Japan is sitting second to the right of the person from Russia and second to
the left of Rekha, who is not sitting near the person who is from Poland. Pooja is
sitting on the smaller side and to the right of the person who is from Poland.
Persons from Russia and Japan are not on the same side of the table. The person
from Japan is sitting third to the right of the person from Brazil, who is not sitting
diagonally opposite the person from Germany. The person from Japan is third to
the left of Ganesh. Sahana sits second to the left of Aarya. The person from
Germany sits opposite to the person from Russia.
1). Which of the following combinations is correctly matched?
Aarya –Brazil
Mishra – Russia
Pooja – Japan
Rekha– China
None of these
2). Who is sitting third to the right of the person from Switzerland?
Gibbs
Person from Russia
Rekha
Person from Poland
None of these
3). From which country does Rekha belong?
China
Australia
Russia
Brazil
e) Germany
4). Who is definitely sitting diagonally opposite to Mishra?
Rekha
Ganesh
Person from Germany
Person from China
None of these
5). According to the sitting arrangement what will come in place of question mark?
Mishra : Japan ::Ganesh : ?
Switzerland
Russia
China
Brazil
None of these

PUZZLE-116
Directions (Q. 6-10): Study the following information and answer the questions
given below.
Eight friends having different Sims as Airtel, Vodafone, Aircel, Docomo,
Idea, BSNL, JIO and Reliance are sitting around a circular table, not necessarily in
the same order. Some of them are facing outside and some of them are facing
towards the centre. Each of them has birthday in different year.

Only two people sit between the one who uses Idea and the one who uses Aircel. The
friends who use Vodafone and Airtel are not neighbours. The one who uses Airtel is
sitting just right of one who uses Idea. The one who uses JIO sits second to the right of
one who uses Aircel. The person who uses Idea faces outside and he is the oldest guy
having birthdays in 1975. Only one person sits in between friends who use Reliance
and Airtel. The one liking Vodafone is 8 years older than the one liking BSNL. The one
who uses Reliance sits third to the left of one who uses Vodafone and he is the
youngest among all having birthday in 1989. The person
who uses BSNL faces towards the direction opposite to the one who uses JIO, but
same to the one who uses Aircel. Person liking BSNL is not a neighbour of either
who uses Airtel or JIO. The one who uses Airtel was born in 1983 and the one who
uses JIO is 4 years younger than him. Immediate neighbours of one who uses
Docomo face in the same direction. The one who uses Airtel faces towards the
centre. The one who uses Vodafone and the one who uses Aircel face the
direction just opposite to the one who uses Docomo. The person facing the one
who uses Airtel was born in 1979. The immediate neighbours of the one who uses
Airtel have faces opposite to each other and they have the age gap of 6 years.
6). Who uses Docomo sim born on which year?
1983
1981
1975
1979
None of these
7). How many persons facing outside?
Four
Three
Two
Six
None of these
8). Which of the following is correctly matched?
Airtel – Facing Centre - 1981
Jio – Facing Outside - 1983
Aircel – Facing Centre - 1987
Idea – Facing Outside - 1975
None of these
9). Who is sitting second to the left of person who born in 1975?
a) One who uses Docomo sim
One who born in 1989
One who uses JIO sim
One who born in 1981
One who uses Vodafone sim
10). How many persons sit between who uses the Airtel sim and who uses BSNL
sim, count from the right of one who uses Airtel sim?
1
3
5
4
6
Answer Key:
Direction (01-05):

c) 2. d) 3. e) 4. b) 5. a)
Direction (06-10):
6. b) 7. a) 8. d) 9. e) 10. d)

PUZZLE-117

Directions (Q. 1-5): Study the following information carefully and answer the
given questions.

There are seven boys Jadeja, Saha, Pujara, Kohli, Vijay, Rahul and Rahane

who participated in a game competition which started on Monday and ended on

Sunday. In the first round of the competition, each of them played different games,

Cricket, Tennis, Football, Hockey, Badminton, Volleyball and Basketball, but not

necessarily in the same order. They like different Social Networks, Facebook, Twitter,

Hike, Whatsapp, Telegram, Instagram and Google, but not necessarily in the same

order. Saha did not play on the day either immediately before or immediately after the

game of Kohli, who does not like either Telegram or Facebook or Whatsapp Social

Network. Two games were held between the game of Rahane and Rahul, neither of

them played on Monday. There was one game between the games of Kohli and

Pujara. But Pujara’s game did not happen either on Monday or


on Wednesday. Pujara likes Twitter Social Network and played Cricket. The one,
who played Football on the last day of competition, likes Hike Social Network. Vijay
played immediately after Pujara and he likes Facebook Social Network. Jadeja
does not like Telegram Social Network and played Hockey. Kohli did not play either
Basketball or Volleyball. The one who played Badminton was scheduled
immediately after the game of Cricket. Rahane, who likes Instagram, played on the
fourth day of the competition but played neither Cricket nor Tennis.

1). Which game organised on Tuesday?

Tennis
Badminton
Football

Hockey
Cricket
2). Rahul like which of the following social Network?
Hike
Facebook
Twitter
Whatsapp
None of these

3). Which of the following statement is true?


Monday – Saha – Volleyball - Telegram
Friday – Pujara - Cricket - Twitter

Thursday – Rahane - Basketball - Instagram


Wednesday – Jadeja – Hockey - Facebook
Sunday – Rahul – Football - Google
4). Which of the following day Cricket game is organized?
Sunday

Saturday
Wednesday
Monday
Friday
5). Which of the following boy like the Instagram social Network?
Vijay

Pujara
Rahane
Saha

Kohli

PUZZLE-118

Directions (Q. 6-10): Study the following information carefully and answer the
given questions.

There are nine people A, B, C, D, E, F, G, H and I in a family. It is a three


generation family. They are sitting in a row facing in North and South directions
not necessarily in the same order. The following information is known about them.
C who is A’s father sits fourth to the right of B who is the only daughter of E and
they both face in the opposite directions. D sits third to the right of F who is B’s
grandmother and one of them sits adjacent to I. D is unmarried and faces in a
direction similar to that of G. E who is the sister of D sits at one of the ends with a
female. C’s neighbours are females and face in opposite directions. F’s spouse C
faces in the North direction. I who is the wife of G sits at the middle facing in South
direction. A, who is male, is the only sibling of G sits exactly between D and B’s
cousin who is a male. The people sitting at ends face in North direction. A faces in

same direction as that of F. C sits adjacent to I.

6). Who among the following sits at one of the ends?


F’s son
H’s mother
A’s daughter
G’s son
None of these

7). How is D related to A?


Wife
Sister in law

Brother in law
Husband
None of these
8). How many males are there in the family?
2

3
1
5
4
9). C and D face in which direction in the respective order?
North, South

South, North
North, North
South, North
None of these
10). Who sits third to the right of H’s mother?

A
F
G
H
Answer Key:
Direction (01 to 05):

d) 2. a) 3. b) 4. e) 5. c)

Direction (06 - 10):


6. d) 7. b) 8. e) 9. c) 10. b)

PUZZLE-119
Directions (1 -6): Study the following information carefully and answer the questions
given below:

Isha, Swati, Ravi, Mohan, Ruchi, Raj, Sohan and Rani are eight friends travelling by
three different models of cars, viz Alto, Scorpio and WaganR. Each model of car is
used by at least one female. The cars go to three places- Dwarka, Varanasi and
Agra. There are three days of travel – Monday, Tuesday and Wednesday. Each
place is visited by at least on female. On each day at least one female travels. Isha
and Mohan travel to Dwarka on the same day by the same model of car. Raj travels
on Monday in Alto, but not to Agra. A person travelling to Agra uses Scorpio on
Monday. WagonR goes only to Varanasi and only on Tuesday. Sohan travels to
Varanasi in Alto on the day before the day on which Isha travels. Ruchi travels to
Agra but she likes to travel in Alto. No male member travels in WagonR. Isha travels
on the same day on which Swati travels. Ravi travels to Varanasi on Wednesday in
Scorpio. Rani does not travel on Tuesday. No person travels to Dwarka on Monday.
Mohan travels by the same model of car by which Sohan travels. Ruchi and Ravi
travel on the same day.

1). Ruchi travel on which of the following day?


Monday
Wednesday
Either Monday or Tuesday
Tuesday
None of these
2). Who among the following travel to Varanasi on Monday?
Raj and Rani
Sohan and Raj
Swati and Sohan
Rani and Swati
None of these
3). How many persons are travelling to Dwarka?
One
Two
Three
Can’t be determined
None of these
4). Swati travels in which of the following cars?
Scorpio
Alto
WagonR
Either Alto or Scorpio
None of these
5). Which of the following combinations is / are true?
Isha – WagonR – Dwarka – Tuesday
Ravi – Scorpio – Varanasi – Wednesday
Ruchi – Alto – Dwarka – Monday
Rani – Alto – Varanasi – Wednesday
None of these
6). Which of the following statements is / are true?
Sohan goes to Varanasi on Monday
Mohan travels in Scorpio on Tuesday
Raj Travels in Alto to Agra
Swati goes to Varanasi on Wednesday
None of these

PUZZLE-120
Directions (Q. 7-11): Study the following information and answer the following
questions:
A, B, C, D, E, G and I are seven friends who study in three different standards,
namely 5th, 6th and 7th, such that not less than two friends study in the same
standard. Each friend has a different favourite subject, namely History, Civics,
English, Marathi, Hindi, Maths and Economics also but not necessarily in the same
order.

A likes Maths and studies in the 5th standard with only one friend who likes Marathi.
I studies with two other friends. Both the friends who study with I like languages
(here languages include only Hindi, Marathi and English). D studies in the 6th
standard with only one person and does not like Civics. E studies with only one
friend. The one who likes History does not study in the 5th or 6th standard. E does
not like languages. C does not like English, Hindi or Civics.

7). Which combination represents E’s favourite subject and the standard in which he
studies?
Civics and 7th
Economics and 5th
Civics and 6th

History and 7th


Economics and 7th
8). Which of the following is I’s favourite subject?
History
Civics
Marathi
Either English or Marathi
Either English or Hindi

9). Who among the following studies in the 7th standard?


G
C
E
D
Either D or B

10). Which of the following combinations is definitely correct?


I and Hindi
G and English
C and Marathi
B and Hindi
E and Economics

11). Which of the following subjects does G like?


Either Maths or Marathi
Either Hindi or English
Either Hindi or Civics
Either Hindi or Marathi
Either Civics or Economics

Answers:
1). b) 2). b) 3). b) 4). c) 5). b) 6). a) 7). c) 8). a) 9). a) 10). c) 11). b)
Directions(1-6):

Name Car Place Day


Isha Alto Dwarka Tuesday
Swati WagonR Varanasi Tuesday
Ravi Scorpio Varanasi Wednesday
Mohan Alto Dwarka Tuesday
Ruchi Alto Agra Wednesday
Raj Alto Varanasi Monday
Sohan Alto Varanasi Monday
Rani Scorpio Agra Monday

Questions (7-11):

Person Standard Subject

A 5th Maths

C 5th Marathi

D 6th Economics

E 6th Civics

I 7th History

B 7th Hindi/English

G 7th Hindi/English
PUZZLE-121
Directions (Q. 1-5): Study the following information carefully and answer the
given questions.
Six department exams – Computer, Maths, Science, Commerce, Economics
and Engineering were written by three males – Gokul, Sukumar and Magesh and
three females – Deepa, Priya and Divya. Three of these department exams were
held in Chennai and the rest in Bangalore. Each of these exams was organized by
different universities– Manipal, SRM, VIT, Amity, IIT and IIS (may not be in the
same order). Each exam was written by a male and a female.

None of the students wrote in a city twice and no two department exams were
written by the same pair of male and female. The females who written Computer
and Maths did not write the exam organized by IIT and IIS. The male student was
same for the exams – Computer and Maths. Sukumar written Science but did not
write the exam organized by IIT or Manipal. Priya did not write any exam with Gokul,
but written Commerce with Magesh. Deepa wrote an exam with Magesh. IIS
organized the exam in Chennai and Amity organized the exam in Bangalore which
was written by Gokul and VIT organized Maths. Economics was organized by
Manipal. IIT and SRM organized the exams in the same city. Maths and Science
exam were written by the same female. The Engineering exam was not organized
by IIT or Amity and it was not written by Deepa.

1). Who wrote the Maths department exam?


Magesh and Divya
Sukumar and Priya
Gokul and Deepa
Gokul and Divya
None of these

2). Which of the following exams organized in Chennai?


a) Maths, Economics and Computer
Maths, Commerce and Engineering
Maths, Economics and Engineering
Science, Economics and Engineering
Maths, Economics and Science

3). Science exam is conducted by which university?


SRM
VIT
IIT
IIS
None of these

4). Which of the following exam written by Magesh and Priya?


Science
Commerce
Computer
Engineering
Maths

5). Which of the following statement is true?


Computer – Gokul – Deepa – Amity - Chennai
Commerce - Magesh - Priya – VIT - Bangalore
Maths - Gokul – Divya – VIT - Bangalore
Engineering - Sukumar – Deepa - IIS - Chennai
Economics – Magesh – Deepa – Manipal - Chennai
Direction (01-05):

PUZZLE-122

Directions (Q. 1-5): Study the following information carefully and answer the
given questions.

M, N, O, P, Q, R, S and T are eight family members sitting around a circular


table in a restaurant. All of them are facing the centre. Three of the family members
are non- vegetarian and others are vegetarian. All the eight family members are
professionals. They are Banker, Engineer, Accountants, Police, Lawyer, Scientist,
Professor and Teacher, but not necessarily in the same order. There are two
married couples, three brothers, two daughters, one granddaughter and one
grandson in the group. Two of the female members in the group M and T order for
the same food. The food ordered in the restaurant are Khichdi, Kufta, Chapatti,
Butter chicken, Vegetable Biryani, Tandoori Chicken and Puri.

S, a Banker, is the head of the family, and is immediate neighbour of T and P. The
granddaughter R is sitting third to the left of S. The grandmother of R is sitting on the
immediate left of her. R is vegetarian. Q is sitting second to the right of his father S. He
is a Lawyer, and orders for Puri. Only S’s daughter is sitting exactly between M and Q.
She orders for Vegetable Biryani and she is a Scientist. T is an Accountant
and N is a Police. The son of T is sitting on the immediate left of his father. His
father is an Engineer. O is a Scientist and is sitting on the immediate left of her
mother. She orders for Vegetable Biryani. P, the husband of the Accountants,
ordered for Tandoori Chicken and his mother orders for Butter chicken. The one
who is a Police sits second to the right of M, who is an immediate neighbour of the
Scientist and the Professor. The Professor does not like Khichdi and the Banker
does not like Chapatti or Kufta.

1). Who among the following working as Professor?

R
P
O
M
None of these
2). Who among the following is ordering Khichidi food?
R
S

M
N
None of these
3). Which of the following is correctly matched?
N – Police - Chappatti
Q – Teacher - Puri

P – Engineer - Tandoori Chicken


O – Scientist – Tandoori Chicken
None of these
4). The only person who is sitting between R and O is working on which of the
following professions?

Engineer
Police
Banker
Accountant
Teacher
5). The person who ordered Butter chicken is working as which of the following

professions?
Teacher
Police

Accountant
Both Teacher and Accountant
Professor

PUZZLE-123

Directions (Q. 6-10): Study the following information carefully and answer the given
questions.

Ten persons Daniel, Rajesh, Michael, Yusuf, Anand, Balaji, Lokesh, Kannan,
Jaffar and Praveen are sitting in two rows with five persons in each row. The
persons in row one are facing south and the persons in row two are facing north.
Each person in row one faces a person from the other row. All of them have a
laptop of different companies, viz Acer, HP, Asus, Dell, LG, Toshiba, Samsung,
Lenovo, Apple and Microsoft, but not necessarily in the same order.

The persons who like LG and Toshiba sit opposite each other. Balaji sits opposite
to Daniel, who likes Acer. The one who likes HP sits opposite the one who likes
Lenovo. Praveen is not facing north but sits third to the left of Lokesh, who likes HP.

There is only one person between Rajesh and Michael. Anand sits at one of the ends of
the row and likes Toshiba. The one who likes Lenovo is on the immediate right of

Yusuf, who does not like Microsoft. The persons who like Asus and Dell respectively

are not facing north. Michael likes Samsung. The one who likes Dell sits opposite the

one who is second to the right of Rajesh. Jaffar does not like Microsoft. Anand sits
opposite the one who sits second to the left of the one who likes Asus.

Who among the following is the immediate neighbour of the one who have Acer
laptop?
The one who have Apple laptop
Michael

Jaffar
The one who have HP laptop
None of these
Who have Apple laptop?
Anand

Rajesh
Yusuf
Balaji
Lokesh
Who sits at the right end in North facing Row ?
Kannan

Praveen
Lokesh
Anand
Yusuf
9). Who sits opposite to Rajesh?
The one who have Acer laptop

The one who have Asus laptop


The one who have Apple laptop
The one who have LG laptop
The one who have HP laptop
10). Who sits opposite to the one who sits second to the left of Anand?
The one who have Asus laptop

The one who have Dell laptop


The one who have Apple laptop
The one who have HP laptop

None of these
Answer Key:
Direction (01-05):
a) 2. b) 3. c) 4. e) 5. d)

Direction (06-10):

Kannan
Facing South Lokesh (HP) Balaji (Asus) Jaffar (Dell) Praveen (LG)
(Microsoft)

Rajesh Michael Anand


Facing North Yusuf (Apple) Daniel (Acer)
(Lenovo) (Samsung) (Toshiba)

6. b) 7. c) 8. d) 9. e) 10. a)

PUZZLE-124
Directions (Q. 1-5): Study the following information carefully to answer the
given questions:
Eight People – Haroon, Amir, Prem, Devan, Ramesh, Siva, Mohan and Irfan
live in eight different floors of building (but not necessarily in the same order). The
lowermost floor of the building is numbered one, the one above that is numbered
two, and so on till the topmost floor is numbered eight. Each one of them also owns
a different brands of watch, namely Fastrack, Titan, Casio, Timex, Sonata, Citizen,
Giardano and Maxima (but not necessarily in the same order).
Only one person lives between Amir and the one who owns Sonata. Siva lives an
odd numbered floor above the floor numbered four. Only three people live between
Devan and the one who owns Citizen. Prem lives on one of the odd numbered
floors above the one who owns Citizen. The one who owns Giardano lives
immediately above Mohan, Mohan owns neither Citizen nor Fastrack. Ramesh does
not own Giardano. Only three people live between Mohan and Haroon. The one
who owns Maxima lives immediately above the one who owns Titan, but not on the
topmost floor. Only one person lives between Siva and the one who owns Casio.
The number of people living above Siva is same as the number of people living
between Siva and Devan. Only two people live between Prem and the one who
owns Fastrack.

1). Who amongst the following stays on the top–most floor?


Who owns Timex Watch
Mohan
Amir
Both a and c
None of these

2). How many floors are there between the floor on which Siva stays and the floor
who owns Titan watch?
a) None b) Three c) Two d) One e) More than three
3). Ramesh owns which of the following Watches?
Sonata
Citizen
Fastrack
Titan
Casio
4). On which of the following floor does Irfan stay?
a) 3rd b) 5th c) 7th d) 2nd e) 1st
5). Which of the following statement is true as per the given information?
Irfan stays on a floor immediately below the floor on which Mohan stays
Prem stays on a floor immediately above the floor on who owns Citizen Watch
Haroon stays on sixth floor and owns a Timex watch
There are two persons live between Ramesh and Amir
None is true

PUZZLE-125
Directions (Q. 6-10): Study the following information carefully to answer the given
questions:

Eight friends Vikram, Ashok, Varun, Gautam, Charan, Basha, Yashwant and
Edward live on eight different floors of a building but not necessarily in the same
order. The lowermost floor of the building is numbered 1 and the topmost floor of
the building is numbered 8. Each of them likes different flavours of fruit juices viz,
Mango Juice, Orange Juice, Apple Juice, Grapes Juice, Lemon Juice, Banana
Juice, Pomegranate Juice and Papaya Juice but not necessarily in the same order.
The one who likes Orange Juice lives on an even-numbered floor but not on the
topmost floor. Only one person lives between Basha and the one who likes Lemon
Juice. Only two persons live between Basha and the one who likes Orange Juice.
Neither Charan nor Varun lives on the first floor. Only one person live between
Varun and the one who likes Apple Juice. Vikram lives just above Basha. Only two
persons live between Charan and Vikram. The one who likes Lemon Juice does not
live on floor number one, Ashok lives on an even-numbered floor and just above
Varun. The one who likes Grapes Juice lives on an even numbered floor and lives
just above the person who likes Papaya Juice. Varun does not like Lemon Juice or
Papaya Juice. Only two persons live between the one who likes Banana Juice and
the one who likes Mango Juice. Gautam does not like Pomegranate Juice. The one
who likes Banana Juice does not live on an odd-numbered floor. Edward lives just
below the one who likes Papaya Juice.
6). On which of the following floor does Ashok stay?
Third
Fourth
Fifth
Second
Seventh
7). How many persons are there between who likes apple juice and who
likes grapes juice?
Six
One
Three
Four
Five
8). Who likes the Papaya Juice?
One who lives in floor number 5
Basha
Gautam
One who lives in floor number 3
Vikram
9). Which of the following is wrong as per the given information?
Charan stays on a floor immediately above the floor on who likes Orange Juice
Only one person stay between who likes Mango Juice and Gautam
Basha lives on seventh floor and likes Banana Juice
More than two persons living between Yashwant and Edward
All statements are true
10). Who amongst the following stays on the fifth floor?
Edward
Varun
Ashok
d) Basha
e) None of these

Answer with Explanation:

Direction (01 to 05):


Floor Person Watch
8 Amir Timex
7 Prem Casio
6 Haroon Sonata
5 Siva Citizen
4 Ramesh Fastrack
3 Irfan Giardano
2 Mohan Maxima
1 Devan Titan
1. d) 2. b) 3. c) 4.a) 5. e)

Direction (01 to 06):


Floor Person Mobiles
8 Vikram Grapes Juice
7 Basha Papaya Juice
6 Edward Banana Juice
5 Charan Lemon Juice
4 Ashok Orange Juice
3 Varun Mango Juice
2 Yashwant Pomegranate Juice
1 Gautam Apple Juice
6. b) 7. a) 8. b) 9. c) 10. e)
PUZZLE-126
Directions (Q. 1-5): Study the following information carefully to answer the given
questions:

There are six persons- Aarthi, Shreya, Kavya, Vidya, Diya and Madhu living in a
triple floor building with six flats. The floors are Ground, Middle and Top, each
having two flats. Those who have four or more books occupy the top floor and go
by different Vehicles. The one who is in engineering department in a company goes
by Cycle and she is not Madhu while the one who is in Computer department of a
company goes by Bus, Both of them occupied same floor. Shreya who is in Science
department has three books and not stayed in the Top floor. Kavya, an unmarried
woman, does not have any book, occupies the middle floor. Diya and Madhu have
Round purses while the rest have Square purses. Vidya, a middle floor occupant,
goes by Car. One of the two persons having two books goes by Bike and is in Agri
department in a company also occupied in ground floor. Two persons, of whom one
is in Commerce department, do not go by Vehicles.
1). Who among the following occupies the top floor?
Aarthi
Diya
Kavya
Vidya
None of these
2). By which Vehicle does Kavya go?
Bus
Cycle
Bus or Cycle
No Vehicle
None of the above
3). What is the least number of books owned by all the persons in the
entire building?
14
16
15
17
Cannot be determined
4). In which department does Vidya work in?
Commerce
Engineering
Agri
Computer
Cannot be determined
5). How many books does Aarthi have?
None
One
Two
Three
None of these

PUZZLE-127
Directions (Q. 6-10): Study the following information carefully to answer the
given questions:
There are seven people M, N, O, P, Q, R and S. They like different Mobiles, Banks and
were born in different States and months. They all were born in the same year. One of
the Mobiles is HTC. One of the States is Maharashtra. One of the Banks is Bank of
India. One of the persons was born in the month of September. The following
information is known about them. R was born either in Tamil Nadu or in Goa. The one
who was born in the month of May likes Indian Bank and Apple mobile and is either O
or S. P is not the one who likes Syndicate Bank. The one who
likes Canara Bank likes LeEco mobile. The one who was born in Goa is the
youngest one and is either N or S. The one who likes Dena Bank was born in
Rajasthan but is not N. Q was born in the month of February. P was born in Bihar
in the month of July and likes Micromax mobile. The one who likes Vijaya Bank
was born in November but is not the youngest person. N likes Motorola mobile. The
one who likes Canara Bank was born in Punjab but that person is not O who likes
Nokia mobile. The youngest person likes Syndicate Bank and he is not S.The one
who was born in Kerala likes Samsung mobile but is not Q. R is the second
youngest person. The one who likes UCO Bank is one month younger than Q.
6). Which of the following Banks does P like?
Vijaya Bank
Bank of India
Indian Bank
UCO Bank
Dena Bank
7). When was O born?
March
May
September
November
Cannot be determined
8). Who likes HTC mobile?
R
S
M
N
O
9). Which of the following statements is correct?
a) E likes Bank of India Bank
M likes Apple mobile
S was born in the month of September
N was born in Tamil Nadu
None of these
10). Who was born in Maharashtra?
Q
M
N
S
O

Explanation with Answer key:

Directions (Q. 1-5):

1). Answer: b)
2). Answer: d)
3). Answer: c)
4). Answer: e)
5). Answer: c)
Directions (Q. 6-10):

6). Answer: b)
7). Answer: c)
8). Answer: a)
9). Answer: e)
10). Answer: d)

PUZZLE-128
Directions (Q. 1-5): Study the following information carefully to answer the
given questions:
Udhaya, Mahesh, Arun, Fasal, Raja, Gopi and Johnson are seven people live on seven
different floors of a building but not necessarily in the same order. The lower most floor
of the building is numbered 1, the one above that is numbered 2 and so on till the
topmost floor is numbered 7. Each one of them earn different amount per month. i.e.
24000, 18000, 12000, 27000, 33000, 39000 and 45000. (But not necessarily in the
same order). The one who earn Rs.24000 lives immediately above the one who earns
Rs.45000. Only one person lives between Mahesh and Raja. Mahesh lives on one of
the floors above Raja. Neither Arun nor Udhaya earns Rs.27000. Raja does not earn
Rs. 12000. Udhaya lives on an odd numbered floor but not on the floor numbered
three. The one who earns Rs.33000 lives immediately above Udhaya. Only two people
live between Udhaya and the one who earns
Rs.12000. The one who earns Rs.18000 lives on one of the odd numbered floors
above Fasal. Only three people live between Arun and the one who earns 18000.
The one who earn Rs.12000 lives immediately above Arun. Gopi earns 12000.

1). Four among the following form a group in a certain way. Which of the
following does not belong to Group?
Johnson – 18000
Udhaya – 45000
Mahesh – 24000
Gopi – 45000
Raja – 12000

2). Which among the following is the salary of Mahesh?


Rs. 24000
Rs. 45000
Rs. 39000
Rs. 33000
None of these

3). Who among the following earns Rs. 39000?


Udhaya
Arun
Fasal
Gopi
None of these.

4). Which of the following combination is true as per the given arrangement?
Udhaya – 18000
Arun – 33000
Mahesh – 39000
Gopi – 24000
e) None of these.

5). Who among the following lives in floor no 3?


Udhaya
Arun
Fasal
Gopi
None of these.

PUZZLE-129
Directions (Q. 6-10):

A group of seven friends Chandru, Chahal, Charan, Chamu, Chandran, Charu and
Chadrasekar has four males and three females. Each of them has wear different
coloured T-shirts, viz Yellow, Orange, Red, White, Green, Blue and Grey, and each
of them belongs to different states –Maharashtra , Odisha, Tamilnadu, Uttar
Pradesh, Madhya Pradesh, Manipur and Karnataka but not necessarily in the same
order. None of the females wear Blue or Yellow. Charan wear Red and she belongs
to neither Uttar Pradesh nor Manipur. Chandru is from Manipur and wear neither
yellow nor blue. Chahal does not wear White. Chandran has Grey and from either
Odisha or Maharashtra . Charu has Yellow or Green and not belong to
Maharashtra. Chadrasekar has either Blue or Yellow T-shirt and from Tamilnadu.
The one who wears White belongs to Karnataka. The one who wears Orange either
from Manipur or Maharashtra . None of the female either from Manipur or Odisha.

6). Which of the following combinations is true?


Chandran -Grey-Odisha-Female
Chandru -Green-Odisha-Male
Chadrasekar -Blue-Maharashtra-Female
Charan -Red-Madhya Pradesh-Male
None of these
7). Which of the following T-shirt colour does Chamu have?
Blue
White
Orange
Red
None of these

8). Which of the following statements is true?


Chamu is from Maharashtra.
Charan is a male and possesses Red T-shirt.
Charu possesses Yellow and from Uttar Pradesh .
Chadrasekar is a female and possesses Blue.
All are true

9). Which of the following groups has only female members?


Charu,Chamu,Chahal
Chandran,Charan,Chadrasekar
Charan, Chamu,Chahal
Chamu,Chandru,Chahal
None of these

10). The one who has Yellow T-shirt is from which of the following states?
Odisha
Madhya Pradesh
Manipur
Uttar Pradesh
Cannot be determined
Directions (Q. 1-5):

1). Answer: d)
2). Answer: d)
3). Answer: b)
4). Answer: a)
5). Answer: c)

Directions (Q. 6-10):

6). Answer: e)
7). Answer: b)
8). Answer: c)
9). Answer: c)
10). Answer: d)

PUZZLE-130
Directions (Q.1 -5): Study the following information carefully and answer the
given questions.

Ten students namely viz Rakesh, Naveen, Kumar, John, Suresh, Divya,
Priya, Ramya, Anjali and Deepa from a banking coaching institute but not
necessarily in the same order have exam on five different days starting from
Monday to Friday of the same week. Students have exam at two different time slots,
i.e 09.30 A.M and 01.30 P.M. Anjali has a exam on Tuesday at 09.30 A.M. The
number of people who have exam between Priya and John is same as the number
of people who have exam between Kumar and Ramya. Naveen has a exam day
immediately before Anjali. Anjali does not have exam on any of the days before
Priya. There is one student who has exam at 09.30 A.M immediately before Deepa.
Ramya does not has exam at 01.30 P.M. John has an exam immediately after the
day of one who has exam on Monday. Divya does not have exam at 01.30 P.M.
John does not has exam on any one of the days after Suresh. Only three people
have exam between Priya and Suresh. Neither Suresh nor Priya does not have
exam on Friday. Only two people have exam between Divya and Deepa. Divya does
not have exam on any of the days after Ramya.

1). Which of the following is correctly matched?


John – Monday
Kumar – Tuesday
Rakesh – Friday
Priya – Tuesday
Suresh – Wednesday
2). How many persons have exam at 01.30 P.M between Priya and Kumar?
5
6
2
4
1

3). Who among the following have exam on Friday?


Anjali, Suresh
Ramya, Deepa
Naveen, Rakesh
John, Suresh
Suresh, Kumar

4). Who among the following person has exam at 01.30 P.M?
Kumar
Priya
Anjali
Divya
Suresh

5). Four among the following form a group in a certain way. Which of the
following does not belong to Group?
Priya – Tuesday
Anjali – Wednesday
Divya – Friday
Priya – Wednesday
Suresh – Thursday
Direction (01-05):
Students
Days
Morning shift Noon shift
Monday Priya Naveen
Tuesday Anjali John
Wednesday Suresh Kumar
Thursday Divya Rakesh
Friday Ramya Deepa

PUZZLE-131
Directions (Q.1 -5): Study the following information carefully and answer the given
questions.
9 friends Ajay, Bhisup, Civesh, Dinesh, Evesh, Fervesh, Gukan, Harish and Ismail
were seated around a circular table facing the centre. They were playing the game of
cards and everyone was holding a single card bearing a number from 2-10 not
necessarily in the same order. It was further known that:

Evesh and Bhisup had cards whose product was equal to the product of Dinesh and
Civesh 's cards.
Dinesh and Civesh had even numbered cards.
B had a number that was twice of Civesh 's card number. Bhisup and Civesh sit
together.
The person with the lowest card number was 2nd to the left of Harish
Dinesh sat 4th to the left of Civesh and the sum of their card numbers was equal
to the card number of Fervesh
Gukan got a smaller card number than Evesh
3 persons were seated between the persons having the card numbers 7 and 5 when
counted in a clockwise manner from 5
Ismail was seated 3rd to the left of Bhisup. Ismail had a card number greater
than Bhisup
Harish was to the immediate right of Fervesh.
1). What is the position of Ajay with respect to the one have card number 6?
Immediate left
Immediate right
Second to the right
Second to the left
Third to the right
2). How many persons had a card number greater than the card number of the one
who to the immediate right of Evesh?
None
One
Two
Three
More than three
3). What is he difference between Ajay and Dinesh’s card numbers?
1
2
3
4
5

4). In which of the following groups is the 3rd person seated exactly in between the
1st and the 2ndpersons?
Bhisup Civesh Ajay
Ajay Dinesh Evesh
Dinesh Harish Evesh
G Civesh Bhisup
Bhisup Gukan Farvesh
5). What card number does the person has who is to the immediate left of Ismail?
3
5
9
6
8

PUZZLE-132
Directions (Q.6-10): Study the following information carefully and answer the
given questions.
Twelve people are sitting in two parallel rows. In row 1, Avikshit, Avinash, Ayush,
Atulya, Avanish and Atmaja are sitting facing towards south. In row 2, Aadesh, Aabheer,
Aafreen, Aakash, Aashish and Abhilash are sitting facing towards north. They are sitting
in a manner that each person sitting in row 1 faces exactly one person sitting in row 2.
Each of these twelve persons except two has a different favorite colour.

Aabheer likes pink and is sitting opposite to the one who is second to the left of
Ayush who likes Grey.
Neither Aabheer nor Ayush sits at any of the extreme end.
Aakash and the one who likes Green sit at extreme end of the row, but they do not
face each other.
Only three people sit between Aafreen and the person who likes Blue.
Aafreen faces Avanish, who likes Red.
Atmaja sits second to the right of Atulya, who likes Brown.
Four people sitting at extreme end like white, green, blue and purple
color. Aadesh likes black and is sitting third to the left of Aafreen
Avinash does not face Aabheer while Aashish faces Atulya
Aakash is a neighbour of Aadesh while Abhilash is not
There are three people between the two who like green and grey
The one who likes yellow is sitting immediate left of the person who likes purple
6). Who is sitting second to the right of the one who faces Atulya?
Aadesh
Aabheer
Aafreen
Aakash
Aashish

7). Find the odd one out?


Atmaja
Avinash
Aakash
Abhilash
Atulya

8). Who is sitting between Atulya and the person who likes Red color?
Avikshit
Avinash
Ayush Atulya
Avanish
None

9). What is the favourite color of Aashish?


Blue
Black
White
Grey
None

10). Which of the following combination between person and color is incorrect?
a) Aadesh-Black
Atulya -Brown
Ayush -Grey
Avanish-Red
All are correct

Answer Key:
Solution(1-5):

Answer: e)
Answer: b)
Answer: a)
Answer: d)
Answer: a)

Solution(6-10):
Answer: c)
Answer: e)
Answer: a)
Answer: e)
Answer: e)

PUZZLE-133
Directions (1-5): Study the following information carefully and answer the
questions given below:
Farmer A of Nirman Nagar goes to trade fair with their family after completing their
daily work. There are nine members in the family after including farmer, i.e. A, B, C,
D, E, F, G, H and I. Each member eats different food items viz. Chocolate, Pizza,
Burger, Rasmalai, Samosa, Jalebi, Khulfi, Dosa and Idli. The relation of farmer with
the family members is defined as Father, Mother, Brother, Sister, Wife, Daughter,
Son, and Brother-in-law but not in the same order. All the family members are
enjoying circular ride, but not necessarily in the same order. Some of them are facing
towards centre while others are facing outward.
I is brother of A’s daughter. A’s brother is seated on the immediate right of the
brother of A’s sister and both face the same direction. I’s father eats Pizza. H has a
sister. The mother of A’s son is E and sits second to the left of the married son of A’s
father. F’s daughter-in-law eats Idli. A faces outward. F is a male. A’s father sits third
to the right A’s daughter. Either I or H sits third to the left of the mother of C, but
both I and H face the same direction. A’s brother and sister eats Burger and Chocolate
respectively. G is not father of H. In two generations, the members of the same
generation are sitting immediate next to each other and face the same direction. A’s
father does not eat Dosa and Khulfi. F’s grandson eats Rasmalai. B is a female and of
the same generation as of A. G sits on the immediate left of the granddaughter of A’s
father. The one, who is the wife of F likes Kulfi . A’s brother-in-law neither eats Dosa
nor Samosa. A’s father doesn’t sit immediate next to his children. G is facing the
centre and is second to the right of A. B sits second to the left of F. No three
members seated together can face the same direction.
Q1. How is B’s husband related to A’s father?
Brother
Father
Son
Grandson
Son-in-law
Q2. Who sits third to the right of A’s mother?
B
H’s brother-in-law
A’s brother-in-law
E’s husband
Both (b) and (c)

Q3. Who among the following sits exactly between A and his wife?
F
A’s sister
A’s daughter
D
None of these

Q4. Four of the following five are alike in a certain way and hence they form a
group. Which one of the following does not belong to that group?
C
F
H
I
A

Q5. Which of the following persons faces inside?


B, G, F and I.
A, G, D and I.
G, D, A’s father and G’s wife.
D, G, B and C.
None of these
PUZZLE-134
Directions(6-10): Answer the questions on the basis of the information given
below.
Six kids– Z, X, C, V, B and N- watch in different Cartoon serials on their TV viz.
Ducktails, Doremon, Shinchain, Baloo, Chota beem and Hanuman – and they are lives
on six different floors of a building, numbered 1 to 6. They like six different Cartoon
Characters viz. Dholu, Sizuka, Munni, Lui, Jiyan and Pumba, in no particular order.
Each Cartoon serial is cast on a different day from Monday to Saturday. X likes Jiyan
and Cartoon serial likes by him is Doremon and lives on the 6th floor of the building. B
and Z lives on even numbered floors who’s cartoon cast on Saturday and Monday. The
one who watch Ducktails is lives on the lowest floor and it is cast on Tuesday. Baloo
Cartoon serial watch by that kid who lives on the floor next to X and it is cast on
Thursday. The kid who watches Hanuman likes Pumba and his cartoon is cast on
Monday. N likes Lui and he lives on first floor. One of the kid who watches Cartoon
serials which cast on friday lives on even numbered floors. V lives two floors below C
and he likes Munni. B, who does not live immediately above or below of C, likes Dholu
and his serial cast on Saturday. The kid who likes Chota beem is lives on an even
numbered floor.

Q6. Who is watching Ducktails cartoon serial?


Z
N
V
X
C

Q7. Which Character does B like?


Pumba
Jiyan
Dholu
Lui
Sizuka

Q8. Who among the following watch that cartoon which is cast on Thursday?
V
N
Z
X
C
Q9. If V is related to C and B is related to Z, in the same way N is related to?
X
N
Z
V
C

Q10. Which of the following combination is correct?


V – Shinchain – Sizuka
C – Baloo – Munni
Z – Chota beem – Pumba
X – Doremon – Dholu
N – Ducktails – Lui

PUZZLE-135
Directions (11-15): Study the following information carefully and answer the
questions.
There are seven friends – S, P, M, A, D, K and C. All of them are working in Army and
they like different type of machine gun like Anti-aircraft machine gun(AA), machine
gun(MG), light Machine gun(LMG), medium machine gun (MMG), heavy machine
gun(HMG), general-purpose machine gun (GPMG), and squad automatic machine
gun(SAMG). They go to temple one by one in the same week starting from Monday but
not necessarily in the same order.
S goes to temple on the third day of the week and he does not like LMG and HMG.
Only one friend goes to temple between S and P, who likes AA. There are two friends
go to temple between the days on which P and M, who likes MG. The one who likes
SAMG goes to temple on the fourth day of the week. Only one friend goes to temple
between the A and one who likes SAMG. The one who likes GPMG goes to temple on
last day of the week. The one who likes HMG goes to temple neither immediately
above nor below the days on which S goes to temple. There are as many as friend go
to temple between the D and one who likes LMG and as between the D and C. K does
not go to the temple last day of the week.

Q11. Who among following goes to temple on Monday?


(a) P (b) D (c) S (d)M (e) K

Q12. S likes which of the following machine gun?


GPMG
SAMG
LMG (d)MMG
(e)None of these

Q13. S goes to temple on which of the following day?


Friday
Tuesday
Monday
Wednesday
Sunday

Q14. Who among following likes HMG?


K
M
C
A
P

Q15. K goes to temple on which of the following day?


Monday
Tuesday
Wednesday
Thursday
(e)None of these
PUZZLE-136
Directions (11-15): Study the following information carefully and answer the
questions.
There are seven students – M, N, O, P, Q, R and S – whose favourite subjects are-
Hindi, English, Math, Science, Physics, Chemistry and Biology but not necessarily in
the same order. They go to the library on different days of the week starting from
Monday. Also, they like different colours, viz- Red, Green, Black, White, Yellow,
Violet and Orange but not necessarily in the same order.
N, who likes Violet, goes to library fourth day of the week but his favourite subject is
neither Hindi nor Biology. Two students go to the library between the days on which N
and R go to the library and neither of them goes to library 1st day of the week. There
is one student goes the library between M and O. But O does not go either on 1st or
3rd day of the week. O likes Green colour and his favourite subject is Hindi. The one
whose favourite subject is Math goes on the last day of the week and like the Black
colour. P does not go to the library on the day either immediately before or
immediately after the day on which M goes to the library, who doesn’t like either
Yellow or Red or White colour.
S goes to the library immediately after O and he likes Red colour. Q does not like the
Yellow colour and his favourite subject is Chemistry. M’s favourite subject is neither
Physics nor Science. The one whose favourite subject is English goes to the library
immediately after the day when the student whose favourite subject Hindi goes.

Q11. Which of the following combinations is definitely false?


(a)O – Green – Hindi
(b)M – Orange –Biology
(c)Q – White –Chemistry
(d)R – Green – Hindi
(e)None of these
Q12.Who among the following goes to the library on the 5th day of the
week? (a)The student who likes Orange colour
(b)The student whose favourite subject is Math
(c)The student whose favourite subject is Hindi
(d)The student who likes White colour (e)None
of these

Q13.Whose favourite subject is Science?


(a)The student who likes Violet colour
(b)The student who likes Orange colour
(c)The student who likes Yellow colour
(d)Either the student who likes Yellow colour or the student who likes Violet colour
(e)None of these

Q14.If ‘N’ is related to ‘Green’ and ‘S’ is related to ‘Black’ then which of the
following is ‘M’ related to?
(a)Yellow
(b)Violet
(c)White
(d)Orange
(e)None of these

Q15.Whose favourite subject is Biology?


(a)The student who goes to library fourth day of the week
(b)The student who goes to library before Q
(c)The student who goes to library just after Q
(d)Can’t be determined
(e)None of these
PUZZLE-137
Directions (1-5): Study the following information given below and answer the
given questions.
Eight friends, viz A, B, C, D, E, F, G and H are sitting in a straight line but not
necessarily in the same order and all of them are facing north direction. Each of them
has belong to diierent cities, viz Varanasi, Patna, Agra, Delhi, Noida ,Puri, Puna and
Chandigarh but not necessarily in the same order. G sits third to the right of the
person who belongs to Noida. The person who belongs to Chandigarh sit second to the
right of G. A and E are immediate neighbours of each other. Neither A nor E belong to
Noida or Chandigarh. Neither A nor E is an immediate neighbour of G. H sits third to
the right of the person who belongs to Varanasi. Neither A nor E belongs to Varanasi.
H does not belong to Chandigarh. Only two people sit between E and the person who
belongs to Puna. The person who belongs to Patna sit on the immediate left of D. Only
one person sits between E and B. C sits second to left of one who belongs to Puna. E
does not belong to Agra. The one who belongs to Puri sits at extreme end of the line.

Q1. Who among following sits on the corner?


A,H
D,G
A,G
D,C
None of these

Q2.How many persons sits between E and H?


two
three
two
more than three
None of these

Q3. C belongs to which of the following city ?


Delhi
Noida
Varanasi
Puna
None of these

Q4. Who among following sits fourth to left of H?


(a) E (b) C (c) B (d) F(e) None of these
Q5. If E is related to Varanasi and B is related to Agra, in the same way F is related
to?
Puma
Agra
Patna
Chandigarh
More than three

PUZZLE-139
Directions(11-15): Read the following information carefully and answer the
questions which follow.
There are seven friends named P, Q, R, S, T, U, V from different cities Agra, Noida,
Patna, Delhi, Varanasi, Jaipur and Puri. They are wearing shoes of different brands
like Nike, Lotto, Lancer, Sparx, Puma, Adidas and Reebok. They like different colours
like Orange, Red, Yellow, Blue, Amber, Indigo, and Pink. T is wearing Sparx shoes and
likes Indigo colour. The one who is wearing Lancer shoes does not like Blue and Pink
colour. U does not wear Reebok and Lotto shoes and likes Blue colour. P does not like
pink or yellow colour. S is from Delhi wears Adidas shoes. Q is from Patna and does
not wear Nike shoes. V wears either Puma or Lotto shoes. Q does not wear Lotto
shoes. The one who likes yellow colour does not belong to either Noida or Delhi.
Person from Jaipur is like Indigo colour. The one who belongs to Varanasi likes Pink
colour and he is wearing Reebok shoes. The person wearing Nike shoes is belong to
either Puri or Jaipur. S likes Orange colour . The one who is wearing Lotto shoes likes
Amber colour. U does not belong to Puri.

Q11. U belongs to which of the following city?


Jaipur
Noida
Agra
Either (a) or (b)
Either(b) or (c)
Q12. P likes which of the following colour?
Red
Amber
Indigo
Blue
None of these

Q13. Who among following is wearing Lancer shoes?


U
R
S
Q
None of these

Q14. Who among following belongs to Puri?


P
R
Q
V
None of these

Q15. Which among the following combination is correct?


R- Pink- Varanasi
U- Yellow- Lancer
P- Indigo- Jaipur
V- Delhi- Lotto
None of these
PUZZLE-140
Directions (1-5): Study the following information carefully and answer the given
questions.
Ten persons are sitting in two parallel rows containing five persons in each. In row 1,
A, B, C, D and E are sitting and all of them are facing south. In row 2, P, Q, R, S and T
are sitting and all of them are facing north. In the given seating arrangement, each
member seated in a row faces another member of the other row but not necessarily in
the same order. All of them like different colours viz. Red, Yellow, Violet, Green,
Black, Brown, White, Pink, Blue, and Orange but not necessarily in the same order.
There are two persons sits between Q and P, who likes violet. Immediate neighbour Q
faces D, who likes blue. The one who likes red sits third to the left of E, one of them
sits to the extreme end of the row. B is not an immediate neighbour of E. There is
only one person sits between T and Q, who likes orange. The immediate neighbour of
E faces the person who likes pink colour. One of the immediate neighbour of E likes
yellow colour. A is not an immediate neighbour of D and he likes white colour. The
immediate neighbour of R faces the person who likes brown colour. The immediate
neighbour of E faces T, who likes green.

Q1. S faces which of the following person?


A
B
C
D
E

Q2. Who among following sits to the extreme end of the row ?
A,S
B,P
E,Q
D,R
A,Q

Q3. Who among following sits second to the right of P?


T
The one who likes orange colour
The one who likes violet colour
The one who likes black colour
Q
Q4. Who among following sits diagonally opposite to R?
C
The one who likes red colour.
The one who likes blue colour.
E
D

Q5. Who among following sits third to right of B?


A
E
C
D
None of these

PUZZLE-141
Directions (11-15): Study the following information carefully and answer the given
questions.
There are nine teachers A, B, C, D, E, F, G, H, and I of different subjects like Hindi,
English, Science, Sanskrit, Computer, Math, Art, Commerce and History but not
necessarily in the same order. They all are living on a different floor. The ground floor
is numbered one and so on up to top floor is numbered nine. Only three teachers live
between I and one who teaches History, both are living on odd numbered floor but
none of them is living either top floor or bottom floor. F and G do not like Science.
Only one teacher lives between I and A, who teaches commerce. There is only one
teacher lives between H and C, who teaches Computer. D teaches Hindi and lives on
the even numbered floor. There is only one teacher lives between the floor on which
History’s teacher and English teacher lives. B teaches Art and lives immediately below
on the floor on which A lives. E does not live immediately above or below on the floor
on which C lives. E does not live on first floor. There is as many as teacher lives
between E and one who teaches English and between G and one who teaches Sanskrit.
Only two teachers live between I and H, who teaches Math. The teacher who teaches
science lives on one of the floors above the floor on which teacher who teaches
Sanskrit lives.

Q11. F teaches which of the following subject?


History
Math
Computer
English
None of these

Q12. Who among following teaches Science?


A
D
E
I
B

Q13. Who among following lives on top floor?


B
E
F
G
A

Q14. Who among following lives between the floor on which I and E lives?
A
C
D
G
B

Q15. If A is related to Science and H is related to Computer, in the same way B is


related to ?
Sanskrit
Math
Hindi
History
Computer
PUZZLE-142
Directions (1-5): Study the information given below and answer the given
questions.
Nine people A, B, C, D, E, F, G, H and I stay in a building. The building has nine
floors and only one person stays on one floor. Each of them have a puppy whose
names are- Tiger, Rony, Panther, Rocky, Jumbo, Sheru, Gabru, Bullet and
Cheetah. Each person belongs to different cities, i.e. London, Bangkok, Paris,
Dubai, Istanbul, New York, Singapore, Seoul and Rome, but not necessarily in the
same order. The ground floor is numbered 1, the floor above it is numbered 2,
and so on, and the topmost floor is numbered 9.
The one who belongs to Seoul stays on the fourth floor. A does not belong to New
York and does not have puppy naming Sheru and Bullet. There are three floors
between the floors on which C and G stay. D stays on a floor immediately above
the I’s floor. The one who has puppy named as Gabru stays on an even numbered
floor. F does not belong to Rome .The one who belongs to London stays on the
topmost floor. The one who has puppy named as Rocky stays immediate below to
the one who has puppy named as Tiger. F has puppy named as Sheru and does not
stay on the ground floor. H belongs to Dubai and stays on an even-numbered floor
and he has puppy named as Rocky. E stays on the second floor and belongs to
Paris. There are three people between the one who has puppy named as Rony and
the one who has puppy named as Bullet. The person who has puppy named as
Rony stays below the person who has puppy named as Bullet. The one who belongs
to Rome stays on the third floor. The one who has puppy named as Jumbo does
not stay on sixth floor. There are two floors between the floors on which the
people who are from Singapore and Dubai stay. The person who has puppy named
as Bullet is from Singapore. C belongs to Bangkok. The one who has puppy named
as Cheetah stays immediate above C. There is one floor between the floors on
which F and G stay. There is one floor between the floors on which the one who
has puppy named as Panther and the one who has puppy named as Cheetah stay. A
stays on an even-numbered floor below the floor on which H stays.

Q1. How many floors are there between the one who has puppy named as Gabru
and the one who has puppy named as Jumbo?
Three
Five
Two
Six
Four

Q2. If B related to Istanbul in the same way as F related to Panther. Which of the
following is D related to, following the same pattern?
Sheru
Lucknow
Bangkok
Dubai
Jumbo

Q3. Which of the following combination is correct for A?


Panther-Seoul
Rony-Bangkok
Gabru-New York
Sheru-Istanbul
Gabru-Istanbul

Q4. Four of the following five are alike in a certain way and hence they form a
group. Which one of the following does not belong to that group?
Bangkok
Bullet
Sheru
Dubai
Tiger
Q5. If all puppies are arranged in alphabetical order from top to lowest floor. Then
the person who stays on fifth floor has which puppy?
Rony
Panther
Jumbo
Rocky
Tiger

PUZZLE-143
Directions (1-5): Study the information and answer the given questions:
Eight persons S, T, U, V, W, X, Y, and Z lives on eight storey building with ground floor
being numbered one and the next floor is number 2 and so on. They have their birth
date on August, May, September, November, July, December, June, and October but
not necessarily in same order.
S lives on even numbered floor above fourth floor. Two people lives between the floors
on which S lives and the one who was born in July. Four people lives between the floors
of U and X. U lives on even numbered floor above X. The person who was born in May
lives on odd numbered floor above the one who born in July and below the floor upon
which S lives. Number of people living between the one who born in July and S, is two
more than the number of persons between the one who was born in May and U. T was
not born in May. As many people live between the one who born in June and October
same as between the one who born in May and person T. The person who was born in
June lives on one of the floor above the one who born in October and lives on even
number floor below 8th floor but not on 4th floor.
Y lives on odd floor immediately above V. Four people lives between the one, who born
in December and November. The person who born in December lives on one of the
floors above the person who born in November. T lives immediately above Z. V was not
born in August. The number of person born between U and the one born in October is
less than 3.

Q1. S lives on which of the following floor ?


First
Fifth
Seventh
Eight
None of these

Q2. T borns in which of the following month?


June
July
October
November
None of these

Q3.How many floor between Z and S?


Three
Four
Two
Five
None of these

Q4.V borns in which of the following month?


July
October
November
September
None of these

Q5. In some way S is related to W and Y is related to V, in the same way T is


related to?
(a) Z (b) Y (c) V (d) X (e) None of these
PUZZLE-144
Directions (1-5): Study the information and answer the given questions:
Eight persons A, B, C, D E, F , G and H all of them faces north direction but not necessarily in
same order. All of them stay in different floors viz. 2nd, 6th, 10th, 15th, 20th, 30th, 31st,
and 37th but not necessarily in same order. D sits fifth to the left of the persons who stays
on 20th floor. The person on the 20th floor does not sit at extreme ends. There are two
persons sitting between the one who stays on 20th floor and F. The person staying on 6th floor
sits second to the right of G. G is not an immediate neighbor of D. G does not sit
on 20th floor. Difference between the numerical values for floor numbers of immediate
neighbours of G is 10. The person staying on 10th floor is not an immediate neighbor of F.
Two persons sitting between A and the person staying on 10th floor. Neither F nor G is staying
on 10th floor. More than one person sitting between B and the person staying on 2nd floor. The
floors on which C and H are staying are higher than the floor upon which A staying. C is not an
immediate neighbor of D. One person is sitting between B and the person staying on 31st floor.

Q1. H lives on which floor?


10th
37th
2nd
30th
None of these

Q2. How many persons sits between A and E?


two
One
three
four
None of these

Q3. C lives on which of the following floor?


15th
20th
6th
31th
None of these

Q4. Who among following sits immediate left of the person one who lives on 15th
floor? (a) A (b) F (c) D (d) B (e) None of these
Q5. Who among following sits third to right of F?
D
C
A
B
None of these

PUZZLE-145
Directions (6-10): Study the following information and answer the given questions:
Seven teachers I, J, K, L, M, N and O live on separate floors of a 7-floor building.
Ground floor is numbered 1, first floor is numbered 2 and so on. Each of them is
teaching different subjects, viz Hindi, English, Math, Science, Civics, History and
Sanskrit but not necessarily in the same order. Only three teachers live above the
floor on which I lives. Only one teacher lives between I and the one who teaches
History. N lives immediately below the one who teaches English. The one who teaches
English lives on an even-numbered floor. Only three teachers live between the
teachers who teach History and Math. M lives immediately above K. M does not teach
Math. Only two people live between J and the one who teaches Civics. The one who
teaches Civics lives below the floor on which J lives. The one teaches Hindi does not
live immediately above or immediately below J. L does not live immediately above or
immediately below I. O does not teach Science.
Q6. Which of the following is true with respect to O as per the given information?
The one who lives immediately below O ,teaches English.
O lives on floor no. 7
O lives immediately below M.
O lives on the lowermost floor.
O teaches History.

Q7. Who among the following lives on floor no. 3?


The one who teaches Science
The one who teaches Civics
K
O
M
Q8. Who lives on the floor immediately above M?
I
J
L
O
N

Q9. L teaches which of the following subject?


English
History
Math
Civics
Science

Q10. How many teachers live between the floors on which L and the one who
teaches English live?
None
Two
One
More than three
Three
PUZZLE-146
Directions (6-10): Study the following information carefully and answer the given
questions.
A, B, C, D, E, F, G and H are sitting around a square table in such a way that four of
them sit at four corners of the square while four sit in the middle of each of the four
sides. The one who sits at the four corners face the centre of the table while those
who sit in the middle of the sides face outside. Each of them likes a different colour,
viz Red, Blue, Pink, Yellow, Violet, Orange, White and Black colour but not
necessarily in the same order. C sits third to the left of the one who likes Black
colour. The one who likes Black colour faces outside. Only two persons sit between C
and H. The one who likes Red colour sits on the immediate right of H. The one who
likes Violet colour sits second to the right of G. G is neither an immediate neighbour
of H nor of C. G does not like Black colour. Only one person sits between A and the
one who likes Violet colour. D sits on the immediate left of the one who likes Orange
colour. G does not like Orange colour. E likes White colour. E is not an immediate
neighbour of A. The one who likes Blue colour is an immediate neighbour of E. The
one who likes Yellow colour is an immediate neighbour of F.

Q6. What is the position of the one who likes Orange colour with respect to G?
Second to the left
Third to the right
Fourth to the left
Second to the right
Third to the left

Q7. Who among the following represent the immediate neighbours of the one who
likes Violet colour?
B, F
C, E
B, E
D, F
F, H

Q8. Who among the following sits exactly between H and B?


C
The one who likes Blue colour
The one who likes Pink colour
G
A
Q9. Which of the following is true regarding B?
B is one of the immediate neighbours of D.
The one who likes Black colour is an immediate neighbour of B.
B sits second to the left of H.
B likes White colour.
B is an immediate neighbour of the one who likes Red colour.

Q10. Who amongst the following sits diagonally opposite the one who likes Red
colour?
The one who likes Blue colour
D
A
The one who likes Pink colour
The one who likes Yellow colour

PUZZLE-147
Directions (1-5): Study the following information carefully and answer the
questions given below:
A, B, C, D, E, F, G, and H are eight persons and they like different colours viz. Red,
Blue, Pink, Black, White, Brown, Yellow, and Violet but not necessarily in the same
order. Some of them are sitting either facing to centre or opposite to centre. There
are two persons sitting between A and H, who likes Violet colour. G sits second to left
of A. G likes yellow colour and sits not adjacent to H. F likes brown colour and sits
third to left of A. F faces the same direction as A. There are three persons sitting
between F and B, who likes black colour. D likes white colour and sits not adjacent to
B. E likes pink colour and sits third to the right of D. A does not like the blue colour. A
faces D. Both the immediate neighbor A faces the same direction as A( Faces the
same direction means if A faces towards the centre, then both neighbor of A faces
towards the centre and vice-versa). The immediate neighbor of D faces opposite
direction ( Opposite direction means if one nighbour of D faces towards the centre
then other neighbor faces opposite to centre and vice-versa). C sits immediate right
of B, and both faces opposite to centre. The immediate neighbor of E faces opposite
direction(Opposite direction means if one nighbour of E faces towards the centre then
other neighbor faces opposite to centre and vice-versa).

Q1. C likes which of the following colour?


Red
Black
Blue
Violet
None of these

Q2. Who sits third to the right of F?


D
H
C
A
None of these

Q3. Who sits third to the left of B?


G
H
A
C
None of these

Q4. Four of the following five are alike in a certain way based on their seating
positions and so form a group. Which of the following is different from the group?
A
C
F
D
H
Q5. Which of the following statements is/are definitely false?
E is the immediate neighbor of A
G sits immediate right of F.
D is the immediate neighbor of F.
G sits immediate right of E.
A likes red colour

PUZZLE-148
Directions (Q. 11-15): Study the following information and answer the questions
given below.
Eight professors A, B, C, D, E, F, G and H of different subjects viz. Science, Hindi,
English, Math, Civics, Biology, Chemistry and Physics not necessarily in same order are
sitting around a rectangular table. Four professor sits on each middle side of
rectangular table, while others four professors on the corner of table. All of them are
facing towards the center. D is sitting second to the right of the professor, who
teaches Physics. E is sitting third to the left of the professor, who teaches Biology.
Professor F and G are sitting opposite to each other. Professor C is sitting opposite the
professor, who teaches Chemistry. Professor B is sitting opposite the professor, who
teaches Science. The professor who teaches Math is sitting second to the right of the
professor, who teaches Civics and second to the left of A, who is not sitting near the
professor who teaches Chemistry. G is sitting on the smaller side and to the
immediate right of the professor who teaches Chemistry. The professor who teaches
Civics and Math are not on the same side of the table. The professor who teaches
Math is sitting third to the right of the professor who teaches Physics, who is not
sitting opposite to the professor who teaches Hindi. The professor who teaches Math
is third to the left of professor C. F sits second to the left of professor H. The
professor who teaches Hindi sits opposite to the professor who teaches Civics.
Who is sitting third to the right of the professor who teaches Biology?
(a)The professor who teaches Civics
(b)A
(c) The professor who teaches Chemistry
(d)None of these
(e) Can’t determine

Who is sitting opposite to D?


A
C
The professor who teaches Physics
The professor who teaches Hindi
(e)Can't be determined

Who among following is sitting third to left of H?


(a) E
(b) D
(c) C
(d) B
(e) A

A teaches which of the following subject?


English
Hindi
Science
Math
Civics

Biology is taught by which of the following professor?


(a) B
(b) C
(c) D
(d) E
(e) F
PUZZLE-149

Directions (1-5): Study the following information carefully to answer the given
questions.
Seven cricket players viz. P, Q, R, S, T, V and W played in seven different matches in
a week starting from Monday and ending on Sunday. Some of them are bowlers and
some of them are batsman. Their Run/Wickets in the seven matches are – 50, 5, 4,
16, 8, 4 and 25. No bowler can take more than 10 wickets.
V plays on Thursday. Two matches are played between the matches played by player
V and player P. On the first day of the week, the batsman scored half century played
in the match.
The sum of Runs/Wickets of the player W and Q is equaled to the Runs/Wickets of S.
All the bowlers should be preceded and succeeded by batsman and T is the only
bowler which cannot be succeeded by any one. P is the only batsman which is
followed by another batsman.
Only one match is played between the matches played by the player T and S. The
score of the batsman who played the match on Thursday is the perfect square of the
score of the player who played on Saturday.
The match played by T is not played on the day immediately before or immediately
after the day when the match of player V is played. The match played by R is played
on the day immediately before the day when the match of player W is played. Match
played by S is not played after the match of player Q.

Q1. As per the given arrangement which of the following combination represents
only the people who are bowlers?.
T, V
Q, P, T
V, W, R
P, T
W, T, S
Q2. As per the given arrangement which of the following person represent the one
who was played in between the W and S?
U
P
R
V
W

Q3. Who among the following takes 8 wicket ?


T
W
S
Q
None of these

Q4. Which of the following combinations is correct as per the given arrangement?
Monday – P
Tuesday – W
Wednesday – Q
Thursday – S
Friday – T

Q5. Who amongst the following scored half century?


P
V
U
W
T
PUZZLE-150
Directions (Q. 11-15): Study the following information carefully and answer the
given questions:
Sadhna, Saraswati, Savita, Sima, Sarita, Sonam, Shivi and Siddhi are eight girls sitting
around a circular table. Four of them are facing towards the centre and four of them
are facing away from the centre. All of them like a different subject viz. Hindi,
English, Sanskrit, Math, Physics, Chemistry, Economics and Commerce, but not
necessarily in the same order.
Sarita faces towards the centre and likes Hindi. Both the immediate neighbours of
Sarita face away from the centre and like Sanskrit or Math. Sima faces away from the
centre. Both the immediate neighbours of Sima do not face away from the centre.
Sarita sits third to the right of Sonam, who likes Physics and faces away from the
centre. Savita sits third to the left of Sonam.
The one who likes Sanskrit sits opposits of Sonam. The one who likes English is not the
immediate neighbour of Sonam and faces away from the centre. Sadhna sits second to
the left of Savita and does not like Economics or Commerce. The one who likes
Economics sits between Siddhi and Sonam. Saraswati faces away from the centre and
does not like English.

Q11. Who sits third to right of that girl , who sits opposite to Siddhi?
Shivi
Sonam
Sarita
Saraswati
(e)None of these

Q12. Who among following is sits second to right of Sima?


(a)Saraswati
Sonam
Shivi
Siddhi
(e)None of these

Q13. Savita likes which of the following subjects?


English
Chemistry
Commerce
Economics
(e)None of these
Q14.Sima likes which of the following subject?
Chemistry
English
Economics
Math
(e)None of these

Q15.Who among following sits third to left of Shivi?


Saraswati
Sadhna
Sima
Savita
(e)None of these

PUZZLE-151
Directions (1 – 5): Study the following information carefully and answer the
question given below:
Ten persons A, B, C, D, E, F, G, H, I and J are sitting around a rectangular table and
likes a different colours i.e. Red, Blue, White, Black, Green, Yellow, Brown, Orange,
Violet and Pink but not necessarily in the same order. All of them are sitting at a
rectangular table in such a way that four of them sit at the corners, two each on the
longer sides and one each on the smaller sides, but not necessarily in the same order.
Some of them are facing the centre while the rest are not facing the center. Not more
than two friends sitting together face the same direction.
E sits on the immediate left of D and is not an immediate neighbor of C. A and E face
in the same direction. The one who likes Red colour sits immediate left of the one
who likes Yellow colour. D and G sit diagonally and face the opposite directions. Five
of them face in the same directions. H does not like Yellow and Violet colour. The one
who likes Orange colour sits immediate right of I, who likes Violet colour. The one
who likes Yellow colour sits second to the left of C. Only two among four sitting on
the corners face outward. H and D are sitting on the immediate left and third to the
left of J respectively. A sits on one of the smaller sides and third to the right of F. D
likes Brown colour and sits third to the left of the one who likes Green colour. J sits
immediate right of the one who likes Pink colour. E likes Blue colour and sits second
to the left of the one who likes Black colour. I is not an immediate neighbour of A, B
or F, but sits on the immediate right of C, who is not facing the centre. One of the
four friends sitting on the corner is I. I faces to the centre.

1 Who among following is sits third to right of B?


The one who likes brown colour
E
The one who likes black colour
I
None of these
What is the position of H with respect to A?
(a) Immediate right
(b) Second to the right
(c) Fourth to the right
(d) Can’t be determined
(e) None of these

A likes which of the following colour?


Green
Pink
Yellow
Red
None of these

Who among the following persons like White colour?


(a)A
(b) C
(c) H
(d) J
(e) None of these
Who sits third to the left of the one who likes Yellow
colour? (a)The one who likes Orange colour
(b) The one who likes Violet colour
(c) B
(d) F
(e) The one who likes Blue colour

PUZZLE-152
There are 16 persons – B,C,D,E,F,G,H,I,P,Q,R,S,T,U,V and W standing in a square plot. Inside a Square
plot, a square shaped garden is developed. The persons who are standing inside the garden facing
outside. The persons who are standing outside the garden facing inside the centre and likes colours
namely viz., – Red, Blue, Black, Brown, Yellow, Green, Violate and Pink. So all the persons standing in the
inner square faces the persons standing in the outer Square and likes fruits namely viz., – Apple, Orange,
Mango, Grapes, Papaya, Pomegranate, Guava and Banana. G faces the centre and W faces G. D sits
second to the right of G. There are four persons sits between G and E. D is not an immediate neighbour
of E. There are three persons standing between I and E. There are two persons standing between I and
B. B stands exactly between the E and F. F stands to the immediate left of G. There are two persons
standing between W and U. U faces H. T faces outside. There are two persons standing between T and
Q. T faces C. Q stands to the immediate left of W. R, the one who faces B stands exactly between the
persons P and V. P faces E. The one who sits in the corner of the square likes Red. The one who likes Red
sits between the persons who like Black and Blue. The one who likes Blue sits second to the right of the
person who likes Green. Three persons sit between one who likes Black and one who likes Green. Two
persons sit between one who likes Black and one who likes Yellow. Two persons sit between one who
likes Yellow and one who likes Pink. G and F do not like Violate and Yellow respectively. The one who
likes Red faces P. The immediate neighbours of P are the one who likes apple and the one who likes
Grapes. The one who likes Apple faces the one who likes Black. Three persons sit between the one who
likes apple and the one who likes Guava. The immediate neighbours of the person who likes Orange are
the one who likes apple and the one who likes Pomegranate. The one who likes Papaya sits exactly
behind to the one who likes Orange. The one who likes Banana sits exactly behind to the one who likes
Mango. The one who likes Banana faces E.

PUZZLE-153
Directions (1-5): Read the following information carefully and answer the questions given
below.
Five friends G, H, I, J and K of one colony A are playing chess with five other friends namely U,
V, W, X and Y of another colony B. Each friend of colony A has a different profession viz.
Doctor, Engineer, Businessman, Lawyer and Manager but not necessarily in the same order
.Each friend of colony B has also a different profession viz; Fashion designer, Choreographer,
Interior Decorator, Politician and Teacher. Among friends, there are only three females – the
one who is playing with W, the one whose profession is Doctor and the one who is playing with
the person who is a Politician. The female whose profession is Engineer is playing with the
person who is Interior Decorator.
The Person who is a Choreographer is playing with the person whose profession is Doctor .X is
neither a Choreographer nor a Fashion designer. Y is not an Interior Decorator. The person who
is playing with Y is a female but she is not J. K’s profession is Engineer and is not a male. G
and I are not females. G’s profession is Manager and is playing neither with Y nor with the
Choreographer. H’s profession is Lawyer. V is a Fashion designer and he is not playing with G.
The one who is a Fashion designer is not playing with J who is a doctor.
Q1. The one whose profession is Businessman is playing with who among the following
person?
(a)W
(b)Choreographer
(c)Y
(d)Fashion designer
(e)None of these

Q2. Which of the following pairs correctly represents the pair of friends who are playing
together?
(a)IW
(b)GX
(c)JY
(d)WH
(e)None of these

Q3. Who among the following is a Politician?


(a)Y
(b)X
(c)W
(d)U
(e)None of these
Q4. Which of the following represents the group of persons having female friend who are
playing with him?
(a)Y, W,U
(b)X, V, W
(c)R, P, T
(d)Y ,X, V
(e)None of these
Q5. Which of the following is the specialization of X?
(a)Teacher
(b)Politician
(c)Interior Decorator
(d)Fashion designer
(e)None of these
PUZZLE-154
There are eight friends A, B, C, D, E, F, G and H, who live in an eight-storey building. The
ground floor is numbered one and the topmost floor is numbered eight. Each of them are
working in different nationalize bank, viz BOI, BOB, Dena, UBI, CBI, IOB, PNB and OBC, but
not necessarily in the same order. There is only one floor between A and the floor on which
person that works in OBC. The person who works in OBC does not live on floor number
D lives just below B. The one who works in BOI live on even-numbered floor and just above
the floor on which person who works in CBI lives. The person who works in IOB lives on an
even-numbered floor but not on the 8th floor. Neither D nor H lives on the 1st floor. Only one
person lives between the one who works in PNB and D. A lives on an odd-numbered floor and E
lives just above A. B lives on the fourth floor. Only two friends live between the one who works
in IOB and A. F lives just below the one who works in CBI. D works neither in CBI nor OBC. The
person who works in Dena bank does not live on an odd-numbered floor. G does not work in
BOB. There are two floors between the floor on which H lives and the floor on which E lives.
Only two persons live between the one who works in UBI and the one who works in DENA
bank.
Q6. Who among the following works in BOB?
D
C
F
E
None of these

Q7. How many persons are there between E and B?


One
Two
Three
Four
None of these

Q8. Who among the following lives on the topmost floor?


The one who works in BOI
The one who works in IOB
The one who works in DENA
The one who works in BOB
None of these

Q9. Which of the following combinations is/are true?


Floor no. 2 – D – UBI
Floor no. 5 – F – OBC
Floor no. 1 – C – BOB
Floor no. 8 – E – BOI
None of these
Q10. A works in which of the following bank ?
BOI
OBC
IOB
CBI
None of these

PUZZLE-155
Directions (1–5): Study the following informa¬tion carefully and answer the questions which
follow.
Eight delegates from different countries, Brunie, Cambodia,Malaysia, Myanmar, Vietnam, Thailand,
Philippines and India are participating in the ASEAN summit in laos. They are A, B, C, D , E, F, G and H
are sitting around a circular table facing to the centre but not necessary in the same order. They all are
the ministers belonging to different departments Defense, Tourism, Nuclear energy, External affairs,
Home affairs , Railway, Finance and Human resource development.
F is seated second to the right of H and they both do not belong to Finance department. G does not work
in tourism or railway department but sits 3rd to the left of the person who belong to Finance department.
C sits immediate left to the person who belongs to Tourism department and sits opposite to one who is
immediate right of H. A is on the 3rd to the left of D and opposite to one who belongs to Finance
department. The person who belongs to Home affairs department is not near to the person who belongs
to Tourism and Finance department. F is near to E. The person belongs to Home affairs department is
3rd to the left of the person who belongs to external affairs department. A and H don't belong to Human
resource department and Railways department. B is not near to the person who belong to Nuclear energy
department . D is seated opposite to one who belongs to Home affairs department.
Who belongs to the Defense department?
(a) F (b) G (c) H (d) C (e) None of these

2.What is the position of the person who belongs to External affairs department with respect to
the person who belongs to Human resource development department?
Second to the left
Immediate left
Second to the right
Third to the right
Immediate right

3.The person who belongs to Finance department sits opposite to one who belongs to
______________________ Department.
Human resource
Nuclear energy
(c)Railways
Home affairs
External affairs

4.Which of the following statement is true?


E is seated opposite to C and belongs to Home affairs department.
F is seated opposite to D and belongs to railway department.
A is seated opposite to E and belongs to Nuclear energy department.
D belongs to Tourism department and seated opposite to the person who belongs to Home affairs
department.
None of the above is true.

5.'H' is related to 'Nuclear energy' in the same way as 'C' is related to 'Defense' based on the given
arrangement. Who among the following is ‘F’ related to, following the same pattern?
(a)External affairs
Human resource
development (c)Railways
Finance
Home affairs
PUZZLE-156
Directions (1-5): Study the following information and answer the questions given below:
There are seven persons, namely P, Q, R, S, T, U and V. They go to gym by different car namely Audi,
Toyota, Tata, Ford, Nissan, Hyundai and Fiat, but not necessarily in the same order, starting from
Monday to Sunday (of the same week). T goes to gym on Thursday. Only one person goes to gym
between T and the one who goes to gym by Nissan car. P goes to gym immediately after the one who
goes to gym by Nissan car. Only three people go to gym between P and the one who goes to gym by
Tata car. Only two people go to gym between the one who goes to gym by Tata car and U. The person,
who goes to gym by Fiat car, goes to gym before U, but after Thursday. More than two people go to gym
between the one who goes to gym by Fiat car and R. The one who goes to gym by Audi car goes to gym
immediately before the one who goes to gym by Hyundai car. The one who goes to gym by Ford car goes
to gym immediately before Q. V does not go to gym on Tuesday.

Q1.Who amongst the following goes to gym by Toyota car?


(a)R (b)S (c)P (d)T (e)V

Q2.Four of the following five are alike in a certain way and so form a group. Which of the following
does not belong to that group?
(a)R-Tuesday
(b)T-Monday
(c)Q-Sunday
(d)P-Friday
(e)U-Saturday

Q3.Which of the following is true about S?


(a)S goes to gym immediately after R.
(b)S goes to gym by Tata car.
(c)Only two people go to gym between S and R.
(d)All the given statements are true
(e)S goes to gym on Sunday.

Q4.T has which of the following car?


Tata
Audi
Hyundai
Ford
(e)Fiat

Q5. As per the given arrangement Q is related to the one who goes to gym by Tata car in a certain
way and P is related to the one who goes to gym by Fiat car , in the same way, U is related to?
The one who goes to gym by Toyota car.
The one who goes to gym by Ford car.
The one who goes to gym by Tata car.
The one who goes to gym by Audi car.
None of these.
Days Person Car

Monday R Audi

Tuesday S Hyundai

Wednesday V Tata

Thursday T Ford

Friday Q Fiat

Saturday U Nissan

Sunday P Toyota

PUZZLE-157
Directions (11-15): Study the following information carefully and answer the questions given
below.
There are ten students are sitting in twelve seats in two parallel rows containing five students on each, in
such a way that there is an equal distance between adjacent students. In row 1, A, B, C, D and E are
seated and all of them are facing south, and in row 2, P,Q, R, S and T are sitting and all of them are
facing north. One seat is vacant in each row. Therefore, in the given seating arrangement each member
seated in a row faces another member of the other row. All of them have a different favorite subject i.e.
Hindi, English, Sanskrit, Urdu, Art, Math, Science, Economics, Biology and Physics.
A sits second to left of one, whose favorite subject is Hindi. Either A or the one, whose favorite subject is
Hindi, seats adjacent to the extreme end position. T sits one of the extreme ends of the row. There are
three students sit between T and S, whose favorite subject is English. Immediate neighbor of T faces B.
One of immediate neighbor of B faces R, whose favorite subject is Sanskrit. There are no vacant seats
adjacent to R. C’s favorite subject is Urdu and sits second to left of vacant seat. One of immediate
neighbor of R is Q. Q’s favorite subject is Art. One of immediate neighbor of Q faces D. D’s favorite
subject is Math. The one whose favorite subject is Science sits immediate left of the one whose favorite
subject is Economics. The one whose favorite subject is Biology sits third to left of one whose favorite
subject is Physics.

Q11. Who among following sits on the extreme end of the row?
E,Q
B,P
B,S
D,P
None of these

Q12. Whose favorite subject is Physics?


(a)P
(b)R
(c)Q
(d)T
(e)S

Q13. Who among following sits second to left of T?


S
P
R
Q
None of these

Q14. Who among following sits opposite to C?


P
Q
R
S
T

Q15. Whose favorite subject is Science?


A
B
C
D
E

PUZZLE-158
Directions (1-5): Study the following information carefully to answer the given questions.
There are ten girls who sits on two parallel rows of bench containing five people each, in such a way that
there is an equal distance between adjacent girls, for doing some college work and they like different
mobile phones viz. LG, Redmi note 4, Redmi , Moto x, Lenovo, Samsung, Nokia, MI5, I Phone 6 and
Vivo. In row 1- C, D, E, F, and G are seated and some of them are facing south and some of them are
facing north. In row 2 – M, N, O, P, and Q are seated and some of them are facing south and some of
them are facing north. Therefore in the given seating arrangement, each member seated in a row either
faces another member of the other row or seated behind each other.
The girl who likes Moto x sits to the immediate right of N, who seated exactly in the middle of the row.
One of the immediate neighbors of the girl, who likes Lenovo, sits behind the girl who likes Redmi. P does
not like Redmi. F likes neither Lenovo nor Samsung. M sits immediate right of the girl who likes I Phone 6.
E sits third to the right of the girl who likes Redmi note 4. M does not face C and faces south direction.
The girl who likes LG sits exactly between the girls who like Vivo and Lenovo C likes Lenovo and sits in
the middle of the row 1. Q faces North Direction and sits immediate left of N. Only one girl sit between the
girls who like Redmi and Moto X. M faces one of the immediate neighbors of the girl who likes Redmi
note 4. F faces one of the immediate neighbors of the girl who likes Redmi. Only One girl sits between the
girl who likes I Phone 6 and N who likes Nokia. E sits to the immediate right of the girl who faces P. Only
two people sit between E and G. P likes neither LG nor MI5. The girl who likes Lenovo sits second to the
right of the one who faces North Direction. O sits one of the extreme ends of the line and likes I Phone 6.
C faces the opposite direction to the girl who likes Nokia. F faces north direction and the one who faces F,
faces south direction. D faces north direction.

Q1. Who amongst the following faces the girl D?


The girl who likes LG
F
The girl who likes Lenovo
The girl who likes Moto X
D

Q2. O likes which Mobile phones?


LG
Nokia
I Phone 6
Samsung
Redmi

Q3. Which of the following is true regarding Q?


Q faces south direction.
None of the given options is true
Q likes Redmi
The girl who likes MI5 faces Q
Q is E‘s immediate neighbour

Q4. Which one is the immediate neighbor of F?


O
P
The one who likes Redmi
The one who likes LG
The one who likes Samsung

Q5. Who amongst the following sit at middile of the row 1?


G
D
E
The girls who likes Lenovo
None of these
PUZZLE-159
Directions (1-5) : Study the following information carefully and answer the questions which
follow–
Mr. X, who lives in Bihar has five sons i.e. A, B, C, D and E and they are working in five different
organizations-Google, Bank of America, HSBC Holdings, Bank of China and NetApp but not necessary in
that order. Mr. Y who lives in Punjab has five daughters i.e. P, Q, R, S and T and they are working in five
different organizations-Barclays, Microsoft, Goldman Sachs, Deutsche Bank and Cisco but not necessary
in that order. These five boys are married to these five girls but not necessary in given order.
S is either working in Barclays or Cisco but does not married to one who works in Bank of America. A
does not work in Bank of America or Google but married to one who works in Cisco. The one who works
in Goldman Sachs is married to E who does not work in Google and Bank of America.
The boy who works in Google is either married to S or T. E is not married to P, who works either in
Goldman Sachs or Deutsche Bank. A is working in Netapp and married to S. D does not work in HSBC
Holdings or Bank of America but married to one who works in Microsoft.
C is married to R but does not work with NetApp or Bank of America. R does not married to one who
works in Bank of America. Q is married to one who works in HSBC Holdings.

Q1. Who is married to T?


(a) A (b) B (c) C (d) D (e) E

Q2. Who is the employee of Bank of China?


(a) A (b) B (c) C (d) D (e) E

Q3. In which of the following bank the wife of E is working?


(a) Goldman Sachs (b) Cisco (c) Barclays (d) Microsoft (e) Deutsche Bank

Q4. Who works with Microsoft?


(a) Wife of C (b) P (c) Q (d) T (e) Wife of one who works in Bank of America

Q5. Who is married to one who works with NetApp?


The one who works with Deutsche Bank
The one who works with Barclays
The one who works with Goldman Sachs
The one who works with Cisco
Cannot be determined
PUZZLE-160
Directions (1-5): Read the following information carefully and answer the questions given below:

Six students A, B, C, D, E and F are sitting around a circular table facing towards the center of the table in
a library. They read different comics like Indrajal, Raj, King, Nutan, Champak and Tulsi but not
necessarily in the same order. They are wearing T-shirts of different colours, i.e. white, violet, green, red,
yellow and blue but not necessarily in the same order. The students, who read Indrajal, King and
Champak are neither in white T-shirt nor in Violet. The students who are in green and yellow T-shirts
have neither read Indrajal nor King comics. A is neither in white T-shirt nor on the immediate left of the
one who reads Nutan comics. The only student who is between E and F reads Raj comics. The one who
is on the immediate left side of the student who is in white T-shirt does not read Tulsi comics. D reads
Nutan comics and the colour of his T-shirt is green, he is facing the student who reads Raj comics. The
One who reads Indrajal comics is seated opposite to the student wearing blue T-shirt, while the student
whose T-shirt is of green colour is on the immediate left of the student who reads Champak comics. One
who reads Tulsi comics is on the immediate right of the student in white T-shirt but on the immediate left
of the student who reads King comics. C does not read King comics while F does not read Indrajal
comics.

Q1. Who among the following is in white T-shirt?


(a) A (b) B (c) C (d) E (e) None of these

Q2. The only student, who is between E and D, is wearing T-shirt of the colour?
red
blue
violet
yellow
None of these

Q3. Who among the following has read Champak comics?


(a) A (b) B (c) C (d) E (e) None of these

Q4. Which of the following is correctly matched?


A-Yellow-Nutan
B-Red-King
E-Red-Indrajal
F-Violet-Champak
None of these

Q5. The colour of the T-shirt of the student, who reads Tulsi comics, is
red
yellow
blue
violet
None of these
PUZZLE-161
Directions (Q. 1-5): Study the following information and answer the questions given below.
Eight persons A, B, C, D, E, F, G and H like different mobile phones viz. Apple, Garmin, Vertu, Thuraya,
Foxconn, Pantech, Wiko and Lanix but not necessarily in same order. All of them are sitting around a
rectangular table. Four persons sits on each middle side of rectangular table, while other four persons on
the corner of table. All of them are facing towards the center. D is sitting second to the right of the one,
who likes Lanix mobile. E is sitting third to the left of the one, who likes Pantech mobile. F and G are
sitting opposite to each other. C is sitting opposite to one, who likes Wiko mobile. B is sitting opposite the
one, who likes Apple mobile. The one who likes Thuraya mobile is sitting second to the right of the one,
who likes Foxconn mobile and second to the left of A, who is not sitting near the one, who likes Wiko
mobile. G is sitting on the smaller side and to the immediate right of the one, who likes Wiko mobile. The
persons who like Foxconn and Thuraya mobile are not on the same side of the table. The one, who likes
Thuraya mobile is sitting third to the right of the one ,who likes Lanix mobile, who is not sitting opposite to
the person, who likes Garmin mobile. The one who likes Thuraya mobile is sitting third to the left of C. F
sits second to the left of H. The one who likes Garmin mobile sits opposite to the one who likes Foxconn
mobile.

Who is sitting third to the right of the one, who likes Pantech
mobile? (a)The one, who likes Foxconn mobile.
(b)A
(c) The one who likes Wiko
mobile. (d)None of these
(e) Can’t determine

Who is sitting opposite to D?


A
C
The one who likes Lanix mobile.
The one who likes Garmin
mobile. (e)Can't be determined
Who among following is sitting third to left of H?
(a) E
(b) D
(c) C
(d) B
(e) A

A likes which of the following mobile?


Vertu
Garmin
Apple
Thuraya
Foxconn

Pantech mobile is like by which of the following one?


(a) B
(b) C
(c) D
(d) E
(e) F

PUZZLE-162
Directions (1-5): Read the following information carefully and answer the questions given below.
Eight family members P,Q,R,S,T,U, V and W are seated in a straight line at an equal distance between
each other, but not necessarily in the same order. There are three generation in the family. Some of them
are facing north and some are facing south. R’s daughter is an immediate neighbour of one who is sitting
at an extreme end of the line. Only three people sit between V and his wife. Q’s daughter sits second to
the right of P’s brother-in-law. R’s granddaughter does not sit at an extreme end of the line. P’s mother
sits on the immediate left of her son. R’s wife is not an immediate neighbour of T’s husband. The
immediate neighbour of T’s brother faces opposite directions.(i.e. If one neighbour faces north then other
faces south and vice versa.) The persons sitting at the extreme ends faces opposite directions.(i.e. If one
person faces north then other faces south and vice versa.) W’s daughter-in-law sits second to the left of
T’s father-in-law. U faces north. U is not an immediate neighbour of S’s aunt. The immediate neighbours
of U faces same directions.(i.e. If one neighbour faces north then other also faces north and if one
neighbour faces south then other also faces south).Both T and Q face a direction opposite to that of
S.(i.e. If S faces north then T and Q faces south and vice-versa.) T is the only daughter of R. W is female
member and U is male member.

Q1. As per the following arrangements, which of the following statements is not true with respect
to P?
P faces south.
P is fourth to the right of Q.
P is 2nd to left of T.
P is between V and W.
None of these

Q2. How many people sit on the left of Q?


(a) One (b) Two (c) Three (d) Four(e) None of these

Q3. What is the position of S with respect to P?


(a) Immediate left (b) Third to left (c) Third to right (d) Fourth to left (e) None of these

Q4. Who among following is grand-mother of S?


(a) T (b) S (c) W (d) R (e) None of these

Q5 Four of the given five are alike in a certain way based on the given arrangement and hence
form a group. Which of them does not belong to that group .?
R
S
P
U
V
PUZZLE-163
Directions (1-5): Read the following information carefully and answer the questions given below:
There are eight persons P, Q, R, S, T, U, V and W are sitting around a circle. Some are facing the center
while some are not. They also like different car like Audi, Maruti, Toyota, Nissan, Ford, Fiat, Skoda and
Hyundai. P is sitting third to right of R. There is one person sitting between P and Q. S is sitting fourth to
right of Q. The person who like Toyota sits second to right of S. P does not like Toyota car. Three persons
sit between the person who like Toyota and Hyundai car. The person who like Audi car sits second to
right of person who like Hyundai car. S does not like Audi car. The person who likes Fiat sits third to right
of person who like Audi car. R likes Fiat car. There is two persons sits between S and T. There is three
persons sits between the person who like Ford and Nissan car. T does not like Ford and Nissan car.
There is one person sits between the person who like Nissan and Maruti car. The person who like Toyota
car sits third to right of person who like Maruti car. The person who like Skoda car sits third to right of
person who like Nissan car. V sits fourth to right of person who like Ford car. U is not immediate neighbor
of V. The person who like Ford car sits third to left of person who like Toyota car and both faces the same
direction.(Same direction means if one faces center then other also faces the center and vice-versa).T
faces opposite the center. P likes the Hyundai car.

Q1. Who among the following like Toyota car?


(a) P (b) Q (c) R (d) T (e) W

Q2.If T is related to R and Q is related to S, in the same way P is related to?


P
Q
R
T
W

Q3. Who among the following sits third to right of person who like Hyundai car?
U
S
V
(d)W
E

Q4. Which of the following is correctly matched?


T-Audi
R-Toyota
S-Skoda
(d)U-Nissan
W-Fiat

Q5. Who sits opposite to person who like Audi car?


U
The person who likes Nissan car.
R
The person who likes Hyundai car.
The person who likes Skoda car.
PUZZLE-164
Directions (1-5): Read the following information carefully and answer the questions given below:
In a recent union budget finance minister Arun Jaitely disclose a budget of 2017. He adequate some
money (crores) in different-different sectors i.e. Digitalization, Agriculture, Defense, Infrastructure, Social
welfare, Science research and Education but not less than 11cr in any sector. Each sector is headed by
different-different minister namely I, J, K, L, M, N and O, but not necessarily in the same order. Each
minister has a different age.
Neither N nor oldest minister is head of Social welfare sector. In defense sector Arun Jaitely adequate
78cr. M is older than K but younger than N. 4th oldest minister is 55 year old and Education sector is
headed by him. 2nd youngest minister is heading Defense sector. In two sectors Arun Jaitely adequate
same money. Total amount adequate by Arun Jaitely is 192cr. K is not youngest minister. The number of
ministers, who is younger and older than O is same. J is the head of that sector for which Arun Jaitely
adequate 35cr. Difference of money allocated for Agriculture sector and Social welfare sector is 20cr.
youngest minister is head of Agriculture sector. I is younger than L but older than K. M is not head of that
sector for which Arun Jaitely adequate 78cr. At least three minister younger than I. 5th youngest minister
is not head of Social welfare sector. 3rd oldest minister is not head of Science Research sector. N is head
of that sector for which Arun Jaitely adequate 1cr more than that sector, which is headed by M.
Infrastructure sector is headed by the 7th youngest minister and in that sector for which Arun Jaitely
adequate 22cr. In digitalization sector Arun Jaitely adequate least money.

Q1. Which of the following sectors is headed by minister N?


Defense
Education
Science research
Digitalization
Agriculture
Q2. Which of the following combination is true?
J-Agriculture- 78cr
None of these
O-Science research-15cr
M-Social welfare- 15cr
K- Defense- 22cr

Q3. If L is 13 year older than 4th oldest minister, then what is the possible age of 6th youngest
minister?
54 year
63 year
Can’t be determined
More than 68 year
Less than 55 year

Q4. If Social welfare is related to 15cr in the same way as Science research is related to 16cr. Then
which of the following is Digitalization related to, following the same pattern?
15cr
None of these
78cr
22cr
11cr

Q5. Which of the following minister is head of that sector for which Arun Jaitely adequate 3rd
least amount?
J
N
O
I
L
PUZZLE-165
Directions (1-5): Study the following information carefully and answer the questions.
There are seven students – M, N, O, P, Q, R and S – who like different subjects viz- Hindi, English, Math,
Science, Physics, Chemistry and Biology but not necessarily in the same order. They read on different
days of the week starting from Monday. They also like different sports, viz- Hockey, Cricket, Football,
Tennis, Badminton, Baseball and Kho-Kho but not necessarily in the same order.
N, who likes Baseball, reads on the fourth day of the week but neither Hindi nor Biology. Two students
read between the days on which N and R read and neither of them read on the 1st day of the week.
There is only one student who reads between M and O. But O does not read either on 1st or 3rd day of
the week. O likes Cricket sport and reads Hindi. The one who reads Math on the last day of the week
likes Football sport. P does not read on the day either immediately before or immediately after the day on
which M read, who doesn’t like either Badminton or Hockey or Tennis sport. P does not read science. S
reads immediately after O and he likes Hockey sport. Q does not like Badminton sport and reads
Chemistry. M does not read either Physics or Science. The one who reads English takes class
immediately after the day when Hindi has been read.

Q1. Which of the following combinations is definitely false?


(a)O – Cricket – Hindi
(b)M – Kho-Kho –Biology
(c)Q – Tennis –Chemistry
(d)R – Cricket – Hindi
(e)None of these
Q2.Who among the following reads on the 5th day of the week?
(a)The student who likes Kho-Kho sport
(b)The student who reads Math
(c)The student who reads Hindi
(d)The student who likes Tennis sport
(e)None of these

Q3.Who read Science?


(a)The student who likes Baseball sport
(b)The student who likes Kho-Kho sport
(c)The student who likes Badminton sport
The student who likes Football
sport (e)None of these

Q4.If ‘N’ is related to ‘Cricket’ and ‘S’ is related to ‘Football’ then which of the following is ‘M’
related to?
(a)Badminton
(b)Baseball
(c)Tennis
(d)Kho-Kho
(e)None of these
Q5.Who reads Physics?
(a)The student who reads on fourth day of the week
(b)The student who reads just before Q
(c)The student who reads just after Q
(d)Can’t be determined
(e)None of these

PUZZLE-166
Directions (1-5): Read the following information carefully and answer the questions given below:
There are nine floors building in the area in which nine people are living on different floor separately. The
ground floor is numbered 1 and so on. Nine persons are A, B, C, D, E, F, G, H, and I. They also like
different colours like Red, Blue, Black, White, Yellow, Violet, Pink, Orange and Brown but not necessarily
in the same order.
I lives on the even number floor but not on the eight floor. There is gap of two floor between the I and the
person who likes Orange colour. There is gap of two floor between the person who like Orange and Blue
colour. The person who likes Orange colour lives on top floor. The person who likes Red colour lives on
the odd number floor. E likes Red colour and he does not live immediately above or immediately below
the floor on which I lives. There are gap of two floors between the floor on which E and D live. D likes
Yellow colour. A likes violet colour and he lives on the even number floor. There are gap of two floors
between A and the person who likes White colour. There is gap of one floor between the person who likes
White and Brown colour. B likes brown colour and lives one of the floor above on which the person who
likes White colour lives. C likes Black colour and lives on even number of floor but not on the second
floor. F does not like Pink and Orange colour. G does not like Orange colour.

Q1. Who lives on floor number 3?


(a) E (b) A (c) G (d) D (e) F

Q2.Which of the following is correct?


(a) C-Black-9th floor (b) I-Blue-7th floor (c) E-Red-2nd floor (d) G-Pink-3rd floor (e) G-Orange-9th floor
Q3.F is related to Brown and A is related to Yellow, in the same way I is related to?
White
Orange
Black
Red
None of these

Q4.How many floors are between the floor on which I and E live?
one
two
three
more than three
None of these

Q5.How many floor below on which D lives?


One
two
more than three
three
None of these

Solution (1-5):
PUZZLE-167
Directions (6-10): Study the following information carefully and answer the question given below-
A, B, C, D, E, F, G and H are eight kids sitting around a circular table. Four of them are facing away from
the centre and four of them are facing towards the centre. Each of them like different candies-Hajmola,
Poppins, Kismi, Satmola, Aam Pachan, Rochak, Pan pasand and Chatmola. All of them are holding a
different colour balloon viz. White, Blue, Orange, Pink, Green, Purple, Yellow and Red but not necessarily
in the same order. E faces towards the centre and holds White balloon. Both the immediate neighbors of
E face away from the centre and are holding either Orange and Pink balloon. D faces away from the
center and his favorite candy is Hajmola. Both the immediate neighbours of D do not face away from the
centre. E sits third to the right of F, who has a Green balloon and faces away from the centre. C sits third
to the left of F. The one who has an Orange balloon sits opposite to F. The one who has Blue balloon is
not the immediate neighbour of F and faces away from the centre. A sits second to the left of C and he
have neither Yellow nor Red balloon. The one who has a Yellow balloon sits between H and F. B faces
away from the centre, likes Poppins and does not have a Blue balloon. E’s favourite candy is Pan
pasand. The person who likes Satmola opposite to D. The person having Purple balloon likes kismi. The
person who likes Chatmola is not near to G nor E. C faces the person who likes Rochak.

Q6. Which one of the following related to Purple?


E
F
B
D
A

Q7. Which one of the following related to Orange?


(a) B (b) H (c) F (d) D (e) None of these

Q8. How many persons are there between the one who related to Pink and the one who related to
Orange when counted in anticlockwise direction from the person who related to Pink?
Four
Two
Three
CND
None

Q9. Which one of the following related to Pink?


B
H
F
D
None of these

Q10. Which of the following is B’s position with respect to F?


Fourth to the left
Third to the right
Second to the left
Second to the right
None of these
PUZZLE-168
Directions (Q. 1 – 5): Study the following information carefully and answer the questions which
follow—
A, B, C, D, E and F are six persons who have joined 6 different banks OBC Bank, Punjab National Bank,
Bank of Maharashtra, Bank of India, Bank of Baroda, and State Bank of India by scoring different marks
in the written exam of a maximum of 200 marks. (marks are an integer value) They all are sitting around a
circle facing the centre with equal distance.
C is second to the right of the person who joined Bank of Baroda whose score is 169 marks which were
the third lowest marks out of the 6 students. A is immediate to the left of the person who is opposite to the
person who joined State Bank of India, who is not near to F. B scored the 2nd highest marks and did not
join Bank of India but is 2nd to the left of the person, who scored 174 marks. D is seated opposite to one
who joined Bank of India. D is not near to B. E has not scored the lowest marks. C is immediate to the left
of the one who joined State Bank of India and C scored 170 marks. The person who joined Punjab
National Bank is second to the left of the one who scored 172 marks. The person who joined OBC did not
score the highest marks. One of the six students scored which was a prime number. The lowest scored
mark is 164.

Q1. Who amongst the following is from OBC?


(a) D (b) E (c) C (d) A (e) B

Q2. Four of the following five are alike in a certain way based on the given seating arrangement
and thus form a group, who is the one that does not belong to that group?
(a) A (b) B (c) C (d) D (e) E

Q3. Who is seated between D and the person from Bank of Baroda?
(a) E (b) A (c) C (d) None (e) The person from Bank of Maharashtra
Q4.Which of the following is true regarding the given information?
F is from Bank of Baroda and seated immediate right of the person who is opposite to the person who
joined bank of Maharashtra.
A is from Punjab National Bank and scored 174 marks and is opposite to C
E is from OBC and is to the immediate right of B
C scored 170 marks and is opposite to one who joined Bank of Maharashtra.
The person from Bank of Maharashtra is seated opposite to one who scored 164 marks.

Q5.Which of the following order of marks in descending order is true?


A>B>C>D>E>F
A>B>C>F>E>D
A>B>E>C>F>D
A>B>C>E>F>D
A>B>C>D>F>E
PUZZLE-169
Directions (1-5): Read the following information carefully and answer the questions that follow
Ten persons are sitting in two parallel rows containing five persons each. In row 1, A, B, C, D and E are
sitting and all of them are facing south. In row 2, P, Q, R, S and T are sitting and all of them are facing
north. In the given seating arrangement, each member seated in a row faces another member of the
other row. Moreover, each of them is going in the market and each person eats different types of foods
i.e. Idali, Cutlet, Dosa, Golgappe, Bread roll, Tikki, Noodles, Biryani, Chili potato and Eggroll, but not
necessarily in the same order.
There are only two person’s sit between the one who eats Idali, who sits at an extreme end and E. P, who
sits in the middle of the row, is not an immediate neighbour of Q, who doesn’t eat Eggroll. T is sitting at an
extreme end. E eats Bread roll, sits on the immediate right of the person who eats Cutlet and faces the
immediate neighhour of Q. B is not sitting at the extreme left end. P does not eat Chili potato. There is
only one person between C and D, who eats Idali. S eats Tikki, is an immediate neighbour of the person,
who eats Eggroll and does not face the person, who eats Dosa. R, who eats Golgappe, is an immediate
neighbour of the person who eats Chili potato and the person who eats chili potato faces the immediate
neighbour of the person who eats Bread roll. There are two persons between the person who eats
Noodles and the person who eats Dosa. B does not eat Noodles. T does not eat Biryani.

Q1.A eats which of the following foods?


Biryani
Noodles
Dosa
None of these
Can't be determined

Q2.Who eats Biryani?


R
B
T
P
Can't be determined

Q3.‘Idali’ is related to ' Noodles ' in a certain way, based on their seating positions. Then Eggroll
is related to whom, following the same seating positions?
Biryani
Chili potato
Cutlet
Tikki
Golgappe

Q4.Four of the following five are alike in a certain way based on their seating positions and
so form a group. Which of the following is different from the group?
Bread roll
Noodles
Eggroll
Golgappe
Tikki
Q5.Which of the following statements is/are definitely false?
B eats Dosa.
There are two persons sitting between the person who eats Tikki and the person, who eats
Chili potato.
The person who eats cutlet faces the person who eats Biryani.
The person who eats Chili potato sits opposite the person, who eats Idali.
All are true

PUZZLE-170
Directions (11-15): Study the following information carefully and answer the question given below:
There are eight persons namely- P, Q, R, S, T, U, V and W sitting around a circular table facing the
centre, but not necessarily in the same order. Each person belongs to different country i.e. Australia,
South Africa, Bangladesh, New Zealand, West Indies, Srilanka, India and Pakistan also likes different
kinds of fruits viz.- Mango, Litchi, Apple, Orange, Pineapple, Grapes, Guava and Banana, but not
necessarily in the same order.
Person belongs to India sits second to the right of S, who belongs to neither Srilanka nor West Indies. U
sits second to the right of the one, who belongs to Srilanka. The one who likes Pineapple sits second to
the left of the one, who belongs to South Africa. Two people sit between the one, who belong to West
Indies and Q. V and the one, who belongs to New Zealand are immediate neighbours of each other.
Persons who like Orange and Grapes are adjacent to the one who belongs to New Zealand. Person likes
Orange sits immediate left of the one who belongs to South Africa. Q does not like Guava and Banana.
The one who likes Banana sits second to the right of the one who likes Guava. R and T is immediate
neighbour of each other. Neither R nor T is an immediate neighbour of either Q or the one who belongs to
Srilanka. Q is not belongs to New Zealand. The one who belongs to South Africa sits second to the left of
the one who belongs to Pakistan. Person who likes Apple sits fourth to the right of the one who likes
Litchi. The one who likes Guava is not immediate neighbour of the one who belongs to Srilanka. Neither
R nor T sits third to the left of the person, who belongs to Srilanka. P likes Banana. Only one person sits
between R and the one who belongs to Australia. W sits third to the left of the one who belongs to
Bangladesh. W does not like Apple.

Q11. What is the position of Q with respect to the one, who belongs to South
Africa? (a)Second to the left
Second to the right
Sixth to the left
Third to the right
Both (b) and (c)
Q12. Which of the following fruits does Q like?
(a)Pineapple
Mango
Apple
Litchi
None of these

Q13. Who sits opposite P?


(a)The one who belongs to Srilanka
(b)The one who belongs to South Africa
(c)The one who belongs to New Zealand
(d)The one who belongs to Bangladesh
(e)None of these

Q14. Four of the following five are alike in a certain way and hence they form a group. Which one
of the following does not belong to that group?
(a)S
The one who likes Banana
The one who belongs to South Africa
The one who belongs to New Zealand
The one who likes Orange

Q15. U belongs to which of the following country?


(a)Bangladesh
India
West Indies
South Africa
New Zealand
PUZZLE-171
Directions (1-5): Study the following information and answer the following questions:

A, B, C, D, E, G and I are seven friends who study in Career Power at three different branches namely
Mukharjee Nagar, Dwarka, and Laxmi Nagar such that not less than two friends study in the same
branch. Each friend also has a different favourite subject viz. History, Civics, English, G.K., Reasoning,
Maths and Economics but not necessarily in the same order. Each of the students attends the classes of
his favourite subjects at different time
B attends his classes at 10 AM. G attends his classes at 7 AM. A likes Maths and studies in the
Mukharjee Nagar branch with only one other friend who likes G.K. I studies with two other friends. There
is only one hour gap between the time of class of the persons from Dwarka and the one whose favourite
subject is G.K. Both the friends who study with I like the same subjects, which can be either Reasoning or
English. D studies in the Dwarka branch with only one person and does not like civics. The one whose
favourite subject is Economics takes his class at 8 AM. I attends his class at 8 ‘O clock. The persons
whose favourite subjects are Economics and History do not have their classes at the same time. E
studies with only one friend. The one who likes history does not study in Mukharjee Nagar nor Dwarka
branch. E does not like G.K. subject. C does not like English, Reasoning or Civics. A and the one whose
favourite subject is Civics have their class at 11 AM. C attends his class after D.

Q1. Which combination represents E’s favourite subject and the branch in which he studies?
Civics and Laxmi Nagar
Economics and Mukharjee Nagar
Civics and Dwarka
History and Laxmi Nagar
Economics and Laxmi Nagar

Q2. I attends his class at?


11 AM
8 AM
10 AM
9 AM
None of these

Q3. Who amongst the following studies in the Laxmi Nagar branch?
G
C
E
D
Either D or B

Q4. Which of the following combinations is definitely correct?


I and Reasoning
G and English
C and G.K.
B and Reasoning
E and Economics
Q5. Which of the following subjects does G like?
Either Maths or G.K.
Either Reasoning or English
Either Reasoning or Civics
Either Reasoning or G.K.
Either Civics or Economics

PUZZLE-172
Directions (1-5): Study the following information carefully and answer the question given below:
Eight employee Q, R, S, T, W, X, Y and Z of the multinational company (MNC) will have to attend the
meeting in the months of August and December but not necessarily in the same order. In each month, the
meeting will conduct on dates 21st, 23th, 27th and 31st of the given month. Only one person will attend
the meeting on these given dates.
Q will attend the meeting on either 27th August or 27th December. There is only one person will attend
the meeting between Q and T. There are two persons will attend the meeting between the T and R. There
is one person will attend the meeting between the R and S. S will attend the meeting on 23th of any
month either December or August. Only one person will attend the meeting between S and W. There are
as many as the person will attend the meeting between W and Y as between Y and Z. X will not attend
the meeting on 21st of August.

Q1.Who among following will attend the meeting on 27th August?


Z
Q
R
Y
S

Q2. How many persons will attend the meeting between R and W?
One
Two
Three
More than three
None of these
Q3. Who among following will attend the meeting on 31st December?
R
Y
S
X
W

Q4. If T is related to Q and R is related to S, in the same way Y is related to?


X
T
Z
Q
R

Q5. X will attend the meeting on which of the following dates?


23th August
31st December
27th August
31st August
None of these
PUZZLE-173
Directions (1–5): Study the following information carefully to answer the given questions.
In Year 2016, seven persons viz. P, Q, R, S, T, V and W donated some money to a charitable trust on
different months of the year starting from January and ending on November. Some of them are females
and some of them are male. Their contribution,(in rupees) are – 5000, 500, 40, 1600, 80, 40 and 2500. V
made his donation in June. Two donations were made between the donations made by V and P. On the
first month of the year, a male made a contribution of 5000 rupees. S did not made his donation on a
month after the donation of Q. None of the given person made his contribution in a month which has 28
days. There are two such cases where the difference in the number of days of the months of two
successive donations is zero. In one such case, the months are consecutive months of the year. The
contribution of Rs 500 was made in November month. The sum of contributions of W and Q is equal to
that of S. Contribution by a female is always preceded and succeeded by the contribution of a male. T is
the only female who cannot be succeeded by any one. P is the only male which is followed by another
male. Only one donation is made between the donations made by T and S. The donation of the male who
donated his money on June is the perfect square of the donation made on August. The donation by T is
not made on the month immediately before or immediately after the month when V made his donation
(considering only those month in which the donations were made. R donated money immediately before
W. Only three persons made their contribution in a month which has 31 days.

Q1. As per the given arrangement which of the following combination represents only the people
who are females?
T, V
Q, P, T
V, W, R
P, T
W, T, S

Q2. As per the given arrangement which of the following person represent the one who was
donated money in between the W and S?
U
P
R
V
W

Q3. Who among the following is a female?


P
R
S
All of the Above
Other than those given as options

Q4. Which of the following combinations is correct as per the given arrangement?
January – P
February – W
April – Q
June – S
July – T
Q5. Who amongst the following is male who donated 2500 Rs?
R
V
U
W
T

PUZZLE-174
Directions (1-5): Study the following information carefully and answer the questions given below:
Eight persons A, B, C, D, E, F, G and H are sitting around a circular table facing the centre. They belong
to different states, viz Punjab, Kerela, Goa, Sikkim, Manipur, Nagaland, Bihar and Telengna, but not
necessarily in the same order. They also like different colours– Red, White and Blue. At least two persons
like one colour.
Only D and E like Red and D sits third to the right of E. A is from Manipur and likes White and sits on the
immediate right of E. F is from Sikkim and likes Blue. H likes the same colour as G. G is third to the left of
F. B sits opposite the person who is from Telengna. C sits exactly opposite the person who belongs to
Punjab. The persons who belongs to Nagaland and Goa like White. G is from Telengna. B is from Goa.
The persons who like White do not sits adjacent to each other. The one who is from Bihar does not like
White or Red. H is not from Nagaland.

Q1. Who among the following is from Kerela?


The one who likes Blue
D
The one who likes White
E
None of these
Q2. Who sits fifth to right of H?
F
C
B
G
A
Q3. E belong to which of the following state?
Nagaland
Kerela
Punjab
Bihar
None of these

Q4. Which of the following combinations is true?


G – White – Telengna
C – Blue – Kerela
D – Red – Punjab
A-Blue- Manipur
None is true

Q5. Which of the following statements is/are true?


G is from Telengna and likes White.
C is from Nagaland and likes White.
F does not like Blue.
H belongs to Kerela and likes Red.
None of these
PUZZLE-175
Eight groups A, B, C, D, E, F, G and H are going forrock climbing in different mountain
ranges, viz. Aravallis,Himalayas, Satpura, Andes, Alps, Rockies, Atlas andAppalachian but
not necessarily in the same order. Thesegroups are also going for rafting in different rivers,
viz.Spree, Hudson, Amazon, Tigris, Tiber, Nile, Danube andThames but not necessarily in
the same order.Group G is going neither to river Danube nor to riverHudson. Neither group
F nor group G is going to raft in theriver Tigris or the river Nile. Group E is going to
theAppalachian mountain range. Group C is going to the Andes mountain range and the
river Tiber. The one who is goingto raft in the Amazon is also going to the
Appalachianmountain range. Group A and B are going for rafting either in the Nile or
Thames. Group D does not want to raft in theTigris and F does not want to raft in the
Hudson. Thegroup which is climbing on the Aravallis is rafting in theDanube and the group
which is climbing on the Atlas israfting in the Hudson. A and G do not climb on the
Rockiesand the Satpura. The group which is climbing on theHimalayas is rafting in the
Thames. Group H and G climbeither on Alps or on the Satpura mountain range.
Name Mountain River
A Himalayas Thames
B Rockies Nile
C Andes Tiber
D Atlas Hudson
E Appalachian Amazon
F Aravallis Danube
G Alps Spree
H Satpura Tigris

PUZZLE-176
PK, SP, RJ, MP, DK, SK and AK are seven shopkeepers of different shops, viz Grocery,
Cosmetics, Clothes,Footwear, Gift items, Watches and Mobile phones. They go to
supermarkets, viz. Big Bazaar, Max, Reliance Trends andEasyDay only on Sunday but not
necessarily in the same order. At least one shopkeeper goes to one supermarket, but no
super market is visited by more than two shopkeepers.SK, who is a shopkeeper of Clothes,
goes alone to Reliance Trends.The one who is a shopkeeper of Footwear does not go to
Big Bazaar. Also, he never goes either with DK or with AK.MP goes to EasyDay with the
person who is the shopkeeper of Mobile phones.RJ goes to Max. AK is not a shopkeeper of
Mobile phones.The one who is the shopkeeper of Footwear goes to the market with the
person who is the shopkeeper of Grocery.The one who is the shopkeeper of Gift items goes
to EasyDay.PK is a shopkeeper of neither Cosmetics nor Mobile phones.The one who is the
shopkeeper of Watches goes to Big Bazaar neither with DK nor with MP.The person who is
the shopkeeper of Cosmetics goes to the supermarket with PK.
Shopkeeper Super Market Shop
PK Big BazaaR Watch
SP Max Grocery/Foodwear
RJ Max Foodwear/Grocery
MP EasyDay Gift Items
DK EasyDay Mobile phones
SK Reliance Trends Clothes
AK Big Bazaar Cosmetics

PUZZLE-177
There are eight actors – Amitabh, Akshay, Shahrukh, Hrithik, Priyanka, Anushka, Diya and
Sonakshi. Out of them four are males and remaining are females. Moreover, each of them
endorses a different brand, vizAirtel, Titan, Hero, Britannia, LUX, Rin, Nokia and Cadbury,
though not necessarily in the same order. All of them are sitting around a circular table,
facing the centre. The person who endorses LUX sits opposite Shahrukh. There are two
persons between Diya and Hrithik, who endorses Nokia. Priyanka, who does not endorse
Rin, sits second to the left of Amitabh, who endorses Airtel. The person opposite Diya
endorses Titan.Shahrukh is not an immediate neighbour of either Amitabh or Priyanka.
Anushka does not endorse LUX. Akshay sits second to the right of Hrithik. Sonakshi sits
third to the right of the person endorsing Titan and she endorses neither Britannia nor
Cadbury.Shahrukh endorses Britannia and the person sitting between Shahrukh and Hrithik
endorses Hero. No three females are sitting together.
PUZZLE-178
T - 20 World Cup is being organized by Bangladesh. Different teams are participating in T-
20 World Cup. There are some new teams which are also participating in this T-20 for the
first time. Seven new teams which are participating for the first time are - Afghanistan,
Nepal, UAE, Bangladesh, Netherlands, Hong Kong and Ireland. They will be playing three
practice matches at different places, namely Mirpur, Chittagong, Sylhet, Dhaka, Begra,
Khulna and Fatullah, but not necessarily in the same order, on different days of the week,
starting from Monday.
Afghanistan will be playing at Chittagong, but neither on Friday nor on Monday.
The host country will be playing on Thursday at Begra
There is a gap of one day between Afghanistan and Bangladesh match
Hong Kong will be playing the match before Ireland but after Netherlands at Sylhet
on Wednesday
Ireland will be playing on the last day of the week
Nepal will be playing at Fatullah
No match will be played at Mirpur on Monday
The match at Khulna will be played on the last day of the week
The match at Dhaka will be played on Tuesday
No match will be played at Fatullah on Tuesday and Friday
Afghanistan Chittagong Saturday
Bangladesh Begra Thursday
HongKong Sylhet Wednesday
Ireland Khulna Sunday
Nepal Fatullah Monday
UAE Mirpur Friday
Netherlands Dhaka Tuesday
PUZZLE-179
Six friends P, Q, R, S, T and U from six different areas, viz. Rohini, Indirapuram, Dwarka,
Kaushambi, Munirka and Vaishali, go for shopping in six different shopping complexes, viz.
Sahara, GIP, CSM, V3S, Shipra and Select Citywalk, but not necessarily in the same order.
Three different types of transport sare used by them, viz. Car, Bus and Metro, in such a
manner that two persons use the same type of transport but not necessarily in the same
order.
P uses car and lives in Vaishali but he does not go to CSM and Select Citywalk.
The person going to Shipra Complex uses car and the one going to GIP uses bus.
T uses the same type of transport as the person from Dwarka.
CSM Complex is chosen by the person who uses bus.
R is neither from Rohini nor from Munirka and he goes to Shipra Complex.
One of the persons who use Metro goes to Sahara Complex.
Q is from Kaushambi and goes to GIP. He does not use the same transport as S.
U is neither from Indirapuram nor from Munirka.
Select Citywalk and GIP Complex are not visited by the persons who use car.
The persons from Kaushambi and Rohini are using the same type of transport.

Person Transport Place Shopping Complex

P Car Vaishali V3S

Q Bus Kaushambi GIP

R Car Indirapuram Shipra

S Metro Dwarka Select Citywalk/Sahara

T Metro Munirka Sahara/Select Citywalk

U Bus Rohini CSM


PUZZLE-180
A, C, E, I, O, N, P and S are eight persons working in an MNC. They are sitting around a
circular table in a meeting but not necessarily in the same order. All of them hold a different
post, viz President, MD, GM, CA, HR, Vice President, Chairman and Vice Chairman, but not
necessarily in the same order. C sits third to the right of the Vice President. Only two
persons sit between Vice President and S. The GM and the MD are immediate neighbours.
Neither C nor S is a GM or an MD. The GM is not an immediate neighbour of the Vice
President. The CA sits third to the right of O, who is not the GM. N sits on the immediate
right of the Chairman. The Vice Chairman sits second to the left of A, who is not an
immediate neighbour of S. The Vice Chairman is an immediate neighbour of both the HR
and the CA. C is not the Chairman. I is not an immediate neighbour of C. E is not an
immediate neighbour of the Vice Chairman.
PUZZLE-181
Ten persons are sitting in two rows containing five persons each. In row 1, A, B, C, D and E
are sitting and all of them facing south. In row 2, P, Q, R, S and T are sitting and all of them
are facing north. In the given seating arrangement each member sitting in a row faces
another member of the other row. Moreover, each of them belongs to a different country
Australia, Brazil, Bulgaria, Belgium, Japan, India, Romania, Indonesia, Iran and Russia, but
not necessarily in the same order. There are only two persons sitting between the
Australian, who sits at an extreme end, and E. P, who sits in the middle of the row, is not an
immediate neighbour of Q, who is not a Russian. T is sitting at an extreme end. E, a
Japanese, sits on the immediate right of the person from Brazil and faces the immediate
neighbour of Q. B is not sitting at the extreme left end of the row. P is not from Iran. There is
only one person sitting between C and D, who is from Australia. S is an Indian and is an
immediate neighbour of the Russian and doesn't face the person from Bulgaria. R, who is
from Belgium, is an immediate neighbour of the person from Iran who is facing an
immediate neighbour of the Japanese. There are two persons between the Romanian and
the Bulgarian. B is not from Romania. T is not from Indonesia.
PUZZLE-182
A, B, C, D, E, F, G and H are bachelor courses. They study in 1st year, 2nd year, and 3rd
year. Not more than three students are there in one year. Each of them has a favourite
subject, viz Geography, Physics, English, Marathi, Mathematics, Chemistry, Biology and
Economics, but not necessarily in the same order. B is not in the second year. E and A are
students of in same year but not with B. C and F are students of the same year. The
students who study in the first year do not like Biology and Mathematics. F likes Physics. D
likes Chemistry and is in final year only with H. C does not like Geography. Marathi is the
favourite subject of A. G does not like Biology. The one who studies in the final year likes
English. (Note: Third year is the final year.)
PERSON SUBJECT YEAR
A Marathi II
B Geography I
C Economics I
D Chemistry III
E Biology II
F Physics I
G Maths II
H English III

PUZZLE-183
T,U, V,W, X, Y and Z live on a seven-storey building but not necessarily in the same
order. The lowermost floor of the building is numbered 1, and the topmost floor is
numbered 7. each of them also likes a watch of a different company, viz, Sonata, 'Titan,
Casio, Rado, Fastrack, Optima and Rolex.
T lives on an Odd-numbered floor but not on floor Number 3. The one who likes Fastrack
lives immediately above T. Only two persons live between W and the one who likes
Fastrack. the one who likes Titan lives on one of the odd-numbered floors but above W.
Only three persons live between V and the one who likes Titan. The one who likes Casio
lives immediately above V. The one who likes Sonata lives immediately above the one
who likes Rolex. Z lives on an odd-numbered floor. Only one person lives between U and
X. U lives on one of the floors above X. Neither V nor T likes Rado. X does not like Casio.

Floor Person Watch


7 Z Rado
6 U Fastrack
5 T Titan
4 X Sonata
3 W Rolex
2 Y Casio
1 V Optima
PUZZLE-184
Eight Friends — Romil, Rakesh, Mukesh, Rohit, Rahul, Anupam, Neeraj and Anil — are
seated around a circular table, but not necessarily in the same order. Four of them are
facing the centre and the others are facing outward. They belong to eight different cities —
Bhopal, Patna, Kolkata, Delhi, Gwalior, Ranchi, Chennai and Rajkot but not necessarily in
the same order. Anupam faces the centre and sits third to the right of Rakesh. Rohit
belongs to Kolkata and faces the person who belongs to Ranchi. Neeraj sits third to the
right of Mukesh, who lives in Bhopal. The persons who belong to Delhi and Gwalior are
facing the same direction (inward or outward). Rahul is sitting between the person who
belongs to Kolkata and the one from Rajkot respectively. Romil belongs to Gwalior and
Rakesh belongs to Patna. The person who belongs to Chennai is facing outward and is the
immediate neighbour of the person who belongs to Rajkot. Anil is the immediate neighbour
of the ones who belong to Gwalior and Chennai. Rahul is on the immediate left of Rohit.
Anil sits third to the right of Rahul.
PUZZLE-185
Nine persons P, Q, R, S, T, U, V, W and X live in a building but not necessarily in the same
order. There are nine floors in that building and only one person lives on each floor. Each of
them has a different vehicle — WagonR, Alto, Hyundai, Corola, Fiat, Maruti, Santro, i20 and
i10 but not necessarily in the same order. The ground floor is numbered 1 and the floor
above it is numbered 2 and so on. The topmost floor is number 9. W has Corola and he
lives on an even-numbered floor. P lives on an even-numbered floor that is below the floor
on which W lives. The person who has i20 lives on the fourth floor. T lives on the 2nd floor
and has Hyundai. The person having i10 lives on the 3rd floor. P doesn't have Maruti car.
There are two floors between the floors on which the persons having Santro and Corola
live. R has Alto. There are three floors between R and V. S lives on the floor immediately
above the floor on which X lives. There is one floor between the floors on which U and V
live. U doesn't have i10. The person having WagonR lives on the topmost floor. U doesn't
live on the ground floor.
Person Floor vehicle

Q 9 Wagonr

W 8 Corola

U 7 Maruti

P 6 Fiat

V 5 Santro

S 4 i20

X 3 i10

T 2 Hyundai

R 1 Alto
PUZZLE-186
Directions (1-5): Study the following information carefully and answer the given questions.
There are seven bollywood actors named Aamir Khan, Akshay Kumar, Ajay Devgan,
Shahid Kapoor, Hrithik Roshan, Salman Khan and Shah Rukh Khan. They are born in a
different month of the same year from January to July, but not necessarily in the same
order. No two people are born in the same month. They also acted in different movies
among Dilwale, Sultan, Dangal, Rustom, Drishyam, Krrish and Dhoom. Akshay Kumar has
his birthday in a month which has 30 days.The ones who have their birthday in the month
immediately preceding & succeeding that of Akshay Kumar's acted in Drishyam and Krrish
respectively.Ajay Devgan has his birthday in the month just before that of Akshay
Kumar.Salman Khan has his birthday in a month that has more than 30 days.Dangal is not
acted by Salman Khan or Shahid Kapoor.Hrithik Roshanacted in Sultan. There is only one
person's birthday between Hrithik Roshan and the one who acted in Dilwale.The persons
who acted in Rustom or Dhoom do not have birthdays in successive months. Akshay
Kumarnot acted in Rustom or Dhoom.There are three persons' birthday between Aamir
Khan and the one who acted in Dangal. Aamir Khan is not born in February.The one who
acted in Dhoom has his birthday before the one who acted in Rustom.
1.The actor who acted in Drishyam born in which of the following month?
April
March
May
July
January

2.Aamir Khanacted in which of the following movies?


Dangal
Dilwale
Krrish
Rustom
Sultan

3.How many actors born between Shahid Kapoor and Hrithik Roshan?
Three
Four
Five
Two
None
4.Four of the following persons form a group, which of the following person does not belong
to that group?
Shah Rukh Khan
Salman Khan
Aamir Khan
Shahid Kapoor
Ajay Devgan

5.Which of the following combinations is true?


July – Hrithik Roshan - Sultan
April – Ajay Devgan - Dilwale
May - Aamir Khan - Rustom
January – Salman Khan - Dangal
February – Shahid Kapoor – Dhoom
PUZZLE-187
Directions (6-10): Study the following information carefully and answer the given
questions.
Eight players are stayed in a building from bottom to top floor numbered from 1-8 each
player from a different international cricket team; Australia, South Africa, England, West
Indies, Sri Lanka, Bangladesh, New Zealand and Afghanistan not necessarily in the same
order. Theyplay for different IPL teams; Mumbai Indians, Delhi Daredevils, Royal
Challengers Bangalore, Gujarat Lions, Rising Pune Supergiants, Kolkata Knight Riders,
Sunrisers Hyderabad and Kings XI Punjab not necessarily in the same order. Each
international player plays for one IPL team only. The arrangement is based on the
following rules:
The player from Sri Lankateam is stayed at an even numbered floor
There are 3 players between the player from Sri Lankateam and the player play
for Kings XI Punjabteam
The player from West Indiesteam is either at the top or at the bottom floor
The player who stayed in floor number 2 plays for Mumbai Indiansteam and he is not
from Sri Lankateam
The player plays for Rising Pune Supergiantsteam is stayed between the player
from New Zealandteam and the player plays for Gujarat Lionsteam
The player from Australiateam is stayed 1floorabove the player from Englandteam
There are 4 players between the player who plays for Delhi Daredevilsteam and the
player from New Zealandteam
There are 3 players between the player who plays for Rising Pune Supergiantsteam
and the player who plays for Royal Challengers Bangaloreteam
The player from Afghanistanteam is stayed between the player who plays for Sunrisers
Hyderabadteam and the player from Bangladeshteam
The player from New Zealandteam does not play for Sunrisers Hyderabadteam.

6.The player from Sri Lanka team is stay in which of the following floor?
8th floor
6th floor
4th floor
2nd floor
None of these

7.How many players are stayed between the player from Australiateam and the player
from West Indies team?
Four
Five
Three
Six
Two
8. Which of the following international team player stayed in 3rd number floor?
Afghanistan
West Indies
Bangladesh
Australia
South Africa
9.Four of the following five are alike in a certain way and hence they form a group. Which one
of the following does not belong to that group?
Delhi Daredevils
Royal Challengers Bangalore
Sunrisers Hyderabad
Rising Pune Supergiants
Kings XI Punjab

10.Which of the following statement is true?


5 – Australia - Rising Pune Supergiants
4 - South Africa -Kings XI Punjab
2 - New Zealand -Mumbai Indians
3 – Bangladesh - Royal Challengers Bangalore
1 - West Indies - Kolkata Knight Riders
PUZZLE-188
Directions (Q.1-5): Study the given information carefully to answer the given question A
show case fixed against the wall of a showroom has six compartments. Each compartment
contains a chain of different e-books vizReasoning, Quantitative Aptitude, English, Computer,
Banking and Current Affairs, but not necessarily in the same order. The Reasoninge-book has
been kept neither on the top nor on the bottom. Only one e-book has been kept between the
Bankinge-book and the Englishe-book. There are two e-books between the Current Affairse-
book and the Reasoninge-book. The Computere-book is just above the Bankinge-book. The
Quantitative Aptitudee-book is below the Reasoninge-book.
1.What is the position of the Quantitative Aptitude e-book when counted from the bottom?
First
Second
Third
Fourth
Cannot be determined

2. What is the position of the Bankinge-book with respect to Current Affairse-book?


Four places above
Four places below
Two places below
Either option a or c
None of these

How many e-books are there between the Current Affairse-book and the
Quantitative Aptitudee-book?
None
Three
Four
Either option a or c
None of these

4.How many e-books are there between the Reasoninge-book and the
Quantitative Aptitudee-book?
None
One
Two
Cannot be determined
None of these
5.How many rearrangements are possible based on the given information?
Two
One
Three
Four
None of these

PUZZLE-189

Directions (Q. 6-10): Study the following information carefully and answer the
given questions.
There are seven friends, namely Deepa, Archana, Lakshmi, Divya, Nisha, Shivya and Geeta.
They work in the different banks, ieAxis Bank, HDFC Bank and ICICI Bank. There are at
least two persons in the same bank. Moreover, each of these seven friends has a different
salary as well. No two of them have the same salary. In addition to this, some information is
given as follows.
I. Shivya works in HDFC Bank and earns more than only Geeta.
II. Deepa does not work with Archana and Geeta.
Archana stands 4th from the top when arranged in descending order of their salaries.
IV. Nisha is neither from Axis Bank not from the bank where Divya works.
V. Geeta and Lakshmi are in the same bank.
VI. Divya works only with Archana but not in Axis Bank and she earns less than only
Lakshmi and Nisha.
VII. At most three people can work in the same bank.

6.Among the following persons, Deepa works with whom?


Divya
Lakshmi
Nisha
Cannot be determined
None of these

7.Who among the following work in the HDFC Bank?


Shivya, Nisha and Deepa
Shivya, Divya and Archana
Lakshmi, Geeta and Shivya
Shivya, Deepa and Lakshmi
None of these

8.Which of the following is definitely true about Lakshmi?


She works in the HDFC Bank.
She earns the most among all the friends.
She works with Geeta in the ICICI Bank.
All are true
None of these

9.Who amongst them earns the most?


Lakshmi
Nisha
Geeta
Cannot be determined
None of these

10.Which of the following statements is/are false?


Geeta earns more than only Deepa
Nisha works in the ICICI Bank
The Axis Bank has the maximum number of persons working
All are false
None of these
PUZZLE-190
Directions (1-5): Study the following information carefully and answer the
given questions
Seven friends Harish, Laxman, Vinoth, Sanjay, Madhan, Rajesh and Ganeshlikes
different days Sunday, Monday, Tuesday, Wednesday, Thursday, Friday and Saturday.
But not necessarily in the same order. Everybody has different favouritemonths namely
January, February, March, April, May, June and July but not necessarily in the same
order.
I. Harish likes Monday and Sanjay likes
Tuesday, II. Ganesh'sfavouritemonth is July.
III. Laxmanlikes Sunday and his favouritemonth is January.
IV. Rajesh likes Wednesday and Vinoth'sfavouritemonth is March.
V. Madhandoes not like Friday and Rajesh favouritemonth is not April.
VI. The person,whose favourite month is May, likes Thursday
VII. Sanjay and Rajesh favourite month is not February.
VIII. The person who likes Saturday his favouritemonth is July.

Which of the following person likes


Friday? Vinoth
Rajesh
Harish
Sanjay
Laxman

2.Ganesh like which of the following days?


Sunday
Saturday
Wednesday
Tuesday
Monday
Madhan likes which of the following month?
March
April May

June

February

Four among the following form a group in a certain way. Which of the following
does not belong to Group?
Laxman
Ganesh
Vinoth
Madhan
Sanjay

Which of the following statement is true? Whose


favourite month is February, likes Sunday The one
who likes Friday, his favourite month is May Madhan
Likes Saturday
Rajesh favourite month is June
None of these
Name Days Month
Harish Monday February
Laxman Sunday January
Vinoth Friday March
Sanjay Tuesday April
Madhan Thursday May
Rajesh Wednesday June
Ganesh Saturday July
PUZZLE-191

Directions (6-10): Study the following information carefully and answer the
given questions
Eight People – Indran, Kavin, Mani, Gandhi, Ajay, Prasanth, Mishra and Varun live in
eight different floors of building (but not necessarily in the same order). The lowermost
floor of the building is numbered one, the one above that is numbered two, and so on till
the topmost floor is numbered eight. Each one of them also makes awareness about
different days, namely National Technology Day, International Nurses Day, World
Migratory Day, International Day of the Family, World Information Society Day, World
Museum Day, World Metrological Day and Anti-Terrorism Day (but not necessarily in
the same order). Only one person lives between Kavin and the one who makes
awareness about World Information Society Day. Prasanth lives an odd numbered floor
above the floor numbered four. Only three people live between Gandhi and the one
who makes awareness about World Museum Day. Mani lives on one of the odd
numbered floors above the one who makes awareness about World Museum Day. The
one who makes awareness about World Metrological Day lives immediately above
Mishra, Mishramakes awareness about neither World Museum Day nor National
Technology Day. Ajay does not makes awareness about World Metrological Day. Only
three people live between Mishra and Indran. The one who makes awareness
aboutAnti-Terrorism Day lives immediately above the one who makes awareness about
International Nurses Day, but not on the topmost floor. Only one person lives between
Prasanth and the one who makes awareness about World Migratory Day. The number
of people living above Prasanth is same as the number of people living between
Prasanth and Gandhi. Only two people live between Mani and the one who makes
awareness about National Technology Day.

On which of the following floor does Varun


stay? 3rd
5th
7th
2nd
1st
7.How many floors are there between the floor on which Prasanth stays and
the floor whomakes awareness about International Nurses Day?
None
Three
Two
One
More than three

Who amongst the following stays on the top–most floor?


Who makes awareness about International Day of the Family
Mishra
Kavin
Both a and c

None of these

Ajaymakes awareness about which of the following days?


World Information Society Day
World Museum Day
National Technology Day
International Nurses Day
World Migratory Day

Which of the following statement is true as per the given information?


Varun stays on a floor immediately below the floor on which Mishra stays
Mani stays on a floor immediately above the floor on who makes awareness
about World Museum Day
Indran stays on sixth floor and makes awareness about International Day of
the Family
There are two persons live between Ajay and Kavin
None is true
Floor Person Days

8 Kavin International Day of the Family

7 Mani World Migratory Day

6 Indran World Information Society Day

5 Prasanth World Museum Day

4 Ajay National Technology Day

3 Varun World Metrological Day

2 Mishra Anti-Terrorism Day

1 Gandhi International Nurses Day


PUZZLE-192
Directions (1-5): Study the following information carefully and answer the
given questions.
There are seven national parks namely Jim Corbett, Ranthambore, Kaziranga,
Periyar, Bandipur, Manas and Satpura located in different cities such as Chennai,
Mumbai, Kochi, Hyderabad, Bangalore, Patna & Kolkata, not necessarily in the
same order. Different animals such as, Lion, Tiger, Elephant and Deer were
presented in the park. Depends on the length of the Parks it was listed from 1 to
7 positions, from top to bottom. The following information is known about them.
Not more than 3 Parks has same animals.
There's only one park has Deer which belongs to Bangalore and listed in bottom 3
positions and is not Kaziranga Park
The one belongs to Chennai listed just ahead of Ranthambore Park, which does not
belong to Bangalore.
The park in which Tiger presented is listed 1st.
The Parks which listed in 3rd and 4th have same animals and it is not Lion.
The park located in Mumbai listed in last and park located in Patna listed as 4th
and none of them are has Tiger.
Manas Park have Lion animal and is not in bottom 3.
Periyar Park belongs to Hyderabad and Satpura Park belongs to Chennai, both
are not having lion animal and in the list there are two Parks between them.
The Parks from Kolkata and Mumbai are having same animals.
Bandipur Park not located in Kochi, Mumbai or Bangalore.

1). Which of the following animal presented in Bandipur Park?


Lion
Elephant
Deer
Either Tiger or Lion
Either Tiger or Elephant
2). Which of the following park listed on 3rd place from the top?
Jim Corbett Park
Kaziranga Park
Periyar Park
Ranthambore Park
Satpura Park

3). Manas park located in______________


Kolkata
Mumbai
Bangalore
Patna
Kochi

4). If ‘Manas’ is related to ‘Ranthambore ’, ‘Periyar ‘ is related to ‘Bandipur’ , then


which following park is ‘Kaziranga’ related to?
Bandipur
Ranthambore
Jim Corbett
Satpura
Manas

5). Which of the following combinations is true?


3 – Periyar – Hyderabad - Lion
1 – Manas – Kochi -Tiger
2 - Jim Corbett – Bangalore - Deer
7 – Ranthambore – Mumbai - Tiger
4 – Bandipur – Patna – Elephant
PUZZLE-193
Directions (6-10): Study the following information carefully and answer the
given questions.
Six friends - Harish, Mohan, Suresh, Vikram, Ram, and Gopal went to six different
cities namely Mumbai, Bangalore, Hyderabad, Delhi, Chennai and Patna (not
necessarily in the same order) to attend different probationary officers exam. They
exams are IBPS PO, SBI PO, BOB PO, Syndicate Bank PO, Indian Bank PO and
Dena Bank PO. They attend exams from Monday to Saturday (not necessarily in
the same order).
Harish attends exam in Delhi and he wrote either IBPS PO or Dena Bank PO.
Ram did not wrote Dena Bank PO exam
Suresh attends exam on Tuesday and he is not wrote IBPS PO or BOB PO exam.
The person who wrote the Syndicate Bank PO exam wrote on Saturday and neither
in Mumbai nor in Bangalore.
Gopal wrote the SBI PO exam in Patna but not on Wednesday.
Ram attends the exam at Hyderabad and wrote the exam one day before
Gopal On first day of the week, Mohan attends the exam in Bangalore.
The person who attends the exam on Thursday is not wrote BOB PO or IBPS
PO exam.
The person who attends the exam on Thursday is not Gopal.
6). Who wrote the exam in Chennai?
Mohan
Suresh
Ram
Vikram
Harish

7). Harish wrote the exam on ___________


Wednesday
Friday
Sunday
Thursday
Monday

8). Which of the following city conduct Indian Bank PO exam?


Bangalore
Hyderabad
Delhi
Chennai
Mumbai

9). Which of the following exam held on Monday?


SBI PO
IBPS PO
Dena Bank PO
Indian Bank PO
BOB PO
10). Which of the following combinations is true?
a) a) Ram - Indian Bank PO - Bangalore - Thursday
Suresh - Dena Bank PO – Mumbai - Tuesday
Mohan –Syndicate Bank PO – Hyderabad - Monday
Harish - IBPS PO - Delhi - Friday
None of these

PUZZLE-194
Directions (1-5): Study the following information carefully and answer the
given questions.
Eight persons from different states such as Maharashtra, Punjab, Rajasthan,
Haryana, Kerala, Tamil Nadu, Karnataka and Bihar eating 8 different sweets such
as Badam Halwa, Besan laddu, Gulab jamun, Jalebi, Rasgulla, Rava laddu ,Mysore
pak and Kaju katli . And each of them having a mobile in the brand names of
Samsung J2, Samsung J3, Samsung J5 and Samsung J7
The person who is from Punjab eating Kaju katli as a sweet
The person who is eating Mysore pak having Samsung J7 mobile
The third floor stayed person eating Besan laddu and having a mobile in the brand
name of Samsung J2.
Gulab jamun is eaten by the person who is having Samsung J3 mobile.
Minimum of three persons stayed below of Bihar state person. And the person
who belongs to Haryana not stayed in the odd numbered floor.
The mobile’s brand name Samsung J3 and Samsung J7 having same number of
state-men.
Rajasthan state person stayed two floor below from the person who is belongs
to Punjab.
The top floor is not occupied by the person from Karnataka.
The Samsung J5 owned person eating Badam Halwa.
The persons from Haryana and Punjab having same mobile brand and both of them
not having Samsung J3 mobile and also three persons stayed in between them.

The persons who are eating Jalebi sweet and Besan laddu are having same
mobile brand with the one who is eating Rasgulla.
The Samsung J5 owned person stayed in fourth floor. There are four
persons stayed in between of Karnataka state and Haryana state. The lowest
floor is occupied by the Kerala state person.
Only three persons having one kind of mobile brand name and the person
from Karnataka who is eating Rasgulla is one of them.
The person from Tamil Nadu and the person who eating Rasgulla are not stayed
in the even numbered floor
Only one person having Samsung J5 mobile
The Maharashtran who likes to eat Rava laddu stayed in even numbered floor and
is having same mobile brand name with the person, who is eating Gulab jamun
also stayed in first floor.
Samsung J7 mobile is owned by the person who is from Haryana.
The person who eating Kaju katli stayed above the floor of the person who is
eating Mysore pak.
The person from Tamil Nadu who is eating Besan laddu having same mobile
brand with the state person from Bihar
1). How many persons have Samsung J2 mobiles?
One
Two
Three
Four
Five

2). Who lives in the sixth floor?


The one who is from Punjab
The one who is from Haryana
The one who is from Karnataka
The one who is from Maharashtra
The one who is from Bihar

3). Rasgulla sweet eats by which of the following state person?


Rajasthan
Punjab
Haryana
Maharashtra
Karnataka

4). Which of the following person sits exactly between who are eating Badam Halwa
and Kaju katli?
The one who eats Besan laddu sweet
The one who eats Mysore pak sweet
The one who eats Rava laddu sweet
The one who eats Jalebi sweet
The one who eats Gulab jamun sweet
5). Which of the following combinations is true?
2 – Gulab jamun – Samsung J7 - Kerala
4 - Badam Halwa - Samsung J5 - Rajasthan
8 – Badam Halwa – Samsung J3 - Kerala
7 - Besan laddu – Samsung J2 - Maharashtra
None of these

PUZZLE-195
Directions (6-10): Study the following information carefully and answer the given
questions.
Eight friends Prakash, Arun, Dinesh, Gupta, Vinoth, Ramesh, Karthik and Anand
live on eight different floors of a building but not necessarily in the same order. The
lowermost floor of the building is numbered 1 and the topmost floor of the building is
numbered 8. Each of them likes different months viz, January, February, March,
April, May, June, July and August but not necessarily in the same order.
The one who likes February lives on an even-numbered floor but not on the topmost
floor
Only one person lives between Ramesh and the one who likes May.
Only two persons live between Ramesh and the one who likes
February. Neither Vinoth nor Dinesh lives on the first floor.
Only one person lives between Dinesh and the one who likes March.
Prakash lives just above Ramesh. Only two persons live between Vinoth and
Prakash.
The one who likes May does not live on floor number one, Arun lives on an even-
numbered floor and just above Dinesh.
The one who likes April lives on an even numbered floor and lives just above
the person who likes August
Dinesh does not like May or August. Only two persons live between the one
who likes June and the one who likes January, Gupta does not like July.
The one who likes June does not live on an odd-numbered floor. Anand lives
just below the one who likes August.

6). How many persons are between the one who likes April and the one who stays
on 4th floor?
One
Two
Three
Four
Five

7). Karthik likes which of the following month?


July
March
April
August
January
8). Who lives on fifth floor?
Arun
Vinoth
The one who likes April
The one who likes May
Both option b and d

9). Which of the following combinations is true?


1 – Gupta - July
4 – Vinoth - February
3 – Ramesh - August
6 – Anand - June
7 – Dinesh - January

10). Four of the following five are alike in a certain way and hence they form
a group. Which one of the following does not belong to that group?
Prakash
Dinesh
Anand
Arun
Karthik
PUZZLE-196
Directions (1-5): Study the following information carefully and answer the
given questions.
Six friends Ashok, Shivya, Baskar, Lalitha, Gayathri and Ritish stay on a 6 floor
building with the ground floor numbered as floor number 1 and so on. All the
friends have visa for different countries such as; Australia, Russia, USA, Canada,
Kuwait and UK not necessarily in the same order. They also like different Months
via; March, May, June, September, October and December not necessarily in the
same order. The arrangement is based on the following rules:
The person on the top floor having visa for countries either UK or USA
The one who likes May lives 2 floors below the one who has Visa for USA
Gayathri likes December and lives on an even numbered floor
One who have visa for USA country likes either September or
June Baskar lives 3 floors below Shivya
One who lives on 2nd floor has the visa for Canada country and likes
neither December nor May
Lalitha’s visa country is Kuwait and she lives 1 floor above
Baskar Baskar does not like Canada
One who has the visa of Australia country likes October
One who likes September lives on an odd numbered floor

Ashok likes June and stays just above the person who likes
September One who has Kuwait country visa does not like September.

1). Who lives in the second floor?


The one who likes March
Ritish
The one who have Canada country visa
All of the above
None of these
2). Which of the following country visa and Month like by Baskar?
Russia - June
Australia - October
USA - December
UK - May
Russia - September

3). Which of the following person like May?


a) Shivya b) Ritish c) Baskar d) Ashok e) Lalitha

4). The one who likes June have which of the following country visa?
a) UK b) USA c) Kuwait d) Russia e) Australia

5). Which of the following combinations is true?


3 – Lalitha – Kuwait - December
1 - Baskar - Canada- October
5 – Shivya – USA - September
6 – Ashok –Kuwait- June
None of these
PUZZLE-197
Directions (6-10): Study the following information carefully and answer the given
questions.
Ritaj, Vaishnavi, Divya, Hema, Subha, Shivani, Bhavani, Prathiba, Nithya and Jaya
have some Debit cards and credit cards. Both kinds of cards are 1-10 in number.
No 2 persons have the same number of same cards. It is also known that:
Vaishnavi has 6 debit cards more than Bhavani.
Vaishnavi has 10 cards in total.
Jaya has 3 credit cards.
The total number of credit cards owned by Divya and Nithya is equal to the number of
credit cards that Prathiba has. Divya has less number of credit cards than Nithya.

Only Prathiba has 18 cards in total such that the number of credit cards he has
is more than his debit cards.
Only Divya has equal number of debit card and credit cards.
Only Bhavani has credit cards twice than the debit cards.
Ritaj has 9 cards in total. He has more credit cards than debit cards.
The number of debit cards owned by Hema is equal to the number of credit
cards owned by Subha.
Subha has 8 cards in total.
Subha and Shivani have 16 credit cards in total.
Nithya and Jaya have 16 debit cards in total.

6). Who among the following have 10 credit cards?


a) Divya b) Prathiba c) Subha d) Nithya e) Hema

7). Who among the following have 3 debit cards?


a) Prathiba b) Jaya c) Bhavani d) Shivani e) Vaishnavi

8). What is the sum of total number of cards owned by Hema and Nithya
together? a) 25 b) 20 c) 17 d) 22 e) 28
9). If ‘Ritaj’ is related to ‘Hema’, ‘Subha’ is related to ‘Vaishnavi’, then which
following person is ‘Bhavani’ related to?
Divya
Hema
Shivani
Ritaj
Jaya

10). Which of the following combinations is true?


Bhavani – 6 debit cards – 3 credit cards
Jaya - 10 debit cards – 8 credit cards
Vaishnavi – 1 debit card – 9 credit cards
Hema – 7 debit cards – 4 credit cards
None of these
PUZZLE-198
Direction (Q.1-5): Study the following information carefully and answer the
questions given below:
Eight persons P, Q, R, S, T, V and W are playing a game of musical chairs and
facing away from the centre. Each person knows a different language, viz Marathi,
Odia, Tamil, Punjabi, Bengali, Hindi, Telugu and English, but not necessarily in
the same order.
Neither S nor T speaks English.
R speaks Hindi and sits third to the left of T.
Those who speak Tamil and Punjabi are immediate neighbours of each other.
Neither the one who speaks Punjabi nor the one who speaks Tamil is an
immediate neighbour of either T or R.
The one who speaks Punjabi sits on the immediate left of Q, who speaks Odia.
Those who speak Telugu and English are immediate neighbours of each other.
P speaks Telugu and is not an immediate neighbour of the one who speaks Hindi.
V sits third to the left of the one who speaks English.
Only U sits between the ones who speak Punjabi and Marathi.

1). Who among the following speaks English?


R
W
S
V
None of these

2). How many persons are there between those who speak Punjabi and
Telugu respectively?
a) None b) One c) Three d) Four e) None of these
3). Who among the following is sitting third to the left of the one who speaks Tamil?
The one who speaks Punjabi
The one who speaks English
The one who speaks Bengali
The one who speaks Telugu
None of these

4). The one who sits second to the right of W speaks which of the following
languages?
Marathi
Bengali
Telugu
Hindi
Odia

5). Who among the following sits opposite the one who speaks Odia?
P
The one who speaks Bengali
The one who speaks English
V
None of these
PUZZLE-199
Directions (Q. 6-10): Study the following information carefully and answer the
questions given below:
Ten persons are sitting in two parallel rows containing five persons in each
row. In row 1— D, E, F, G, and H are sitting and all are facing north. In row 2 — V,
W, X, Y and Z are sitting and all are facing south. Each person also likes a
different TV channel, viz StarPlus, Life OK, Sony, Star Gold, Zee TV, Colors, ETV,
SAB TV, &TV and DD, but not necessarily in the same order.

H sits in the middle of the row and faces the one who likes Zee TV. Only one
person sits between Y and the one who likes Zee TV. Y faces one of the immediate
neighbours of the one who likes Colors. Only two persons sit between E and the one
who likes Colors. E faces the one who likes SAB TV. V faces one of the immediate
neighbours of F. V does not like SAB TV. G is an immediate neighbour of F.

The one who likes Star Gold is an immediate neighbour of V. Z is not an


immediate neighbour of V and faces the One who likes Life OK. X faces the one who
likes &TV. The one who likes DD faces north. V does not face the one who likes

The one who likes Star Plus sits on the immediate right of the one who likes ETV. Y
does not like Star Plus. The one who faces F does not like ETV and G faces Y.

6). Which of the following pairs represent the people sitting at the extreme ends of
the rows 2?
XG
YZ
WF
VD
EV
7). Which of the following TV channels does Z like?
Star Plus
Colors
Zee TV
ETV
SAB TV

8). Which of the following statements is definitely true?


F faces X.
W likes SAB TV
W is an immediate neighbour of V.
The one who likes &TV is an immediate neighbour Of V.
The one who likes, Sony faces X.

9). Who among the following likes Star Gold?


Y
V
Z
X
W

10). Who among the following is/are the immediate(s) neighbour of the one who
likes &TV?
The one Who likes Sony and G
The one who likes ETV and H
W and E
The one who likes Life OK and H.
Z
PUZZLE-200
Directions (Q. 1-5): Study the following information carefully and answer the
given questions.
Ten persons Shami, Kushak, Nanda, Pranav, Akil, Dileep, Jayanth, Ganapati,
Rohan and Vendan are sitting in two rows with five persons in each row. The
persons in row one are facing south and the persons in row two are facing north.
Each person in row one faces a person from the other row. All of them have a
watch of different companies, viz Maxima, Sonata, Matrix, Fossil, Fastrack, Casio,
Timex, Citizen, Titan and Espoir, but not necessarily in the same order.
The persons who like Fastrack and Casio sit opposite each other.
Dileep sits opposite to Shami, who likes Maxima.
The one who likes Sonata sits opposite the one who likes Citizen.
Vendan is not facing north but sits third to the left of Jayanth, who likes
Sonata. There is only one person between Kushak and Nanda.
Akil sits at one of the ends of the row and likes Casio.
The one who likes Citizen is on the immediate right of Pranav, who does not
like Espoir.
The persons who like Matrix and Fossil respectively are not facing north.
Nanda likes Timex. The one who likes Fossil sits opposite the one who is second to
the right of Kushak.
Rohan does not like Espoir. Akil sits opposite the one who sits second to the left
of the one who likes Matrix.

1). Who have Titan watch?


Akil
Kushak
Pranav
Dileep
Jayanth

2). Who sits opposite to Kushak?


The one who have Maxima watch
The one who have Matrix watch
Ganapati
The one who have Fastrack watch
Jayanth

3). Who sits opposite to the one who sits second to the left of Akil?
The one who have Matrix watch
The one who have Fossil watch
The one who have Titan watch
The one who have Sonata watch
None of these
4). Who sits at the right end in Row II?
Ganapati
Vendan
Jayanth
Akil
Pranav

5). Who among the following is the immediate neighbour of the one who
have Maxima watch?
The one who have Titan watch
Nanda
Rohan
The one who have Sonata watch
None of these
PUZZLE-201
Directions (Q. 6-10): Study the following information carefully and answer the given
questions.

Ten Friends are sitting in two parallel rows of six seats each. One seat is vacant in
each row. Bhavya, Janvi, Hemanika, Preeti and Dharini are sitting in row-1 facing
south. Veni, Radha, Charmy, Nisha and Usha are facing north. Each likes a
different brand of Soaps i.e. Hamam, Dove, Lux, Liril, Vivel, Pears, Nivea, Mysore
Sandal, Medimix and Margo. Nisha sits third to the right of Charmy and likes Liril.
Only two people sit between Radha and the vacant seat. Radha does not like Vivel
or Lux Soap. Dharini is not an immediate neighbour of Hemanika. Janvi likes Margo.
The one who likes Lux Soap faces the one who likes Mysore Sandal. The one who
likes Lux sits opposite to the one who sits third right of the person who sits opposite
to Nisha. Hemanika is not an immediate neighbour of Preeti. Usha, who likes
neither Vivel nor Pears, does not face the vacant seat. Neither Nisha nor Charmy
sits at any of the extreme ends of the row. Preeti faces Charmy. Vacant seats are
not opposite to each other. Two seats are there between Hemanika and Janvi, who
sits third right of the one who likes Nivea. The one who likes Medimix Soap faces
the one who likes Liril. The friends who like the Hamam and Mysore Sandal are
adjacent to each other. Vacant seat of row – 1 is not an immediate neighbour of
Preeti. Radha sits at one of the extreme ends of the row. Charmy does not like
Hamam and Mysore Sandal. Vacant seat of row-1 does not face Nisha who doesn’t
sit at any of the extreme ends of the row.

6). How many people will sit between Veni and Radha?
One
Two
Three
Four
More than four
7). Which of the following brand Usha likes?
a) Margo b) Liril c) Nivea d) Dove e) Medimix

8). Who among the following likes Pears?


Usha
Nisha
Radha
Preeti
None of these

9). Four among the following form a group in a certain way. Which of the
following does not belong to Group?
Dharini, Margo
Veni, Lux
Radha, Vivel
Janvi, Medimix
Usha, Liril

10). Which of the following is correctly matched?


Preeti – Margo
Veni – Vivel
Usha – Nivea
Radha – Medimix
Charmy – Liril
PUZZLE-202
Directions (Q. 1-5): Study the following information and answer the given questions.
Seven people, namely, Pavi, Panav, Pavin, Punit, Poorna, Prithvi and Prajin
like seven different cartoon characters, namely Tweety, Wiene. Superman,
Garfield, Jerry, Ariel and Popeye. Each of them studies in one of the three
standards, viz IV, VIII and XI with at least two of them in a standard.
(Note: None of the information given is necessarily in the same order.)

Only one person studies with Punit in XI standard. Panav studies with the
one who likes Ariel. Punit does not like Ariel. Prajin studies only with the one who
likes Garfield. Neither Panav nor Punit likes Garfield. Pavi likes Popeye. Pavi does
not study with Panav. The one who likes Tweety studies with Pavi. Poorna studies
with the one who likes Jerry. Poorna does not study in IV standard. Neither Panav
nor Prithvi likes Jerry. The one who likes Superman studies with Poorna.

1). Who among the-following likes Wiene?


Poorna
Prithvi
Punit
Prajin
Panav

2). Which of the following cartoon characters does Pavin like?


Superman
Ariel
Tweety
Jerry
Garfield

3). Which of the following statements is true?


Both Punit and Pavi study in the same standard.
Panav studies in IV standard.
Pavi studies with the one who likes Wiene
More than two people study in the standard in which Prithvi studies.
None of the given statements is true

4). Which of the following combinations represents the combination of


people studying in VIII standard?
Poorna and the one who likes Wiene
Pavi, Prithvi
Panav, Pavin, Poorna
Panav and the one who likes Tweety
None of these

5). Four of the following five are alike in a certain way based on the given
arrangement and hence form a group. Which of the following does not belong to
that group?
Prajin, Punit
Pavin, Panav
Panav, Poorna
Pavin, Poorna
Prithvi, Prajin

6). Which of the following combinations represents the standard in which


Prithvi studies and the cartoon character she likes'?
VIII - Garfield
VIII - Superman
XI - Tweety
IV - Ariel
IV - Garfield
JOIN TELEGRAM CLICK HERE

To help you in your exam


preparation, AIMBANKER TEAM is
regularly posting helpful posts,
please subscribe, follow & Share
,LIKE US ON FACEBOOK PAGE
OF AIMBANKER.

JOIN FACEBOOK CLICK HERE

Join Whatsapp @Click Here

8/8

You might also like